Sie sind auf Seite 1von 302

bankersadda.

com

http://www.bankersadda.com/2015/08/banking-pathway-2015-quant-quiz_6.html

Banking Pathway : 2015 Quant Quiz


Directions (1- 5): Read the following graph carefully and answer the questions given below.
Production of Tea and Coffee by a factory in different years.

1.What was approximate percentage increase in quantity of exported tea in 2009 in comparison to
previous year?
(1) 42
(2) 37
(3) 34
(4) 45
(5) 49
2.What is the average quantity of exported coffee in 2008 and 2010? (in kg.)
(1) 975
(2) 965
(3) 935
(4) 948
(5) None of these
3.The quantity of exported coffee in 2007 is approximately what percentage of the quantity of exported
tea in the same year?
(1) 57

(2) 53
(3) 66
(4) 70
(5) 60
4.What is the difference between the total quantity of exported tea in 2007 and 2208 and total quantity of
exported coffee in both the same year? (in kg.)
(1) 408
(2) 412
(3) 416
(4) 422
(5) None of these
5.What is the total quantity of exported coffee in all the years? (in kg.)
(1) 9330
(2) 9545
(3) 9214
(4) 9528
(5) None of these
6.The compound interest accured on an amount of Rs. 25,500 at the end of three years in Rs. 8,440.50.
what would be the simple interest accured on the same amount at the same rate in the same period?
(1) Rs. 4,650
(2) Rs. 5,650
(3) Rs. 6,650
(4) Rs. 7,650
(5) None of these
7.The profit earned after selling an article of Rs. 1,754 is the same as the loss incurred after selling the
article for Rs. 1,492. What is the cost price of the article?
(1) Rs. 1,623
(2) Rs. 6,192
(3) Rs. 6,708
(4) Rs. 7,224
(5) None of these
8.A sum of money is delivered among A, B, C and D in the ratio of 3:4:9:10. If the share of C is Rs. 2,580
more than the share of B, what is the total amount of money of A and D together?
(1) Rs. 5,676
(2) Rs. 6,192
(3) Rs. 6,708
(4) RS. 7,224
(5) None of these
9.Bovina spent Rs. 44,668 on her air tickets, Rs. 56,732 on buying gift for the family members and the
remaining 22% of the total amount she had as cash with her. What was the total amount?
(1) RS. 28,600
(2) Rs. 1,30,000
(3) Rs. 1,01,400
(4) RS. 33,800
(5) None of these
10.Rubina decided to donate 16% of her monthly salary to an NGO. On the day of donation she changed

her mind and donated RS. 6,567. Which was 75% of what she had decided earlier. How much is Rubinas
monthly salary?
(1) Rs. 8,756
(2) Rs.54,725
(3) Rs. 6,56,700
(4) Rs. 45,696
(5) None of these
Answers
Solution
1.(4) Increase percent = 451/1014 * 100 = 45(Approximate)
2.(1)
3.(3) Required % = 1104/1671 * 100 = 66(Approximate)
4.(5)
5.(2)
6.(4) R % = 33940.5*100/255000 = 33.1 %
R % for 1 Year = 10 %
S.I = 25500*10*3/100 = 7650
7.(1) Required CP = (1754 +1492)/2 = 1623
8.(3) 5 = 2580
13 = 2580*13/5 = 6708
9.(2) 78 % 101400
100 % = 101400*100/78 = 130000
10.(2) 75 % = 6567
100 % = 6567 *4/3
16 % = 6567 *4/3
100 % = 6567 *4/3*100/16 = 54725

bankersadda.com

http://www.bankersadda.com/2015/08/banking-pathway-2015-quant-quiz_14.html

Banking Pathway : 2015 Quant Quiz


1.The present ages of three persons in proportions 4 : 7 : 9. Eight years ago, the sum of their ages was 56.
Find their present ages (in years).
a.8,20 ,28
b.20 ,35 ,45
c.16,28,36
d.12 ,24 ,36
e.None of the above

2.A and B can do a job together in 7 days. A is 7/4 times as efficient as B. The same job can be done by A
alone in :
a.28/3 Days
b.11 days
c.49/4 Days
d.49/3 days
e.None of the above
3.Twenty Girls can do a work in sixteen days. Sixteen Boys can complete the same work in fifteen days.
What is the ratio between the capacity of a Boys and a Girls?
a.3:4
b.4:3
c.5:3
d.1 :3
e.None of these
4.The least number, which when divided by 12, 15, 20 and 54 leaves in each case a remainder of 8 is:
a.504
b.536
c.544
d.548
e.600
5.The average weight of X, Y and Z is 45 kg. If the average weight of X and Y be 40 kg and that of Y and Z
be 43 kg, then the weight of Y is:
a.17 kg
b.20 kg
c.26 kg
d.31 kg
e.34 kg
6.Two taps A and B can fill a cistern in 15 and 30 minutes respectively. There is a third exhaust tap C at
the bottom of tank. If all taps are opened at the same time, the cistern will be full in 25 minutes. In what
time can exhaust tap C empty the cistern when full?
a.10 Min
b.12 Min
c.11 Min
d.8 Min
e.14 Min

7.Vijay Wada is at a distance of 340 km from Hyderabad. A train starts from Hyderabad to Vijay Wada at 4
A.M. with a speed of 60 km/hr. Another train starts from Vijay Wada to Hyderabad at 5 A.M. with a speed of
80 km/hr. At what distance from Hyderabad will the two trains cross each othe r and also find the time
when they cross each other?
a.6.30 AM
b.7.15 AM
c.7.00 AM
d7.30 AM
e.8.00 AM
8.A alone can do a piece of work in 6 days and B alone in 8 days. A and B undertook to do it for Rs. 3200.
With the help of C, they completed the work in 3 days. How much is to be paid to C
a.Rs.300
b.Rs. 400
c.Rs. 500
d.Rs. 600
e.Rs. 450
9.A contractor undertakes to do a piece of work in 150 days, so he deployed 200 men. He found that only
a quarter of work is done in 50 days. How many additional men must be deployed so that the work may be
finished on time?
a.Rs.105
b.Rs. 110
c.Rs. 95
d.Rs. 100
e.Rs. 120
10.A rectangular park 60 m long and 40 m wide has two concrete crossroads running in the middle of the
park and rest of the park has been used as a lawn. If the area of the lawn is 2109 sq. m, then what is the
width of the road?
a.2.91 M
b.5.82 M
c.3.0 M
d.3.5 M
e.None of these
Answers :
1.C
Let their present ages be 4x, 7x and 9x years respectively.
Then, (4x - 8) + (7x - 8) + (9x - 8) = 56
20x = 80
x = 4.
Their present ages are 4x = 16 years, 7x = 28 years and 9x = 36 years respectively.
2.B
(A's 1 day's work) : (B's 1 day's work) =7/4 :1 = 7:4
Let A's and B's 1 day's work be 7x and 4x respectively.
Then, 7x + 4x =1/7 x = 1/77
A's 1 day's work = 1 X7 /77 = 1/11
= 11 Days
3.B
(20 x 16) Girls can complete the work in 1 day.

1 Girl's 1 day's work = 1/320


(16 x 15) Boy can complete the work in 1 day.
1 Boy s 1 day's work = 1/240
Req. ratio = 1/240 : 1/320
= 4:3
4.D Required number = (L.C.M. of 12, 15, 20, 54) + 8
= 540 + 8 = 548.
5.D
X + Y + Z = (45 x 3) = 135 .... (i)
X + Y = (40 x 2) = 80 .... (ii)
Y + Z = (43 x 2) = 86 ....(iii)
Adding (ii) and (iii), we get: X + 2Y + Z = 166 .... (iv)
Subtracting (i) from (iv), we get : Y = 31.
Y's weight = 31 kg.
6.A
Here, X = 15, Y = 30 and Z = 25
C can empty the full tank in = (XYZ /YZ +ZX XY) Minutes
= ( 15*30*25/ ( 30*25 +15*25 -15*30)
= 11250/1125 = 10 Minutes

7.C
s1 = 60, s2 = 80, T = time from 4 A.M. to 5 A.M. = 1 hr
Distance of meeting point from Hyderabad = S1 {( d +S2 t) / (S1 +S2)} km
= 60 {( 340 + 80X1) / (60+80)} = 180 km
Time of their meeting = {( d +S2 t) / (S1 +S2)}
{( 340 + 80X1) / (60+80)}
3 Hrs after 4 o clock
@ 7:00 AM

8.B
C's 1 day's work =
1/3( 1/6+ 1/8) = (1 / 3 7 /24)
=1/ 24
= A:B:C=1/ 6:1/8:1/ 24

=4:3:1
Cs Share=3200 * 1/8 =400
9.D
200men x 50Days = 1/4 of work ,
so Total work = 200*50*4.
Now Let extra Men required is x
Therefore, (200+x)*100(remaining Days) =3/4 of work (That is remaining work).
So , (200+x)*100 = 200*50*4*(3/4)
=> 200+x=300
=> x= 100

10. C

bankersadda.com

http://www.bankersadda.com/2015/07/banking-pathway-quant-di.html

Banking Pathway : Quant (DI )


Directions (1-5): Study the graph carefully to answer the questions that follow.
Income (in lakhs) of three companies over the years
per cent profit =(Incom Expenditure)*100/Expenditure

1.If the per cent profit of Company A in the year 2002 was 20, what was its expenditure in that year?
(1) Rs. 250000
(2) Rs. 275000
(3) Rs. 175000
(4) Rs. 150000
(5) None of the above
2.If the expenditure of Company C in 2003 was Rs. 1.75 lakh, what was its per cent profit in that year?
(rounded off to two digits after decimal)
(1) 38.29
(2) 42.86
(3) 53.41
(4) 58.64
(5) None of the above
3.What is the average Income of Company A over the years?
(1) Rs. 275000
(2) Rs. 3000000
(3) Rs. 2750000
(4) Rs. 30000
(5) None of the above
4.What is the approximate percent increase in income of company B in the year 2006 from the previous
years?
(1) 28
(2) 11
(3) 17
(4) 22
(5) None of the above

5.Percent increase/decrease in income of company C was highest for which year?


(1) 2004
(2) 2006
(3) 2003
(4) 2002
(5) None of the aboves
Directions (6-10): Study the graph carefully to answer the questions that follow.
Number of students enrolled in three different disciplines in in five different colleges

6.Number of students studying B.Com. in college C forms approximately what per cent of the total
number of students studying B.Com. in all the colleges together?
(1) 39%
(2) 21%
(3) 44%
(4) 52%
(5) None of the above
7.What is the respective ratio of total number of students studying B.Sc., B.A and B.Com.in all the
colleges together?
(1) 71 : 67 : 75
(2) 67 : 71 : 75
(3) 71 : 68 : 75
(4) 66 : 73 : 79
(5) None of the above
8.What is the total number of students studying B.Sc. in all the colleges together?
(1) 1825
(2) 1775
(3) 1650
(4) 1975
(5) None of the these
9.Number of students studying B.A. in college B forms what per cent of total number of students
studying all the disciplines together in the college? (rounded off to two digits after decimal)
(1) 26.86%
(2) 27.27%
(3) 29.84%
(4) 28.27%
(5) None of these

10.What is the respective ratio of total number of students studying B.Sc. in the colleges C and E
together to those studying B.A. in the same colleges together?
(1) 24 : 23
(2) 25 : 27
(3) 29 : 23
(4) 29 : 27
(5) None of these
Answers with Explanation..
1. (1) If the expenditure = x lakhs
So, 20 = (3-x)/x *100
x = 250000
2. (2) Percentage profit = (2.5 1.75)/1.75*100
= 42.85
3. (5)Required average = 1/6(2.9 + 3 + 3.5 + 3 +2.5 + 3.25) lakhs
= 302500
4. (3)
5. (1)
6. (5) No of students in B.Com in college C = 350
Total no of students in B.Com in all the colleges
= 425 + 475 + 350 + 400 + 225 = 1875
Required % == (350/1875)*100 = 18.66
7. (1)
8. (2)
9. (2) No of students in BA is college B = 300
No of students studying all the disciplines in college B = 300 + 325 + 475 = 11000
Required % = 300/1100X100= 27.27%
10. (4) No of Students in B.Scin college C & E together =300 +425 = 725
No of students in B.A. in college C & E together
= 350+325=675
Required ratio = 725 : 675 = 29 : 27

To view the above post in hindi click here

bankersadda.com

http://www.bankersadda.com/2015/07/banking-pathway-2015-quant-di-quiz.html

Banking Pathway 2015 : Quant ( DI Quiz)


Q.1-5.Study the following graph carefully and answer the questionsgiven below it.
The following graphs represent percentage of five types of employees in an organization for two
consecutive years.

Total number of workers in 2008 =12000 and Total numbers of workers in 2009 = 15000
Q.1. If 400 workers joined A type of organization in 2009. How many workers ofC type of organization
joined the company in that year ?
(1) 350
(2) 260
(3) 408
(4) 460
(5) Data inadequate
Q.2. If 360 E type of workers left the company at the end of 2008, how many E type of workers joined the
company 2009?
(1) 850
(2) 1020
(3) 1080
(4) 980 (5) None of these

Q.3. If 600 A type of workers joined the company in 2009, how many workers of A type left the company in
2008 ?
(1) 1420
(2) 1540
(3) 1560
(4) 1350
(5) None of these
Q.4.In terms of number the variation of which of the following types of employees was minimum between
2008 & 2009?
(1) A
(2) B
(3) C
(4) D
(5) E
Q.5. What was the difference between D type of workers in 2008 and B type of workers in 2009?
(1) 550
(2) 500
(3) 630
(4) 620
(5) None of these
(6-10) the following informations carefully and answer the questions given below.
In India, the total number of tourists in a tourist place is 800 who are from different countries viz, USA, Italy, China,
Australia and Mauritius. 54% of the total numbers of tourists are male. One-fourth of the total numbers of females
are from Australia. 25% of the total number of females is from USA. The number of females from Mauritius is half
of the number of females from USA.5/6 of the remaining number of females are from Italy. Total number of tourists
from Australia is 192. One-fourth of the total numbers of males are from China. 100 males tourists are from Italy.
Three-fourth of the remaining number of males are from Maurtius.
Q.6. What is the respective ratio between the number of female tourists from USA and number of male
tourists from Australia?
(1) 23:24
(2) 23:50
(3) 23:25
(4) 20:23
(5) None of these
Q.7 What is the approximately average number of male tourists from USA, China and Mauritius?
(1) 82
(2) 77
(3) 86
(4) 72
(5) 67
Q.8 What is the two-third of the total number of female tourists from China and Mauritius?
(1) 46
(2) 48
(3) 54
(4) 58
(5) None of these
Q.9. The total number of tourists from Mauritius is approximately what percent of the female tourists from

Italy?
(1) 90
(2) 80
(3) 120
(4) 70
(5) 150
Q.10. What is the total number of tourists from China?
(1) 121
(2) 143
(3) 111
(4) 153
(5) None of these
Answers with Explanation..
1.(5) Data inadequate
2.(3) Number of E type of workers who joined the company in 2009
= 16% of 15000 (14% of 12000 360)
= 2400 (1680 360)
= 2400 1320
= 1080
3.(3) 15000X24/100 + 600 - 12000X22/100
(3600 +600 ) -2640 =1560
4.(3)
5.(5) Required difference = 3600 3000 = 600
6.(3)Ratio = 92 : 100 = 23 : 25
7.(2). Average numbers = ( 93 + 108 + 31 )/3 = 77 approx
8.(1).Required numbers = (23 + 46 )X2/3 = 46
9.(3) Required percentage = (139/115)X100 = 120 approx

10.(5)

bankersadda.com

http://www.bankersadda.com/2015/08/banking-pathway-2015-quant-quiz_12.html

Banking Pathway 2015 : Quant Quiz


Directions (1-3) In each of the following questions What will come in place of ____ in the following
questions
1.6 , 3 , 3 , 4.5 , 9 , ?
(1)22.5
(2)25
(3)30
(4)32.5
(5) 27
2.9 , 28 , 65 , 126 , ?
(1)234
(2)230
(3)217
(4)216
(5)220
3.1 ,5 , 3 , 9 , 5, 13 ,7 , 17,11, 21 ,?
(1)13
(2)23
(3)19
(4)23
(5)none of these
4.A can finish a work in 18 days and B can do same work in half the time taken by A. then working
together, what part of same work they can finish in a day
(1)1\5
(2)1\6
(3)1\7
(4)1\8
(5)none of these
5.A is twice as good as workman as B and together they finish a piece of work in 18 days. In how many
days will B alone finish the work.
(1)27 days
(2)54 days
(3)56 days
(4)68 days
(5)none of these
Directions (6-10) Study the following graphs carefully and answer the questions that follow: Distribution
of proteins in human body and second pie chart show the distribution of elements in human body.

6.What is the ratio of distribution of proteins in the muscles to


the distribution of proteins in bones?
(1) 1: 2
(2) 2: 1
(3) 18:1
(4) 1: 18

7.What percent to the total weight of the human body is


equivalent of the weight of the skin in the human body?
(1) 0.016
(2) 1.6
(3) 0.16
(4) Data Insufficient
(5) None of these
8.To show the distribution of proteins and other dry elements in the
human body, the arc of the circle should subtend at the centre an
angle of
(1) 1 2 6
(2) 5 4
(3) 1 0 8
(4) 2 5 2
(5) None of these
9. What will be the quantity of water in the body of a person weighing
50 kg?
(1) 35 kg
(2) 120 kg
(3) 71.42 kg
(4) 20 kg
(5) None of these
10.What part of the human body is made of neither bones nor skin?
(1)2/5
(2)11/15
(3)1/40
(4)3/80
(5) None of these
Answers :
Solution
1.(1) 1/2,1,3/2..
2.(3) 2^3+1,3^3+1,......
3.(1) Two series mixed ,one prime no series,so next no will be 13
4.(2)
5.(2)Let A efficiency=2 & B efficiency=1
Total work=(2+1)*18=54

So A can do a work in 18 days


6.(2) 1/3:1/6 = 2:1

7.(2)
8.(3) 100 % = 360
30 % = 360/100*30 = 108

9.(1) 70 % of 50 = 35 kg

10.(2) 1 - (1/6 +1/10) = 11/15

bankersadda.com

http://www.bankersadda.com/2015/07/banking-pathway-2015-quant-quiz.html

Banking Pathway 2015 : Quant Quiz


Dear Readers,
In continuance to the Banking Pathway 2015, we are providing you a Quant Mix quiz that can be helpful for all
the exam including IBPS CWE-RRB,/ IBPS CWE-PO /RBI Assistant and many more.
1. A sum of money is divided among A,B,C and D in the ratio of 3 : 5 : 9 : 13 respectively. If the share of C
is Rs. 2412 more than the share of A, then what is the total amount of money of B and D together?
A) Rs. 4422
B) Rs. 7236
C) Rs. 6030
D) Rs. 4824
E) None of these
2. The average age of 80 girls was 20 years, the average age of 20 of them was 22 years and that of
another 20 was 24 years. Find the average age of the remaining girls.
A) 17 years
B) 19 years
C) 21 years
D) 15 years
E) None of these
3. Ms. Pooja invests 13% of her monthly salary, i.e. Rs. 8554 in Mediclam Policies. Later she invests 23%
of her monthly salary on child Education Policies; also she invests another 8% of her monthly salary on
Mutual Funds. What is the total annual invested by Ms. Pooja?
A) Rs. 28952
B) Rs. 43428
C) Rs. 347424
D) Rs. 173712
E) None of these
4. The profit earned after selling an article for Rs. 878 is the same as loss incurred after selling the article
for Rs. 636. What is the cost price of the article?
A) Rs. 797
B) Rs. 787
C) Rs. 767
D) Rs. 757
E) None of these
5. In a class of 240 students, each student got sweets that are 15% of the total number of students. How
many sweets were there?
A) 3000
B) 3125
C) 8640
D) 9789
E) None of these.
6. Sonika spent Rs. 45, 760 on the interior decoration for her home, Rs. 27896 on buying air conditioner
and the remaining 28% of the total amount she had as cash with her. What was the total amount?
A) Rs. 98540
B) Rs. 102300

C) Rs. 134560
D) Cannot be determined
E) None of these
7. The ages of Khushi and Jagriti are in the ratio of 5 : 8 respectively. After 8 years the ratio of their ages
will be 3 : 4. What is the difference in their ages?
A) 16 years
B) 8 years
C) 10 years
D) 12 years
E) None of these
8. By how much is 6/11 th of 506 lesser than 3/5 of 895?
A) 178
B) 219
C) 143
D) 261
E) None of these
9. The average of the ages of Sumit, Krishna and Rishabh is 43 years and the average of the ages of
Sumit, Rishabh and Rohit is 49 years. If Rohit is 54 years old, what is Krishna'a age?
A) 45 years
B) 24 years
C) 36 years
D) Cannot be determined
E) None of these
10. The compound interest occured on an amount at the end of three years at 15 p.c.p.a is 6500.52. What
is the amount?
A) Rs. 12480
B) Rs. 10500
C) Rs. 14800
D) Rs. 13620
E) None of these
11. P liters of oil were poured into a tank and it was still r% empty How much oil must be poured into the
tank in order to fill it to the brim?
A) (P r)/(100-r)
B) P/(100-r)
C) 100P/(100-r)
D) 100Pr/ (100-r)
E) None of these
12. A sum of money is accumulating at compound interest at a certain rate of interest. If Simple interest
instead of compound interest were reckoned, the interest for the first two years would be diminished by
Rs. 20 and that for the first three years by Rs. 61. Find the Sum?
A) 8000
B) 6000
C) 10,000
D) 12,000
E) None of these
13. In Utkarshs opinion, his weight is greater than 65 kg but less than 72 kg. His brother doesnt agree
with Utkarsh and he thinks that Utkarshs weight is greater than 60 kg but less than 70 kg. His mother's

view is that his weight cannot be greater than 68 kg. If all are them are correct in their estimation, what is
the average of different probable weights of Utkarsh?
A) 67 kg
B) 68 kg
C) 69 kg.
D) Data inadequate
E) None of these
14. Three pipes A, B and C can fill a tank from empty to full in 30 minutes, 20 minutes, and 10 minutes
respectively. When the tank is empty, all the three pipes are opened. A, B and C discharge chemical
solutions P,Q and R respectively. What is the proportion of the solution R in the liquid in the tank after 3
minutes?
A) 5/11
B) 7/11
C) 9/11
D) 3/11
E) None of these
15. A and B can complete a work in 15 days and 10 days respectively. They started doing the work
together but after 2 days B had to leave and A alone completed the remaining work. The whole work was
completed in :
A) 10 days
B) 12 days
C) 20 days
D) 25 days
E) None of these
ANSWERS & SOLUTIONS
1. B
2. A
3. C
4. D
5. C
6. B
7. E
8. D
9. C
10. A
11. A
12. A
13. A
14. E
15. B
1.(B) Let the original sum be Rs. x.
Sum of the Ratios = 3 + 5 + 9 + 13 = 30
C's share = Rs. 9x/30 = Rs. 3x/10
A's share = Rs. 3x/30 = Rs. x/10
According to the question,
3x/10 - x/10 = 2412
2x/10 = 2412
x = 2412 * 5 = Rs 12060
Amount received by B and D together
= Rs. (((5+13))/30 12060)

= 7236
2.(A) Total age of remaining 40 girls
= (80 * 20 - 20 * 22 - 20 * 24) years
= (1600 - 440 - 480) years
= 680 years
Required average age = 680/40 = 17 years
3.(C) Let Ms. Pooja's monthly salary be Rs. x
According to the question,
13% of x = Rs. 8554
= x = Rs. (8554 * 100)/13 = Rs. 65800
Total monthly investment in percentage
= 13 + 23 + 8 = 44
Total montly investment = 44% of Rs. 65800
= (44 * 65800)/100
= Rs. 28952
Total annul investment = Rs. (12 * 28952)
= Rs. 347424
4. (D) Let the CP of the article be Rs. x.
According to the question,
878 - x = x - 636
= 2x = 878 + 636 = 1514
x = 1514/2 = Rs. 757
5. (C) Number of sweets received by each student
= 15% of 240
= 15 * 240/100 = 36
Total number of sweets = 240 * 36 = 8640
6. (B) Let the total amount be Rs. x
The amount spent = 100 - 28 = 72%
72% of x = Rs. (45760 + 27896)
= 72 * x /100 = 73656
x = 73656 * 100/72 = Rs. 102300.
7.(E) Let the present ages of Khushi and Jagriti be 5x and 8x years respectively.
After 8 years,
5x + 8/8x + 8 = 3/4
= 24x + 24 = 20x + 32
= 4x = 32 - 24 = 8
x = 8/4 = 2
Required difference = (8x - 5x) years
= 3x = 3 * 2 = 6 years
8. (D) Required difference = 3/5 * 895 - 6/11 * 506
= 537 - 276 = 261
9. (C) (Sumit + Krishna + Rishabh)'s age
= 43 * 3 = 129 years .........................(i)
(Sumit + Rishabh + Rohit)'s age
= 3 * 49 = 147 years .........................(ii)
By equation (i) - (ii), we have,
Krishan's age - Rohit's age = 129 - 147 = -18

Krishna's age = 54 - 18 = 36 years


10. (A) Let the principal be Rs. P.
According to the question,
CI = P [(1+ R/100)^T- 1]
6500.52 = P [(1+ 15/100)^3- 1]
= 6500.52 = P [1.520875 -1]
P = 6500.52/0.520875 = Rs. 12480
11. (A) After filling P liter still r % empty. So (100-r)% = P liter ;
Let x liter require to fill remain tank.
So full capacity of tank = 100P/(100-r)
So x= r% of capacity = 100P/(100-r) * r/100 = Pr/(100-r)
12. (A) P(r/100)^2= 20 (1)
P(r/100)^2 (300+r)/100 =61 -(2)
Putting value from eqn 1
(300+r)/100 =61/20
On solving r = 5%
After put value in eqn(1) P= 8000
13. (A) Go through statements, its clear that average of probable weight of Utkarsh is 67 kg.
14. (E) Part filled by A+B+C in 3 min = 3(1/20+1/30+1/10) = 11/20
Part filled by C in 10 min = 3/10
Required ratio = 3/10:11/20 = 6:11
So the solution R in the liquid in the tank after 3 minutes = 6/11
15. (B) A and B together 1 day work =1/15+1/10 =5/30=1/6
Both work for 2 day = 1/3
Remain work =2/3
Left work done in (2/3) (1/15) =10 days
So total day = 10+2 = 12 days

bankersadda.com

http://www.bankersadda.com/2015/08/banking-pathway-2015-quant-quiz_11.html

Banking Pathway 2015 : Quant Quiz


Directions (1-5): In the following number series one number is wrong. Find out the wrong number.
1. 5 348 564
(1) 716
(2) 788
(3) 348
(4)689
(5) 780

689 716 780

788

2. 444 2224 1114 556 281.5 142.75 73.375


(1)2224
(2) 2815
(3) 1114
(4) 556
(5) 14275
3.4.5
(1) 33
(2) 385
(3) 42
(4) 435
(5) 25

16

4.180 181
(1) 181
(2) 210
(3) 1369
(4) 676
(5)1405
5.38
567
(1) 567
(2) 998
(3) 139
(4)1658
(5) 1883

25

210

333

998

33

38.5

676

1369 1658

42

43.5

1405

1883

6.What will be the approximate difference in the simple and compound interest accrued on an amount of
Rs. 2,600/- @ 15 p.c.p.a. at the end of three years ?
(1) Rs.l67/(2) Rs. 194/(3) Rs. 202/(4) Rs. 172/(5) Rs. 184/7.A 320 metre long train crosses a platform thrice its length in 40 seconds. What is the speed of the train
in km/hour?
(1) 120.6
(2) 115.2

(3) 108.4
(4) Cannot be determined
(5) None of these
Directions (8-10): Study the following information carefully to answer the question that follow.
A committee of ve members is to be formed out of 4 students, 3 teachers and 2 sports coaches. In how
many ways can the committee be formed if 8.The committee should consist of 2 students, 2 teachers and 1 sports coach?
(1)256
(2)64
(3)9
(4)36
(5)None of these
9.Any ve people can be selected?
(1)126
(2)45
(3)120
(4)24
(5) None of these
10.In how many different ways can the letters of the word LEASE be arranged?
(1) 240
(2) 120
(3) 25
(4) 60
(5) None of these
Answers
1.(1) difference between the numbers 343 , 216 , 125 ,
2(4) series is 1/2 + 2 ,1/2 + 2 ,1/2 + 2 .....
3(5)
4 (2) difference between the numbers 1 , 27 , 125 ,343
5(2) difference between the numbers 529 , 441 , 361 ,289
6(5) Difference = 2600*15*15 (300 + 15)/100^3 = 184
7(2) Required Speed = (320 + 3*320)/40 = 32 m/s
8(4) 4C1 X 3C2 X 2C1 = 36
9 (1) 9C5 = 126
10 (4) 5!/2! = 601.(1) difference between the numbers 343 , 216 , 125 ,

bankersadda.com

http://www.bankersadda.com/2015/08/banking-pathway-2015-quant-quiz_43.html

Banking Pathway 2015 : Quant Quiz


1.Mehuls monthly salary is one and a half times Shaileshs monthly salary. Prashants monthly salary is
ve-fourth Mehuls monthly salary. If the total of the monthly salaries of all the three is Rs. 1,83,750/-, what
is Prashants monthly salary?
(1)Rs.42,000/(2)Rs68,500/(3)Rs.78,750/(4)Rs.63,000/(5)None of these

2.On the annual day, sweets were to be distributed equally amongst 600 children of the school. But on
that particular day, 120 children remained absent. Thus, each child got 2 extra sweets. How many sweets
was each child originally supposed to get?
(1) 8
(2) 14
(3) 10
(4) 6
(5) None of these
3.Gold is 19 times as heavy as water and copper is 9 times as heavy as water. In What ratio should these
be mixed to get an alloy 15 times as heavy as water?
(1) 1 : 1
(2) 1 : 2
(3) 2 : 3
(4) 3 : 2
(5) None of these
4.The average age of boys in the class is twice the number of girls in the class. The ratio of boys and girls
in the class of 50 is 4 : 1. The total of the ages (in years) of the boys in the class is :
(1) 2000
(2) 2500
(3) 800
(4) 400
(5) None of these
5.There are 100 students in 3 sections A, B and C of a class. The average marks of all the 3 sections was
84. The average of Band C was 87.5 and the average marks of A is 70. The number of students in A was:
(1) 30
(2) 35
(3) 20
(4) 25
(5) None of these
6.A sold an article to B at 20% profit and B sold it to C at 15% loss. If A sold it to C at 15% loss. If A sold it
to C at the selling price of B, then A would make :
(1) 5 % profit
(2) 2% profit
(3) 2% loss
(4) 5% loss
(5) None of these

7.The monthly salaries of A and B together amount to Rs. 40,000/-. A spends 85% of his salary and B, 95%
of his salary. If now their savings are the same, then the salary (in Rs.) of A is :
(1) Rs. 10,000/(2) Rs. l2,000/(3) Rs. l6,000/(4) Rs. l8,000/(5) None of these
8.It takes 8 hours for a 600 km journey, if 120 km is done by train and the rest by car. It takes 20 min more
if 200 km is done by train and the rest by car. The ratio of the speed of the train to that of the car is:
(1) 2 : 3
(2) 3 : 2
(3) 3 : 4
(4) 4 : 3
(5) None of these
9.If a train runs at 70 km/hour, it reaches its destination late by 12 min. But if it runs at 80 km/hr, it is late
by 3 min. The correct time to cover the journey is :
(1) 58 min
(2) 2 hr
(3) 1 hr
(4) 59 min
(5) None of these
10.A man borrowed some money from a private organisation at 5% simple interest per annum. He lended
50% of his money to another person at 10% compound interest per annum and thereby the man made a
profit of Rs. 3,205/- in 4 years. The man borrowed:
(1) Rs. 80,000/(2) Rs. 1, 00, 000/(3) Rs. l, 20, 000/(4) Rs. 1, 50, 000/(5) None of these
Answers with Explanation

bankersadda.com

http://www.bankersadda.com/2015/08/banking-pathway-2015-quant-quiz_7.html

Banking Pathway 2015 : Quant Quiz


1.What is the difference between the compound interests on Rs. 5000 for 1.5 years at 4% per annum
compounded yearly and half-yearly?
(1)Rs. 2.04
(2)Rs. 3.06
(3)Rs. 4.80
(4)Rs. 8.30
(5)none of these
2.The compound interest on Rs. 30,000 at 7% per annum is Rs. 4347. The period (in years) is:
(1)2
(2)5
(3)8
(4)2.5
(5) none of these
3.The length of a rectangle is halved, while its breadth is tripled. What is the percentage change in area?
(1)25% increase
(2)50% increase
(3)50% decrease .
(4)75% decrease
(5)none of these
4.The length of a rectangular plot is 20 metres more than its breadth. If the cost of fencing the plot @
26.50 per metre is Rs. 5300, what is the length of the plot in metres?
(1)40
(2)50
(3)120
(4)Data inadequate
(5)None of these
5.In how many different ways can the letters of the word 'CORPORATION' be arranged so that the vowels
always come together?
(1)810
(2)1440
(3)2880
(4)50400
(5)5760
6.In how many ways can the letters of the word 'LEADER' be arranged?
(1)720
(2)360
(3)840
(4)368
(5) none of these
7.Tickets numbered 1 to 20 are mixed up and then a ticket is drawn at random. What is the probability that
the ticket drawn has a number which is a multiple of 3 or 5?
(1)9/20
(2)5/4

(3)8/7
(4)2/9
(5)none of these
Directions (Qs 8-10): Each of the questions below consists of a question and three statements numbered
I ,II and III given below it. You have to decide whether the data provided in the statements are sufficient to
answer the question. Read the question and statements
8.What is the area of the hall?
I Material cost of flooring per square metre is Rs. 2.50
II Labour cost of flooring the hall is Rs. 3500
III Total cost of flooring the hall is Rs. 14,500.
(1)I and II only
(2)II and III only
(3) All I, II and III
(4)Any two of the three
(5)none of these
9.What is the area of a right-angled triangle?
I The perimeter of the triangle is 30 cm.
II The ratio between the base and the height of the triangle is 5 : 12.
IIIThe area of the triangle is equal to the area of a rectangle of length 10 cm.
(1)I and II only
(2)II and III only
(3)I and III only
(4)III, and either I or II only
(5) none of these
10.In a cricket team, the average age of eleven players in 28 years. What is the age of the captain?
I The captain is eleven years older than the youngest player.
IIThe average age of 10 players, other than the captain is 27.3 years.
III aving aside the captain and the youngest player, the average ages of three groups of three players each are 25
years, 28 years and 30 years respectively.
(1)Any two of the three
(2)All I, II and III
(3)II only or I and III only
(4)II and III only
(5)None of these
Answers

bankersadda.com

http://www.bankersadda.com/2015/08/banking-pathway-2015-quant-quiz_7.html

Banking Pathway 2015 : Quant Quiz


1.What is the difference between the compound interests on Rs. 5000 for 1.5 years at 4% per annum
compounded yearly and half-yearly?
(1)Rs. 2.04
(2)Rs. 3.06
(3)Rs. 4.80
(4)Rs. 8.30
(5)none of these
2.The compound interest on Rs. 30,000 at 7% per annum is Rs. 4347. The period (in years) is:
(1)2
(2)5
(3)8
(4)2.5
(5) none of these
3.The length of a rectangle is halved, while its breadth is tripled. What is the percentage change in area?
(1)25% increase
(2)50% increase
(3)50% decrease .
(4)75% decrease
(5)none of these
4.The length of a rectangular plot is 20 metres more than its breadth. If the cost of fencing the plot @
26.50 per metre is Rs. 5300, what is the length of the plot in metres?
(1)40
(2)50
(3)120
(4)Data inadequate
(5)None of these
5.In how many different ways can the letters of the word 'CORPORATION' be arranged so that the vowels
always come together?
(1)810
(2)1440
(3)2880
(4)50400
(5)5760
6.In how many ways can the letters of the word 'LEADER' be arranged?
(1)720
(2)360
(3)840
(4)368
(5) none of these
7.Tickets numbered 1 to 20 are mixed up and then a ticket is drawn at random. What is the probability that
the ticket drawn has a number which is a multiple of 3 or 5?
(1)9/20
(2)5/4

(3)8/7
(4)2/9
(5)none of these
Directions (Qs 8-10): Each of the questions below consists of a question and three statements numbered
I ,II and III given below it. You have to decide whether the data provided in the statements are sufficient to
answer the question. Read the question and statements
8.What is the area of the hall?
I Material cost of flooring per square metre is Rs. 2.50
II Labour cost of flooring the hall is Rs. 3500
III Total cost of flooring the hall is Rs. 14,500.
(1)I and II only
(2)II and III only
(3) All I, II and III
(4)Any two of the three
(5)none of these
9.What is the area of a right-angled triangle?
I The perimeter of the triangle is 30 cm.
II The ratio between the base and the height of the triangle is 5 : 12.
IIIThe area of the triangle is equal to the area of a rectangle of length 10 cm.
(1)I and II only
(2)II and III only
(3)I and III only
(4)III, and either I or II only
(5) none of these
10.In a cricket team, the average age of eleven players in 28 years. What is the age of the captain?
I The captain is eleven years older than the youngest player.
IIThe average age of 10 players, other than the captain is 27.3 years.
III aving aside the captain and the youngest player, the average ages of three groups of three players each are 25
years, 28 years and 30 years respectively.
(1)Any two of the three
(2)All I, II and III
(3)II only or I and III only
(4)II and III only
(5)None of these
Answers

bankersadda.com

http://www.bankersadda.com/2015/08/banking-pathway-2015-quant-quiz.html

Banking Pathway 2015 : Quant Quiz


1.The length of a rectangular plot is thrice its breadth. If the area of the rectangular plot is 7803 sq. meter,
what is the breadth of the rectangular plot?
(1) 51 meters
(2) 153 meters
(3) 104 meters
(4) 88 meters
(5) None of these
2.What would be the compound interest obtained on an amount of Rs. 20000 at the rate of 15 p.c.p.a. after
4 years?
(1) Rs. 14,980.125
(2) Rs. 19,680.125
(3) Rs. 16.780.125
(4) Rs. 18,980.125
(5) None of these
3.Mr.Duggal invested Rs. 20000 with rate of interest at 20 p.c.p.a. The interest was compounded half
yearly for first year and in the next year it was compounded yearly. What will be the total interest earned
at the end of two years?
(1) Rs. 8,800
(2) Rs. 9,040
(3) Rs. 8,040
(4) Rs. 9,800
(5) None of these
4.What will be the compound interest accrued on an amount of Rs. 10000 at the rate of 20 p.c.p.a. in two
years if the interest is compounded half yearly?
(1) Rs. 4400
(2) Rs. 4600
(3) Rs. 4641
(4) Rs. 4680
(5) None of these
5.One card is drawn at random from a pack of 52 cards. What is the probability that the card drawn is a
face card (Jack, Queen and King only)?
(1) 1/13
(2) 3/13
(3) 1/4
(4) 9/52
(5) None of these
6.A bag contains 6 black and 8 white balls. One ball is drawn at random. What is the probability that the
ball drawn is white?
(1) 3/4
(2) 4/7
(3) 1/8
(4) 3/7
(5) None of these

7.While selling a watch, a shopkeeper gives a discount of 15%. If he gives a discount of 20%, he earns
Rs. 51 less as profit. What is the original price of the watch?
(1) Rs. 920
(2) Rs. 985
(3) Rs. 1125
(4) Rs. 1020
(5) None of these
8.A shopkeeper sold a T.V. set for 17,940/-, with a discount of 8% and gained 19.6% If no discount is
allowed, what will he his gain per cent?
(1) 25%
(2) 26.4%
(3)24.8%
(4) Cannot be determined
(5) None of these
9.Prithvi spent Rs. 89,745 on his college fees. Rs.51,291 on Personality Development Classes and the
remaining 27% of the total amount he had as cash with him. What was the total amount?
(1) Rs.1,85,400
(2) Rs. 1,89,600
(3) Rs.1,91,800
(4) Rs.1,93,200
(5) None of these
10.In a class of 65 students and 4 teachers, each student got sweets that are 20% of the total number of
students and each teacher got sweets that are 40% of the total number of students. How many sweets
were there?
(1) 845
(2) 897
(3) 949
(4) 104
(5) None of these

bankersadda.com

http://www.bankersadda.com/2015/07/banking-pathway-2015-quant-quiz_31.html

Banking Pathway 2015 : Quant Quiz


1.Cost of 3 cricket balls = cost of 2 pairs of leg pads.
Cost of 3 pairs of leg pads = cost of 2 pairs of gloves.
Cost of 3 pairs of gloves = Cost of 2 cricket bats.
If a cricket bat costs Rs. 54, what is the cost of a cricket ball?
(a) Rs. 12
(b) Rs. 14
(c) Rs. 16
(d) Rs. 18
(e) none of these

2.Pure ghee costs Rs. 100 per kg. After adulterating it with vegetable oil costing Rs. 50 per kg, a
shopkeeper sells the mixture at the rate of Rs. 96 per kg, thereby making a profit of 20%. In what ratio
does he mix the two?
(a) 1 : 2
(b) 3 : 2
(c) 3 : 1
(d) 2 : 3
(e) None of these
3.Rs. 2189 are divided into three parts such that their amounts after 1, 2 and 3 years respectively may be
equal, the rate of simple interesting being 4% p.a. in all cases. The smallest part Is:
(a) Rs. 702
(b) Rs. 597
(c) Rs. 756
(d) Rs. 1093
(e) Rs. 729
4.If two pipes function simultaneously, the reservoir will be filled in 12 hours. One pipe fills the reservoir
10 hours faster than the other. How many hours it takes the second pipes to fill the reservoir?
(a) 25 hrs
(b) 28 hrs
(c) 30 hrs
(d) 35 hrs
(e) none of these
5.Bucket P has thrice the capacity as bucket Q. It takes 60 turns for bucket P to fill the empty drum. How
many turns it will take for both the buckets P and Q, having each turn together to fill the empty drum?
(a) 30
(b) 40
(c) 45
(d) 90
(e) 75
6.In a stream, B lies in between A and C such that it is equidistant from both A and C. A boat can go from A
to B and back in 6 h 30 minutes while it goes from A to C in 9 h. How long would it take to go from C to A?
(a) 3. 75 h
(b) 4 h

(c) 4.25 h
(d) 4.5 h
(e) none of these
7.The wheel of an engine is 3 meters in circumference and makes 4 revolutions in 2 seconds. The speed
of the train is
(a) 27 km/hr
(b) 31 km/hr
(c) 35 km/hr
(d) 24 km/hr
(e) None of these
8.Without any stoppage a person travels a certain distance at an average speed of 42km/hr and with
stoppages he covers the same distance at an average speed of 28 km/hr. How many minutes per hour
does he stop?
(a) 25 minutes
(b) 30 minutes
(c) 20 minutes
(d) 15 minutes
(e) None of these
9.A train overtakes two persons who are walking in the same direction in which the train is going, at the
rate of 2 kmph and 4 kmph and passes them completely in 9 and 10 seconds, respectively. The length of
the train is
(a) 72 m
(b) 54 m
(c) 50 m
(d) 45 m
(e) 100 m
10.Two stations A and B are 110 km apart on a straight line. One train starts from A at 7 a.m. and travels
towards B at 20 kmph. Another train starts from B at 8 a.m. and travels towards A at a speed of 25 kmph.
At what time will they meet?
(a) 9 a.m.
(b) 10 a.m.
(c) 11 a.m.
(d) 10.30 a.m.
(e) 10.15 a.m.

To solve above quiz in hindi click here

bankersadda.com

http://www.bankersadda.com/2015/07/banking-pathway-2015-quant-quiz_25.html

Banking Pathway 2015 : Quant Quiz


1. The price of shirts at Sahara Ganj is defined as Rs. (100 + 10 x2) where x is the number of shirts. Mallika
purchased 5 shirts from the above shop. The average price of a shirt is :
(a) Rs. 70
(b) Rs. 50
(c) Rs. 75
(d) Rs. 85
(e) none of these

2. There are 10 compartments in passenger train which carries on an average 20 passengers per
compartment. If at least 12 passengers were sitting in each compartment and no any compartment has
equal number of passengers then maximum how many passengers can be accommodated in any
compartment:
(a) 64
(b) 45
(c) 56
(d) 65
(e) none of these
3. There are twice the number of two wheelers as there are three wheelers and the number of 4 wheelers
are equal to the number of two wheelers. The average number of wheel per vehicle is :
(a) 3
(b) 4
(c) 5
(d) 7
(e) none of these
4. In a mixture of milk and water, there is only 26% water. After replacing the mixture with 7 litres of pure
milk, the percentage of milk in the mixture become 76%. The quantity of mixture is :
(a) 65 litre
(b) 91 litre
(c) 38 litre
(d) 87 litre
(e) none of these
5. The ratio of expenditure and savings is 3 : 2. If the income increases by 15% and the savings increases
by 6%, then by how much per cent should his expenditure increases?
(a) 25
(b) 21
(c) 12
(d) 24
(e) 54
6. The ratio of petrol and kerosene in the container 3 : 2. when 10 litres of the mixture is taken out and is
replaced by the kerosene, the ratio becomes 2:3. The total quantity of the mixture in the container is :
(a) 25
(b) 30
(c) 45
(d) 96

(e) none of these


7. Two liquids are mixed in the ratio 4 : 3 and the mixture is sold at Rs. 20 with a profit of 33 1/3%. If the
first liquid is costlier than the second by Rs. 7. Find the sum of the costs of both the liquids:
(a) Rs. 11
(b) Rs. 29
(c) Rs. 35
(d) Rs. 70
(e) Rs. 125
8. Alloy A contains 40% gold and 60% silver. Alloy B contains 35% gold and 40% silver and 25% copper.
Alloys A and B are mixed in the ratio of 1 : 4. What is the ratio of gold and silver in the newly formed alloy
is?
(a) 20% and 30%
(b) 36@ and 44%
(c) 25% and 35%
(d) 49% and 36%
(e) 64% and 25%
9. Mitthu Bhai sells rasgulla (a famous sweets) at Rs. 15 per kg. A rasgulla is made up of flour and sugar in
the ratio of 5 : 3. The ratio of price of sugar and flour is 7 : 3 (per kg). Thus he earns
66 2/3% profit. What is t he cost price of sugar?
(a) Rs. 10/kg
(b) Rs. 9/kg
(c) Rs. 18/kg
(d) Rs. 14/kg
(e) Rs. 21/kg
10. In an office there were initially n employees. The HR manager first hired P% employees then after a
month q% employees left the office, then there were finally n employees remained in the office, the value
of p-q is.
(a) pq
(b) pq/100
(c) p/q
(d) p2q/n
(e) none of these
11. The pressure of a definite mass of a gas is directly proportional to the temperature and inversely
proportional to the volume under the given conditions. If temperature is increased by 40% and the volume
is decreased by 20% then the new pressure will:
(a) be increased by 75%
(b) reduce to 25%
(c) be increased by 20%
(d) increase by 28%
(e) None of these
12. A computer typist types a page with 20 lines in 10 minutes but he leaves 8% margin on the left side of
the page. Now he has to type 23 pages with 40 line on each page which he leaves 25% more margin than
before. How much time is now required to type these 23 pages.
(a) 7 hrs
(b) 7 2/3 hrs
(c) 23 hrs
(d) 3.916 hrs

(e) None of these


13. Eklavya can do the 6 times the actual work in 36 days while Faizal can do the one fourth of the original
work in 3 days. In how many days will both working together complete the 3 times of the original work?
(a) 6
(b) 10
(c) 12
(d) 15
(e) none of these
14. A group of workers was put on a job. From the second day onwards, one worker was withdrawn each
day. The job was finished when the last worker was withdrawn. Had no worker been withdrawn at any
stage, the group would have finished the job in 55% of the time. How many workers were there in the
group?
(a) 50
(b) 40
(c) 45
(d) 10
(e) none of these
15. A group of men decided to d a job in 4 days. But since 20 men dropped out every day, the job
completed at the end of the 7th day. How many men were there at the beginning?
(a) 240
(b) 140
(c) 280
(d) 150
(e) none of these

Answers..

bankersadda.com

http://www.bankersadda.com/2015/07/banking-pathway-2015-quant-quiz_24.html

Banking Pathway 2015 : Quant Quiz


1. A motor starts with the speed of 70 kmph with its speed increasing every two hours by 10 kmph. In how
many hours will it cover 345 kms?
a. 2
b. 4
c. 4 hours 5 minutes
d. Cannot be determined
e. None of these

2. A train running at the speed of 20 metres/second crosses a pole in 24 seconds less than the time in
requires to cross a platform thrice its length at the same speed. What is the length of the train?
a. 270 metres
b. 340 metres
c. 180 metres
d. Cannot be determined
e. None of these
3. In a group of 6 boys and 4 girls , 4 children are to be selected. In how many different ways can they be
selected such that at least one boy should be there?
a. 159
b. 205
c. 194
d. 209
e. None of these
4. Out of 11 members of a family 4 are males and rest females. The family hired three taxis on rent to see
a museum. The members have to sit in the cars in such a way that not more than 4 members are in any
one car and each car has at least one male member. In how many different ways the members can
travel?
a. 658
b. 126
c. 140
d. 523
e. None of these
5. If r is the remainder when each of 7654, 8506 and 9997 is divided by the greatest number d (d > 1), then
(d r) is equal to
a. 14
b. 18
c. 24
d. 28
e. None of these
6. For numbers a and b, define as a* b = (a + b) (a-b). then the value of (1 * 2) * 3 is
a. -2/3
b. -1/5
c. 0
d.

e. None of these
7. Four of the following five parts numbers a, b, c, d and e are equal. Which of the following is not equal to
the other four?
a. 242 122 + 112/14 =
b. 17 * 12 + 59 * 4 =
c. 15 * 28 + 20 =
d. 27 * 16 + 56/8 =
e. 185 * 6/2 23 * 5
8. Suppose x and y are inversely proportional and positive. If x increases by 10%, then y decreases by
a. 10%
b. 10/11%
c. 100/11 %
d. 1/11 %
e. None of these
9. In a mathematics examination, the average score of students who passed is x and those who failed is y.
If the average score of all the students who appeared in the examination is z, then the percentage of
students who failed was
a. 100xy/z2
b. 100(y-z)/(x y)
c. 100 (x y)/(x z)
d. 100 (x z)/(x y)
e. None of these
10. The batting average of 40 innings of a cricket player is 50 runs. His highest score exceeds his lowest
score by 172 runs. If these two innings are excluded the average of the remaining 38 innings is 48. His
highest score was :
a. 172
b. 173
c. 174
d. 176
e. None of these
11. A demographic survey of 100 families in which two parents were present revealed that the average age
A, of the oldest child is 20 year less than the sum of the ages of the two parents. If F represents the age
of one parents and M the age of the other parents, then which of the following is equivalent to A?
a. F + M 20/2
b. F + M/2 + 20
c. F + M/2 20
d. F + M 10
e. None of these
12. One of the angles of a triangle is two-third of sum of adjacent angles of parallelogram. Remaining
angles of the triangle are in ratio 5 : 7 respectively. What is the value of the second largest angle of the
triangle?
a. 25
b. 40
c. 35
d. Cannot be determined
e. None of these
13. The largest and the smallest angles of a triangle are in the ratio of 3: 1 respectively. The second

largest angle of the triangle is equal to 44. What is the value of 150 per cent of the largest angle of the
triangle?
a. 149
b. 129
b. 153
d. 173
e. None of these
14. By selling 33 m of cloth, a shopkeeper gains the price of 11 m of cloth. His gain percent is :
a. 7%
b. 50%
c. 20 %
d. 22%
e. None of these
15. On selling a pen at 5% loss and a book at 15% gain. Karim gains Rs. 7. If he sells the pen at 5% gain
and the book at 10% gain, then he gains Rs. 13. The actual price of the book is
a. Rs. 100
b. Rs. 80
c. Rs. 10
d. Rs. 400
e. none of these

bankersadda.com

http://www.bankersadda.com/2015/07/banking-pathway-2015-quant-quiz-di.html

Banking Pathway 2015 : Quant Quiz (DI)


Q.1-5.Study the following graph carefully and answer the questions given below it.
Distribution of total voters in six different cities and detail of female.voters in these cities
Numbers of total voters = 64000
Ratio of male and female voters = 5 : 3

Q.1. What is the difference between the male voters in city B and female voters
in city A ?
(1) 7000
(2) 6880
(3) 6890
(4) 6850
(5) None of these

Q.2.What is the ratio of male voters in city A and that in city D ?


(1) 58 : 59
(2) 57 : 53
(3) 51 : 55
(4) 59 : 57
(5) None of these
Q.3. The number of female voters is what percent of number of male voters in city E ?
(1) 50
(2) 55
(3) 65
(4) 68
(5) None of these
Q.4.The number of male voters in city F is approximately what percent more than that of male voters in
city E ?
(1) 38
(2) 34
(3) 40
(4) 45
(5) 30
Q.5. What is the average number of male voters in city C and D ?
(1) 5200
(2) 5600
(3) 5400
(4) 5800
(5) None of these
Q.6-10.Study the following graph carefully and answer the questions given below it.
Percentage wise distribution of students in six different schools.
Total number of students = 4000

Q.6. Number of boys school (III) is approximately what % of total number of students in school (III)?
(1) 42%
(2) 60%
(3) 45%
(4) 39%
(5) None of these
Q.7. What is the difference between the total number of boys & girls in all the schools?
(1) 450
(2) 600
(3) 400
(4) 490
(5) None of these
Q.8. What is the respective ratio between the number of boys in school II, number of girls in school IV and
total number of students in school V?
(1) 5 : 7 : 11
(2) 4 : 9 : 12
(3) 5 : 4 : 14
(4) 7 : 9 : 12
(5) None of these
Q.9. What is the total number of girls in school III, number of girls in school (IV) and the number of boys in
school V together?
(1) 1120
(2) 1210
(3) 1080
(4) 1320
(5) None of these
Q.10. What is the percentage number of girls in school IV & VI to that of total number of boys?
(1) 33.33 %
(2) 29.33 %
(3) 27.33 %
(4) 37.33 %
(5) None of these
Answers with Explanation !!!!
Solution

1(2) Diffence = 10240 - 3360 = 6880


2(4) Ratio = 9440 : 9120 = 59 : 57
3(5) Percentage = 2400/4000 *100 = 60
4(1) Percentage increase = 1520/4000 *100 = 38
5(3) Average number = 10800/2 = 5400
6(5) Required % = (160/400)*100 = 40 %
7(5)Required difference = 2250 -1750 = 500
8 (3) Required ratio = 300 : 240 :840 = 5 :4 : 14
9(3) Required total students = 240 + 240 + 600 = 1080

10(2) Required % = ( 420 +240)/2250*100 =29.33

To view the above quiz in Hindi : Click Here

bankersadda.com

http://www.bankersadda.com/2015/07/banking-pathway-2015-quant-quiz-di_22.html

Banking Pathway 2015 : Quant Quiz (DI)


Directions (1-5): Study the graphs carefully to answer the questions that follow:
Total number of children in 6 different schools and the percentage of girls in them
1.What is the total percentage of boys in schools R
and U together (rounded off to two digits after
decimal)
(1)78.55
(2)72. 45
(3)76.28
(4)75.83
(5)None of these
2.What is the total number of boys in School T ?
(1)500
(2)600
(3)750
(4)850
(5)None of these
3.The total number of students in school R. is
approximately what per cent of the total number of
students in school S?
(1)83
(2)75
(3)78
(4)82
(5)94
4.What is the average number of boys in schools P and Q together?
(1)1425
(2)1575
(3)1450
(4)1625
(5)None of these
5.What is the respective ratio of the number of girls in school P to the number of girls in school Q?
(1)27 : 20
(2)17 : 21
(3)20 : 27
(4)41 : 21
(5)None of these
Directions (6-10): Study the following pie-charts carefully and answer the questions given follow:
Discipline wise breakup of number of candidates appeared in interview and Discipline wise breakup of
number of candidates appeared in interview by the organization.

6.What was the ratio between the number of candidates appeared in interview from other disciplines and
number of candidates selected from Engineering discipline respectively (rounded off to the nearest
integer)?
(1)3609 : 813
(2)3094 : 813
(3)3094 : 1035
(4)4125 : 1035
(5)3981 : 767
7.The total number of the candidates appeared in interview from management and other disciplines was
what percent of number of candidates appeared form engineering discipline?
(1)50
(2)150
(3)200
(4)Cannot be determined
(5)None of these
8.Approximately what was the difference between the number of candidates selected from Agriculture
discipline and number of candidates selected from Engineering discipline?
(1)517
(2)665
(3)346
(4)813
(5)296
9.For which discipline was the difference in number of candidates selected to number of candidates

appeared in interview the maximum?


(1)Management
(2)Engineering
(3)Science
(4)Agriculture
(5)None of these
10.Approximately what was the total number of candidates selected from Commerce and Agriculture
discipline together?
(1)1700
(2)1800
(3)2217
(4)1996
(5)1550

Answers with Explanation !!!!!!


1.(4) No. of boys in school R & U together
= (2000 * 72.5)/100 + (1000 * 82.5)/100 = 1450 + 825 = 2275
Required % = 2275/3000 * 100 = 75.83
2.(3)
3.(1)
4.(2) Required average = {(2500 * 60)/100 + (3000 * 55)/100} = 1575
5.(3) 40% of 2500 : 45% of 3000
6.(2)
7.(2) (24/16)*100=150%
8.(5) (11-7)% of 7390= 296 (approx )
9.(3)
Science = (25780 * 28)/100 - (7390 * 32)/100 = 4853 appr.
Engineering = (25780 * 16)/100- (7390 * 11)/100 =4124 813 = 3311 appr.
Commerce = (25780 * 18)/100- (7390 * 16)/100 = 4690 1182 = 3458 Approx
10.(1) (16+7)% of 7390 = 1700

bankersadda.com

http://www.bankersadda.com/2015/07/banking-pathway-2015-quant-quiz_21.html

Banking Pathway 2015 : Quant Quiz (Previous Years Question )


1.A rectangular field is to be fenced on three sides leaving a side of 20 feet uncovered. If the area of the
field is 680 sq. feet, how many feet of fencing will be required?
(1)87
(2)88
(3)97
(4)98
(5)110
2.A tank is 25 m long, 12 m wide and 6 m deep. The cost of plastering its walls and bottom at 75 paise per
sq. m, is:
(1)456
(2)885
(3)558
(4)459
(5)656
Directions:(3 - 5) Each of the questions given below consists of a statement and / or a question and two
statements numbered I and II given below it. You have to decide whether the data provided in the
statement(s) is / are sufficient to answer the given question. Read the both statements and
Give answer (1) if the data in Statement I alone are sufficient to answer the question, while the data in
Statement II alone are not sufficient to answer the question.
Give answer (2) if the data in Statement II alone are sufficient to answer the question, while the data in
Statement I alone are not sufficient to answer the question.
Give answer (3) if the data either in Statement I or in Statement II alone are sufficient to answer the
question.
Give answer (4) if the data even in both Statements I and II together are not sufficient to answer the
question.
Give answer(5) if the data in both Statements I and II together are necessary to answer the
3.What will be the cost of painting the inner walls of a room if the rate of painting is Rs. 20 per square
foot?
(1)Circumference of the floor is 44 feet.
(2)The height of the wall of the room is 12 feet.
4.The area of playground is 1600 m2. What is the perimeter?
(1)It is a perfect square playground.
(2)It costs Rs. 3200 to put a fence around the playground at the rate of Rs. 20 per metre.
5.What is the height of the triangle?
(1)The area of the triangle is 20 times its base.
(2)The perimeter of the triangle is equal to the perimeter of a square of side 10 cm.

6.The difference between simple interest and compound Interest on Rs. 1200 for one year at 10% per
annum reckoned half-yearly is:
(1)3
(2)6
(3)2

(4)3.5
(5)1.5
7.The compound interest on a certain sum for 2 years at 10% per annum is Rs. 525. The simple interest
on the same sum for double the time at half the rate percent per annum is:
(1)600
(2)500
(3)400
(4)550
(5)580
8.Find the sum of the series, 51+53+55+.+99
(1)1875
(2)1870
(3)1885
(4)1935
(5) 2175
9.A plane left 30 min later from the schedule time & in order to reach the destination 1500 km away on
time ,it had increase the speed by 250 kmph from the usual speed,its usual speed is
(1)700kmph
(2)750kmph
(3)250kmph
(4)350kmph
(5)500kmph
10.A certain distance is covered at a certain speed.if half of this distance is coveredin double the time
,the ratio of the two speed is.
(1)4:1
(2)1:4
(3)1:8
(4)1:6
(5)8:1
11.One card is drawn at random from a pack of 52 cards. What is the probability that the card drawn is a
face card (Jack, Queen and King only)?
(1)1/4
(2)6/7
(3)3/13
(4)3/5
(5)1/13
12.Two cards are drawn together from a pack of 52 cards. The probability that one is a spade and one is a
heart, is:
(1)13/102
(2)5/102
(3)3/13
(4)3/5
(5)1/13
13.A card is drawn from a pack of 52 cards. The probability of getting a queen of club or a king of heart
is:
(1)13/102

(2)1/26
(3)3/13
(4)3/5
(5) 1/13
14.A man rows to a place 48 km distant and come back in 14 hours. He finds that he can row 4 km with
the stream in the same time as 3 km against the stream. The rate of the stream is:
(1)1kmph
(2)1.5kmph
(3)2kmph
(4)2.5kmph
(5)2.6kmph
15.A man takes twice as long to row a distance against the stream as to row the same distance in favour
of the stream. The ratio of the speed of the boat (in still water) and the stream is:
(1)3:1
(2)2:1
(3)1:2
(4)3:2
(5)none of these

Answers with Explanation !!!!!!!!

To View the above quiz in Hindi : Click here

bankersadda.com

http://www.bankersadda.com/2015/08/banking-pathway-2015-quant-quiz_15.html

Banking pathway 2015 : Quant Quiz


1.The age of a man is three times the sum of the ages of his two sons. Five years hence, his age will be
double of the sum of the ages of his sons. The fathers present age is
1)40 years
2)45 years
3)50 years
4)55 years
5)None of these
2.In a company, 60% of the employees are men. Of these 40% are drawing more than Rs. 50,000 per year
if 36% of the total employees of the company draw more than Rs. 50,000 per year, what is the percentage
of women who are drawing less than Rs. 50,000 per year?
1)70
2)60
3)40
4)30
5)None of these
3.A trader fixed the price of an article in such a way that by giving a rebate of 10% on the price fixed, he
made a profit of 15%. If the cost of the article is Rs. 72, the price fixed on it, is
1)Rs. 82.80
2)Rs. 90.00
3)Rs. 92.00
4)Rs. 97.80
5)None of these
4.A man fills a basket with eggs in such a way that the number of eggs added on each successive day is
the same as the number already present in the basket. This way the basket gets completely filled in 24
day. After how many days the basket was th full?
1)6
2)12
3)17
4)22
5)None of these
5.The difference between the simple interest received from two banks on Rs. 500 for two years is Rs.2.50.
what is the difference between their rates?
1)0.25%
2)0.5%
3)1%
4)2.5%
5)None of these
6.In a garrison, there was food for 1000 soldiers for one month. After 10 days, 1000 more soldiers joined
the garrison. How long would the soldiers be able to carry on with the remaining food?
1)25 days
2)20 days
3)15 days
4)10 days

5)None of these
7.The tank-full petrol in Aruns motor cycle lasts for 10 days. If he starts using 25% more everyday, how
many days will the tank-full petrol last?
1)5
2)6
3)7
4)8
5)None of these
8.There are some balls of red, green and yellow colour lying on a table. There are as many red balls as
there are yellow balls. There are twice as many yellow balls as there are yellow balls as there are green
ones. The number of red balls.
1)Is equal to the sum of yellow and green balls
2)Is double the number of green balls.
3)Is equal to yellow balls minus green balls
4)Cannot be determined
5)None of these
9.A and B can complete work together in 5 days. If A works at twice his speed and B at half of his speed,
this work can be finished in 4 days. How many days would it take for A alone to complete the job?
1)10
2)12
3)15
4)18
5)None of these
10.Three men start together to travel the same way around a circular track of 11 km. their speeds are 4,
5.5 and 8 kmph respectively. When will they meet at the starting point for the first time?
1)After 11 hours
2)After 21 hours
3)After 22 hours
4)After 33 hours
5)None of these

Answers
Displaying 1.JPG
Displaying 2.JPG

bankersadda.com

http://www.bankersadda.com/2015/07/banking-pathway-2015-quant-percentage.html

Banking Pathway 2015 : Quant- Percentage (Study notes &


Quiz)
Dear Reader ,
In continuance to provide study notes(Short Tricks ) today we are Providing Notes on Percentage. Which is very
crucial topic for SBI MainS(in DI), IBPS PO, IBPS Clerk, RBI assistant, IBPS RRB and many more competitive
examinations.

Here we also Providing some concept clearing quiz for analysing yourself. Time management also very crucial
part for gain good marks in exam. so use wisely your time. Do not waste time on lengthy questions.
These Study notes are provided by one of our ardent BA reader Insomniac. We wish you good luck for future.
Percentage:
The word defines itself Per means 1 upon something and Cent Is like Paise. In India we have 1 rupee = 100
paise
So per cent = 1/100 Part of something or %[ This sign even means 1/100] So if i say 20% of something Just
multiply that something by 20/100 or 0.2
Like wise if is say 30% then it simply means i want to know 30/100th or 3/10th part value of something.
Why Do we Use percentage ?
Basically It is used for comparison.
Like If i say i got 400 marks in 10th and the other guy says that he got 600 marks in 10th. So Numerically He has
got more marks than me But does his score is relatively better than me ?
For that purpose we must know that He got 600 marks out of how many marks. Let's say he got 600 out of 1000.
So his percentage marks will be 60%
And I got 400 out of 500. So my % marks will be 80%.
Now you can easily say that My marks are better because i am getting 80% and he is getting 60%.
How to calculate Percentage [ You all know it but just say I am telling it to myself :P ]
Well the simple formula is [Value/total value] * 100
For example A Ring Contains 63 gm of Gold and total weight of ring is 70 gm. Find the percentage of Gold
in the ring ?
So By the Formula [Value/Total Value]*100
[63/70]*100
[9/10]*100
90%.
% to Decimal Conversion or Fraction Conversion.
Just remember in Fraction Conversion we leave the fraction as it is without Converting Into Decimal. While in
Decimal Conversion we first convert into fraction then Write the Decimal Value of that fraction.
Very easy You just have to dive by 100 nothing else.
For example 30% = 30/100 = 0.3
21% = 21/100 = 0.21
99% = 99/100 = 0.99
60% = 60/100 or 3/5 = 0.6

Case 1 [ Percentage of Quantity ]


Find the no. of male Students i.e boys, If there are 47% male students in the school and Total no. of
students in the school is 1000.
As i said If you See anywhere % of something. Just convert the no. into it decimal value and multiply by that
Something.
So in the above Question Boys are 47%[ Convert this into Decimal and you will get 0.47] of 1000 [ Something]
So what we gonna do friends we will just multiply it by 0.47
So the no. of Boys will be 0.47*1000 = 470.
Lets See another Example.
A student scored 85% marks. Total marks are 400. How much marks did he score.
So a student got 85% marks out of 400
So again 85%[ Convert 85% in decimal i.e 0.85] of 400 [something]
So the answer will be 0.85*400 = 340
so 340 is our answer.
Well they can also Change the Final Question.
Like in First Example they asked Find the no. of male students. They could have asked the no. of students that
are not male.
So what we should do in that case. Nothing to worry my friend just do the usual job 47% are male that means that
the rest 53% are not male now calculate 535 of 1000 that will be your answer i.e 530
Case 2 [ Inverse Case]
Now in the case 1 we were just asked to Find the % value something. But What if % value of something is given
and we have to find the Total Value. ?
Now to worry below example will make it clear.
30% of a Number is 150. What is the number.
So after examining the question we can say that 30% of Some number is 150[ But we don't know yet what is the
original number]
When we don't know about something Just Assign a variable to that value.
So we say that the Original Number is x
So as mentioned in the question 30% of x = 150
[ Convert 30% into decimal] 0.3*x= 150
0.3x = 150
x = 150/0.3
x = 500
So you see it's Quite easy.
Likewise Many Different Question can be formed on the same logic. Lets discuss 1
There are 200 girls in the class and girls and girls make up 25% of the class. Find the total No. of students in the
class.
We don't know the No. of students so assume that no. of students is x
So what is given in the Question.
25% of Total Students in the class are Girls and Total Girls in the class is 200
lets just try convert above [English] Statement into mathematical Form
25% of x = 200
0.25*x = 200x = 200/0.25
x = 800
So total no. of students in the class = 800.
Case 3 - Percentage Change.[ Very Important For DI]
The simple Way to put that is [( Change in Quantity/ Original Quantity) *100 ] Also [change in quantity = Final
Quantity - Initial Quantity]
Note- The quantity in whose respect % change is asked is considered as the base By base i mean the original
value in the above formula.

Let me make it clear to you with the help of Some Example.


The height of Nikhil some times ago was 160cm. Now his height is 200cm. Find the % change in his
height?
So if we analyse the above question We can say that all we have to calculate is the % change in the height of
Nikhil with respect to his Earlier Height.
So now lets apply the formula here [( Change in quantity/ Original Quantity) * 100]
Which will be [{( 200-160)/160} * 100 ]
[{40/160}*100]
[(1/4)*100]
25%
So the % change in height = 25%.
Case 4- Use of Base Value and With Respect To Cases[ Very very Important For DI]
Suppose Salary of Raman is 80,000 and Salary of Ved is 1,00,000. The questions are
What per cent is the salary of Ved to that of Raman?
It's a very simple Question If you just know in whose respect you have to find the %.
Now in the above Question we have to find the % of ved salary with respect to ELF's Salary [ remember jiske
respect me % nikalna hota hai Wohi base hota hai ]
So here we have to find Ved's Salary with respect to Raman So we use the formula [ (Value/ In whose respect it is
asked)*100]
So ved's salary in respect to Raman's salary will be [ (1,00,00/80,000)*100 ] = 125%
So VED's Salary is 125% of Raman's Salary.
If the question was just opposite.
Like What percent is the salary of Raman to that of Ved. (In this Question the Base will be Ved's Salary)
So lets just apply the formula [ Value/ In whose respect it is asked) * 100]
(80,000/1,00,000)*100 = 80%
So elf's salary is 80% of VED's Salary.
Case 5 - Product Constancy [ Most Important Because With it's Application You can also solve Questions
related to Time and Work, Speed Time Distance, Average etc. This Concept has a very huge application]
i - Speed*Time = Distance
ii- efficiency*time = work
ii- Length*breadth= area
iv- Average*No. of elements = Total value
v - rate*quantity = Expenditure
let me make you clear with an example.
The price of sugar is increased by 25% then by how much per cent should a customer reduce the consumption (
i.e quantity used) Of sugar so that he has not increase his expense on Sugar.
Just remember If one factor of product constancy is increased by P% then the other factor will be decreased by
[(p)/(100+p) * 100] To maintain the Product Constancy.
Now in the above Question The rate of sugar is increased by 25% So by how much % we should reduce the
quantity to maintain the same expenditure
Just apply the above formula [(p)/(100+p)*100] = (25/125)*100 = 20%
Now It sound Simple but It is difficult to remind these formulas at the time of Solving Question So let me Give you
simple method of learning this Formula.
Just Imagine In Your mind that the Quantity is 100. Ok
if the value is inreased by 25 % how much should the consumption be reduced.
Now all you have to remember is [(How much % value is Increased/ What it becomes after increase) * 100 ]
No as i said In your mind the Quantity is 100. How much the value is increased in the above Question yeah 25%
And how much it will become after 25% increase if the Quantity was 100 yeah That will be 125. So the answer will

be
(25/125)*100 = 20%
lets try again If the price of petrol is increased by 50%. By how much % the consumption be reduced so the
expenditure remains same.
Just apply the formula How much increased = 50
What it will become after 50% increase = 150
% redcution required = (50/150)* 100 = 33.33%
In the same way you can also use the same formula for calculating just the opposite.
For Example If the price of Sugar is reduced by 20% by how much should the family increase it's
consumption So the expenditure remains same?
How much % decrease ? = yeah it is 20%
What it will become after 20% decrease = Yeah 80
So Increase required = (20/80)*100 = 25%
lets do one more question.
The price of petrol is reduced by 33.33% but how much % should a person increase his consumption so that His
expenditure remains constant.
How much decrease = 33.33
What it will become after 33.33% decrease = 66.66
So % increase required = (33.33/66.66)*100 = 50%
The Length of Rectangle is Increased by 25% By what % the breadth be reduced so that are remains
Constant?
Try Again How much Increase 25
What it becomes 125
% to be reduced = (25/125)*100 = 20%
Same way the Question of Time Speed Distance can also be solved But i will teach that when i will Explain Time
Speed And Distance.

Case 6 - Increase or decrease In value to Get Back the Original Value.


Remember if a value P is increase by x % then we have to decrease the resultant value by
[{ x/(x+100)} *100]% to get back the original value.
For Example Rocky's Salary is 1000rs and it Increased by 10%. How much % His salary must be Decreased So
that he Gets His original Salary.
Apply the above Formula [{x/(x+100)}*100 = [ {10/(100+10)}*100] = 100/11 or 9.09090%
In case of Decrease.
The formula will be [ {x/(100-x)}*100]%
Rocky Salary is 1000 and it is decreased by 10%. By how much % his salary must be increased so that he gets
His Original Salary.
Apply the formula here [{10/(100-10)}*100]% = 100/9 % or 11.11%
But Instead of Doing All this BS you can Also Apply My previous formula here.
Like Salary Increased 10. what wit will become 110.
How much it should be reduced = (10/110)*100 = 100/11 = 9.09%
Again Salary decreased = 10, What it will become 90.
So how much it should be increased to get the original salary = (10/90)*100 = 100/9 = 11.11%
CASE - 7 Concept of "by" and 'to"
Please note that there is very Big Difference between by and to.
Eg . The income is reduced BY 40% it means the New Income 60% of the original value.

And If income is to 40% it means The new Income is 40% of the Original Value.
Case 8 - Consecutive Increase in Percentage.
Suppose the Salary of Sumit is first increase by 20% and Then again it's Increased by 20%.
What is the Total Percentage Increase in His Salary.
Now don't try to be smart here and just add 20% and 30%And say That it's 50% Increase-- THAT WILL BE
TOTALLY WRONG.
Actuall let me make the Picture a Little bit Clear. What actually Happens in the case of Consecutive Increase and
Decrease.
Now Just Suppose that The Salary of Sumit was 1000 Rs. it gets Increased by 20% so What it will become ?
Yeah you are right 1200rs.
Now When it is Again Increased by 30% Then we are Calculating that 30% increase on 1200RS ans not on 1000
So the Inrease will be 30% of 1200 which will be 360. So increased salary will be 1200 + 360 = 1560.
Now are two Shortcut Methods Here.
1st Simple Multiplication.
When I say Something is increased by 20% It means It's Value Is increased by 20 % or It's Total value is 120% of
the original Value Ok ?
Like 100 is incresed by 20% That means it's final value will be 100 + 20% of 100 = 100 + 20 = 120.
So if Sumit Salary is Increased By 20% that the Value will be 100*1.2.
And If it's Again increased by 30% then the value will be 100*1.2*1.3 = 100*1.56 = 156.
Total increase = 56.
Percentage Increase = 56%
2nd Method Formula Approach.
Well the formula is [x + y + (xy)/100]% [Note this formula works only when there is 2 increases]
Now apply the formula in above Question you will get 20 + 30 + (20*30)/100 = 50 + 600/100 = 50 + 6 = 56%
Same Sumit's Salary is 1000rs and if it's asked the Salary of Sumit is Increased First by 20% then 30% and then
again by 40% then what will be Total Increase and Final Salary.
It's Pretty simple now 1000*1.2*1.3*14 = 2184
That's the final Salary and % inrease = [(2184-1000)/1000]*100 = (1184/1000)*100 = 118.4%
Some Similar Questions Are like.
The Side of Square is Increased by 10% what will be the increase in Area.
So Just Let The Value of Eaxh side is x, If it's Increased by 10% Then it will become 1.1x
As You know are = Side*side
So Initially The Area without Increase Would Have been x*x = x^2
After Increase the Area will be 1.1x*1.1x = 1.21x^2
So total % increase in area will be 21%.
Case -9 Consecutive Increase and Decrease Simultaneously
In the last case we saw the case of % increase but now we will learn how to solve when there is a consecutive
Increase and as well As decrease.
It's same as the last example.
Sumit's Salary is 1000rs Suppose the salary of Megamind was first Increased by 30% and Then decreased by
20%. What will be final Increase or Decrease in His Salary.
Just do The same Thing 30% increase means 1000*1.3
And then 20% Decrease mean 0.8times [ remember we have to decrease here and 20% decrease means 0.2
Point decrease]
So total decease = 1000*1.3*0.8 = 1040.

Final Increase = 40RS


% Increase will be 4%.
With Formula.
Remember the formula x + y + (xy)/1000
Same formula can be used here But remember Increase means +ve Sign and decrease means -ve sign,
So apply here now 30 - 20 + (30)(-20)/100 = 10 - 600/100 = 10 - 6 = 4%.
Thank you Insomniac..
Quiz :
Time : 5-6 minutes.
1. Two students appeared at an examination. One of them secured 9 marks more than the other and his
marks was 56% of the sum of their marks. What are the marks obtained by them?
A) 42, 36
B) 44, 39
C) 42, 33
D) 44, 37
E) None of these.
2. A fruit seller had some oranges. He sells 40% oranges and still has 420 oranges. How many oranges he
had originally?
A) 690
B) 700
C) 720
D) 745
E) None of these.
3. A batsman scored 110 runs which included 3 boundaries and 8 sixes. What percent of his total score
did he make by running between the wickets?
A) 45(6/11) %
B) 45(7/11) %
C) 45(5/11) %
D) 46(5/11) %
E) None of these.
4. In an election between two candidates, one got 55% of the total valid votes, 20% of the votes were
invalid. If the total number of votes was 7500, what was the number of valid votes that the other candidate
got?
A) 3690
B) 2700
C) 5720
D) 4745
E) None of these
5. A student has to obtain 33% of the total marks to pass. He got 125 marks and failed by 40 marks. The
maximum marks are
A) 600
B) 700
C) 520
D) 500
E) None of these.

6. A housewife saved Rs. 2.50 in buying an item on sale. If she spent Rs. 25 for the item, approximately
how much percent she saved in the transaction ?
A) 6 %
B) 7 %
C) 10 %
D) 9(1/11) %
E) None of these.
7. A pipe X is 30 meters and 45% longer than another pipe Y. find the length of the pipe Y?
A) 600/29 meter
B) 37.25 meter
C) 20 meter
D) 50 meter
E) None of these.
8. On my sisters 15th birthday, she was 159 cm in height, having grown 6% since the year before. How
tall was she the previous year ?
A)156 cm
B) 150 cm
C) 155 cm
D) 172 cm
E) None of these
9. Sumit got 30% of the maximum marks in an examination and failed by 10 marks. However, Sujith who
took the same examination got 40% of the total marks and got 15 marks more than the passing marks.
What were the passing marks in the examination?
A) 96
B) 150
C) 75
D) 85
E) None of these
10. 30% of the men are more than 25 years old and 80% of the men are less than or equal to 50 years old.
20% of all men play football. If 20% of the men above the age of 50 play football, what percentage of the
football players are less than or equal to 50 years?
A) 60 %
B) 50%
C) 80 %
D) 85 %
E) None of these

Answers:
1. C
2. B
3. C
4. B
5. D
6. D
7. A
8. B
9. D
10.C

bankersadda.com

http://www.bankersadda.com/2015/07/banking-pathway-2015-quantitative.html

Banking Pathway 2015 : Quantitative Aptitude Mix Quiz


1. By selling 12 marbles for a rupee, a shopkeeper loses 20%. In order to gain 20% in the transaction, he
should sell the marbles at the rate of how many marbles for a rupee?
(1) 8
(2) 6
(3) 4
(4) 3
(5) None of these

2. In a certain store, the prot is 320% of the cost. If the cost increase by 25% but the selling price
remains constant, approximately what percentage of the selling price is the profit?
(1) 30%
(2) 70%
(3) 100%
(4) 250%
(5) None of these
3. A horse and a carriage together cost Rs.8000. If by selling the horse at a prot of 10% and the carriage
at a loss of 10% a total prot of 2.5% is made, then what is the cost price of the horse ?
(1) Rs. 3000
(2) Rs. 3500
(3) Rs. 4000
(4) Rs. 5000
(5) None of these
4. A piece of cloth costs Rs. 35. If thelength of the piece would have been 4 m longer and each metre
costs Rs. 1 less, the cost would have remained unchanged. How long is the piece?
(1) 14 m
(2) 10 m
(3) 12 m
(4) 9 m
(5) None of these
5. If the total emoluments of A and B are equal. However, A gets 65% of his basic salary as allowances
and B gets 80% of his basic salary as allowances. What is the ratio of the basic salaries of A and B?
(1) 16:13
(2) 5 : 7
(3) 12:11
(4) 7:9
(5) None of these
6. A contractor employed 25 labourers on a job. He paid 275 for the work. After retaining 20% of this sum,
he distributed the remaining amount amongst the labourers. If the number of men to women labourers
was in the ratio 2 : 3 and their wages in the ratio 5 : 4, what wages did a woman labourer get?
(1) 10
(2) 8
(3) 12
(4) 15

(5) None of these


7. The dimensions of a rectangular room when increased by 4-m are in the ratio of 4 : 3 and when
decreased by 4 m are in the ratio of 2 : 1. The dimensions of the room are
(1) 6 m and 4 m
(2) 12 m and 8 m
(3) 16 m and 12 m
(4) 24 m and 16 m
(5) None of these
8. If 1066 are divided among A, B, C and D such that A : B = 3 : 4 B:C=5:6andC:D= 7 : 5, who will get the
maximum?
(1) B
(2) A
(3) C
(4) D
(5) None of these
9. If the numerator of a fraction is increased by 350% and the denominatior of the fraction is increased by
300%. the resultant fraction is 9/22. What is the original fraction?
(1)
(2) 5/12
(3) 7/9
(4) 4/11
(5) None of these
10. Ms. Sujata invests 7% i.e. Rs. 2170 of her monthly salary in mutual funds. Later she invests 18% of
her monthly salary in recurring deposits also. She invests 6% of her salary on NSCs. What is the total
annual amount invested by Ms. Sujata?
(1) Rs. 1,25,320
(2) Rs. 1,13,520
(3) Rs. 1,35,520
(4) Rs. 1,15,320
(5) None of these
11. Fresh grapes contain 80% waterby weight, whereas dried grapescontain 15% water by weight. How
many kg of dried grapes canbe obtained from 3.4 kg of freshgrapes?
(1) 0.51
(2) 0.6
(3) 0.68
(4) 0.8
12. An interview panel found that a candidate has given a wrong date about his height. while lling up his
form he lled up 20% more than his actual height. His actual height is 5 feet 2 inches. By what
approximate percent shouldhe reduce his height to get actual height?
(1) 15
(2) 14
(3) 18
(4) 17
(5) None of these
Direction (1-3): Study the following information carefully and answer the questions that follow:

A committee of 12 persons is to be formed from 9 men and 8 women


13. In how many ways this can be done if at least 5 men to have to be included in a committee
(1) 6062
(2) 6000
(3) 6010
(4) 6005
(5) None of these
14. In how many of these committee the women are in majority
(1) 1008
(2) 1100
(3) 120
(4) 1225
(5) None of these
15. In how many of these committee the men are in majority
(1) 2000
(2) 2702
(3) 2705
(4) 2700
(5) None of these
Answers

bankersadda.com

http://www.bankersadda.com/2015/08/banking-pathway-2015-quant-di-quiz.html

Banking Pathway 2015: Quant (DI Quiz)


Directions (1-5): Study the radar graph carefully and answer the questions that follow: Number of
students (in thousands) in two different universities in six different years

1.What was the difference between the number of students in university-1 in the year 2010 and the
number of students in university-2 in the year 2012?
(1) Zero
(2) 5,000
(3) 15,000
(4) 10,000
(5) 1,000
2.What is the sum of the number of students in university-1 in the year 2007 and the number of students
in university-2 in the year 2011 together?
(1) 48000
(2) 55000
(3) 45000
(4) 57000
(5) 40000
3.If 25% of the students in university-2 in the year 2010 were females, what was the number of male
students in the university-2 in the same year?
(1) 11250
(2) 12350
(3) 12500
(4) 11500
(5) 11750
4.What was the percent increase in the number of students in university-1 in the year 2011 as compared
to the previous year?
(1) 135

(2) 15
(3) 115
(4) 25
(5) 35
5.In which year was the difference between the number of students in university-1 and the number of
students in university-2 highest?
(1) 2008
(2) 2009
(3) 2010
(4) 2011
(5) 2012
Directions (6-10):In the following multiple graphics, the number of selected candidates for 6 different
posts by three different companies A, B and C has been shown. Read the graph carefully and answer the
questions.

POST 1: HR Officer
POST 2: IT Officer
POST 3: Assistant Manager
POST 4: Advertisement Office Assistant
POST 5: Office Assistant Operations
POST 6: Junior Office Administrator
6.What is the ratio between the number of all candidates selected for company A and that selected for the
posts of assistant managers and junior office administrators in all three companies?
(1) 103. : 107
(2) 102 : 107
(3) 103 : 106
(4) 113 : 117
(5)None of these
7.The number of candidates recruited for the post of office assistant operations in company B is
approximately what percent of total candidates recruited in that company?
(1) 28%
(2) 24%
(3) 30%
(4) 31%
(5) None of these

8.The number of candidates recruited for the posts of assistant manager and advertisement office assistant is
what per cent of the candidates recruited for the post of junior office administrator and HR officer by the company
C?
(1) 115%
(2) 120%
(3) 135%
(4) 141%
(5) None of these
9.The total number of candidates recruited for the post of HR officers in all the companies is
approximately what per cent of the total candidates recruited by the company A for all posts?
(1) 16%
(2) 11%
(3) 12%
(4) 14%
(5) None of these
10.What is the respective ratio between the average number of candidates selected for all the posts by
company A and company C?
(1) 113 : 115
(2) 115 : 113
(3) 113 : 117
(4) 117 : 113
(5) None of these
Answers

To solve above quiz in Hindi... Click Here

bankersadda.com

http://www.bankersadda.com/2015/07/banking-pathway-2015-quant-mix-quiz.html

Banking Pathway 2015: Quant Mix Quiz


1. The ratio of the no. of boys to the no. of girls in a school of 720 students is 3: 5. If 18 new girls are
admitted in the school, find how many new boys may be admitted so that the ratio of the no. of boys to
the no. of girls may change to 2 : 3.
A) 80
B) 50
C) 42
D) 35
E) None of these
2. The values of x in the equation 7(x+2p)^2 + 5p^2= 35xp + 117p^2 are
A) (4p, 3p)
B) (4p, 3p)
C) (4p, 3p)
D) (4p, 3p)
E) None of these.
3. What is the amount of compound interest and effective rate of interest if an amount of Rs.20000 is
deposited in a bank for one year at the rate of 8% per annum compounded semi annually.
A) 1632, 8.16%
B) 1600, 9.36%
C) 1936, 10.36%
D) 1756, 9.56%
E) None of these.
4. 3 men can complete a piece of work in 6 days. Two days after they started the work, 3 more men joined
them. How many days will they take to complete the remaining work?
A) 5 days
B) 2 days
C) 8 days
D) 12 days
E) None of these.
5. Two pipes A and B can fill a tank in 36 min. and 45 min. respectively. A water pipe C can empty the tank
in 30 min. First A and B are opened. After 7 minutes, C is opened. In how much time, the tank is full?
A) 45 minutes
B) 46 minutes
C) 52 minutes
D) 56 minutes
E) None of these.
6. Abhishek purchased 140 shirts and 250 trousers at the rate of Rs. 450 and Rs. 550 respectively. What
should be the overall average selling price of shirts and trousers so that 40% profit is earned? (Rounded
off to next integer).
A) Rs.725
B) Rs.710
C) Rs. 720
D) Rs. 700
E) None of these

7. In a college the students in Arts and Commerce faculties were in the ratio of 4:5 respectively. When 65
more students joined Commerce faculty the ratio became 8:11 respectively. How many students are there
in Arts faculty?
A) 520
B) 650
C) 715
D) Cannot be determined
E) None of these
8. An amount of Rs. 10,000 becomes Rs. 14,641 in two years if the interest is compounded half yearly.
What is the rate of compound interest % per annum?
A) 10
B) 20
C) 16
D) 12
E) None of these.
9. Area of a rectangle is equal to the area of the circle whose radius is 21 cm. If the length and the breadth
of the rectangle are in the ratio of 14:11 respectively, what is its perimeter?
A) 142cm
B) 140cm
C) 132cm
D) 150 cm
E) None of these
10. Prashant purchased a TV set for Rs. 12,500 and spent Rs. 300 on transportation and Rs. 800 on
installation. At what price should he sell it so as to earn an overall prot of 15%?
A) Rs. 14,560
B) Rs. 14,375
C) Rs. 15,640
D) Rs. 15,375
E) None of these.
11. Three girls start jogging from the same point around a circular track and each one completes one
round in 24 seconds, 36 seconds and 48 seconds respectively. After how much time will they meet at one
point?
A) 2 minutes, 20 seconds
B) 2 minutes, 24 seconds
C) 4 minutes, 12 seconds
D) 3 minutes, 36 seconds
E) None of these
12. A 240-metre long train running at the speed of 60 km/hour will take how much time to cross another
270 metre long train running in opposite directions at the speed of 48 km/hour?
A) 17 seconds
B) 3 seconds
C) 12 seconds
D) 8 seconds
E) None of these
13. Sarita started a boutique investing an amount of Rs. 50,000. Six months later Meeta joined her with an
amount of Rs. 80,000. At the end of one year they earned a profit of Rs. 18,000. What is Sarita's share in

the profit?
A) Rs. 9,000
B) Rs. 8,000
C) Rs. 12,000
D) Rs 10,000
E) None of these
14. A alone can make 100 baskets in 6 days and B alone can make 100 baskets in 12 days. In how many
days can A and B together make 100 baskets?
A) 3 days
B) 5 days
C) 2 (1/2 ) days
D) 3(1/2) days
E) None of these
15. Samirs age is one-fourth of his fathers age and two third of his sister Reemas age. What is the ratio
of the ages of Samir, Reema and their father respectively?
A) 3:2:8
B) 3:4:8
C) 2:3:8
D) 4:3:8
E) None of these
Answers & Explanation:

1.(C) The ratio of the no. of boys to the no. of girls = 3 : 5


Sum of the ratios = 3 + 5 = 8
So, the no. of boys in the school = (3 720)/8 = 270
And the no. of girls in the school = (5 720)/8 = 450
Let the no. of new boys admitted be x, then the no. of boys become (270 + x).
After admitting 18 new girls, the no. of girls become 450 + 18 = 468
According to given description of the problem, (270 + x)/468 = 2/3
or, 3 (270 + x) = 2 x 468
or, 810 + 3x = 936 or, 3x = 126 or, x = 42.
Hence the no. of new boys admitted = 42.
2.(A) 7(x+2p)^2+5p^2= 35px+117 p^2
7x^2+33p^2+28px = 35px+ 117 p^2
x^2-px+12p^2 = 0
on solving x= 4p,-3p
3.(A) Interest = 20000[(1+4/100)^2-1]= 1632
Efeective rate of interst = 1632/20000*100= 8.16%

4.(B) Work done by 3 men in 2 days = (1/6x 2 )=1/3.


Remaining work = (1 1/3)= 2/3
Now 3 men's 1 day's work =1/6
1 man's 1 day's work = 1/18
6 men's 1 day's work = (6 x 1/18=1/3
Now, 1/3 work is done by them in 1 day.
2/3 work is done by them in (3 x2/3)= 2 days.

5.(B) Part filled in 7 min = 7[ 1/36 + 1/45] = 7/20


Remaining part = [ 1- 7/20] = 13/20
Net part filled in 1 min. When A,B and C are opened
= [ 1/36 + 1/45 + 1/30 ] = 1/60
Now, 1/60 part is filled in 1 min.
13/20 part is filled in [60x13/20] = 39 min
Total time taken to fill the tank = (39 + 7) min. = 46 min.
6. (C); CP of 140 shirts and 250 trousers
= Rs (140 * 450 + 250 * 550)
= Rs. (63000 + 137500) = Rs 200500
SP for 40% profit = Rs (140/100 200500)=280700
Average selling price = Rs (280700/(140+250) )=Rs.720
7. (A): Let the initial number of students in Arts and Commerce faculties be 4x and 5x respectively.
According to the question,
4x/(5x+65)=8/11
44x = 40x + 520
= x = 520/4 = 130
Number of students in Arts faculty = 4 * 130 = 520
8. (B) Time = 4 half years
Rate = R% per half year
A = P (1+R/100)^T
14641 = 10000 (1+R/100)^4
(1+R/100)^4 = 14641/10000= (11/10)^4
1+R/100=11/10 ? R/100=1/10
R =10%
Rate % per annum = 20%
9. (D); Area of rectangle = Area of circle
= 22/7 * 21 * 21 = 1386 sq.cm
Let the length and breadth of rectangle be 14x and 11x cm respectively. Then,
14x * 11x = 1386
x^2=1386/(14 11)=9
x = v9=3
Perimeter of rectangle = 2(14x + 11x)
= 50x = 50 * 3 = 150cm.
10.(C); Total cost of TV = Rs (12500 = 300 + 800) = Rs 13600
For a profit of 15%
Selling price = Rs. (115/100 13600)=Rs.15640
11. (B); required time = LCM of 24, 36 and 48 second = 144 sec. = 2 min 24 sec.
12. (A); Relative speed = (60 + 48) kmph = 108 kmph
= (108 5/18) m/sec = 30m/sec
Total distance = (240 + 270) metre = 510 metre
Time taken in crossing = (510/30) = 17 seconds
13. (D); Ratio of equivalent capitals of Sarita and Neeta for 1 month = 50000 * 12 : 80000 * 6
Sarita's share = Rs (5/9 18000) = Rs. 10000
14. (E) A's 1 day's work = 1/6

B's 1 day's work = 1/12


(A +B)'s 1 day's work = 1/6 + 1/12 = 2 + 1/12 = 1/4
A and B together will make 100 baskets in 4 days.
15. (C); Samir's age = x years
His father's age = 4x years
Reema's age = 3/2x years
Required ratio = x : 3x/2 : 4x = 2: 3 : 8

To view above question in hindi click here

bankersadda.com

http://www.bankersadda.com/2015/08/banking-pathway-2015-quant-quiz_18.html

Banking Pathway 2015: Quant Quiz


1.A shopkeeper mixes two types of wheat, each costing Rs. 45/kg and Rs. 58/kg, so that by selling the
resultant mixture at Rs. 55/kg, he makes a profit of 10%. In which ratio did he mix them?
(1) 3:10
(2) 1:1
(3) 8:5
(4) 2:7
(5) None of these
2.Price of an article increases by 25%. By what percent should its consumption be reduced so that there
is no change in expenditure?
(1) 20%
(2) 25%
(3) 33.335
(4) 10%
(5) None of these
3.A and B can complete a work in 20 days and 60 days working alone. In how many days can they
complete the work, working together?
(1) 12 days
(2) 30 days
(3) 15 days
(4) 80 days
(5) 40 days
4.Two numbers are in ratio of 3:5. If the first number is doubled while second number is increased by 20,
the new ratio is 2:3. Find the sum of the initial numbers?
(1) 40
(2) 50
(3) 64
(4) 7
(5) None of these
5.In how many ways a team of 5 members, comprising of 3 boys, be selected from a group of 7 boys and
3 girls?
(1) 70
(2) 105
(3) 540
(4) 14
(5) None of these
6.In how many ways can the letters of the word PAPAYA be arranged?
(1) 60
(2) 120
(3) 360
(4) 720
(5) None of these
7.Find the difference between compound interest and simple interest on Rs. 15000 at 20% per annum
compounded yearly for 2 years?

(1) Rs. 400


(2) Rs. 500
(3) Rs. 600
(4) Rs. 750
(5) Rs. 1000
8.If selling price of 40 articles is same as cost price of 50 articles, then find the profit/loss %?
(1) 20% loss
(2) 20% profit
(3) 25% loss
(4) 25% profit
(5) None of these
Directions (9-10): What should come in place of the question mark (?) in the following number series?
9.5, 7, 11, 13, 17, 19, ?
(1) 23
(2) 21
(3) 25
(4) 29
(5) None of these
10.2, 6, 12, 20,
(1) 50
(2) 56
(3) 63
(4) 75
(5) None of these

30,

42, ?

Answers with explanation !!!!

bankersadda.com

http://www.bankersadda.com/2015/08/banking-pathway-2015-quant-quiz_13.html

Banking Pathway 2015: Quant Quiz


Directions (1-5):Each Question below is followed by two statements I and II. You have to determine
whether the data given in the statement is sufficient for answering the question or not. You should use
the data and your knowledge of Mathematics to choose between the possible answers

Give answer (1):If the data in statement I alone are sufficient to answer the question, while the data in
statement II are not sufficient to answer the question.
Give answer (2):If the data in statement II alone are sufficient to answer the question, while the data in
statement I are not sufficient to answer the question.
Give answer (3):If the data either in statement I alone or in statement II alone are sufficient to answer the
question.
Give answer (4):If the data given in both the statement I and II together are not sufficient to answer the
question.
Give answer (5):If the data in both the statement I and II together are necessary to answer the question.
1.The age of Anu and Vinay are in the ratio of 6: 5. What is the age of Vinay?
I.The ages of Anu & Shalu are in the ratio of 3: 2
II.After 6 years the ratio of Shalus & Vinays ages will be 7: 6.
2.In how many days can 5 men complete a piece of work?
I.16 children can complete the same piece of work in 48 days.
II.9 men can complete the same piece of work in 27 days.
3.What is a 3-digit number?
I.One-third of that number is less by 27 from half of that number.
II. One-fifth of that number is 20% of that number.
4.What is salary of C, in a group of A, B, C, D, E and F, whose average salary is Rs 60,600?
I.Total of the salaries of A and E is Rs 64,500.
II.Total of the salaries of B and F is Rs 52,600.
5.What will be the cost of carpeting the floor of a Square room in terms of the area of the room, A ?
I.If the cost of carpeting per square meter is Rsa and the fixed service charge will be Rs. C
II.If the cost of carpeting a room twice as large is Rs. 150 more than the original room.
6.In how many different ways can the letters of the word TROPHY be arranged?
(1) 840
(2) 720
(3) 360
(4) 120
(5) None of these
7.If the numerator of a fraction is increased by 300% and the denominator of the fraction is increased by
200%, the resultant fraction is 3/5. What is the original fraction?
(1) 9/17
(2) 9/20
(3) 6/17
(4) 4/15
(5) None of these

8.Manish covers a distance of 36 km at the speed of 9 km/hr and a distance of 16 km at the speed of 8
km/hr. further, the covers a distance of 24 km at the speed of 4 km/hr. what is his average speed in
covering the whole distance.
(1) 5.5 km/hr
(2) 6 km/hr
(3) 6.3 km/hr
(4) 6.9 km/hr
(5) None of these
9.The mean of the marks obtained by 150 aspirants is 70. If the marks obtained by one of the aspirants
was incorrectly calculated as 85, whereas the actual marks obtained by him was 75, what is the correct
mean of the marks obtained by the aspirants?
(1) 69
(2) 69.93
(3) 70
(4) Cannot be determined
(5) None of these
10.The difference between a two-digit number and the number obtained by interchanging the two digits
of the numbers is 45. What is the difference between the two digits of the number?
(1) 6
(2) 4
(3) 5
(4) Cannot be determined
(5) None of these

Answers with Explanation

bankersadda.com

http://www.bankersadda.com/2015/07/banking-pathway-2015-quant-quiz_11.html

Banking Pathway 2015: Quant Quiz


1. Mr. Rao invests a sum of Rs. 41, 250 at the rate of 6 p.c.p.a. What approximate amount of compound
interest will he obtain at the end of 3 years?
A) Rs. 8,l00
B) Rs 7,425
C) Rs 8,210
D) Rs 7,879
E) Rs 7,295
2.The prot after selling a pair of trousers for Rs. 863 is the same as the loss incurred after selling the
same pair of trousers for Rs. 631. What is the cost price of the pair of trousers?
A) Rs 750
B) Rs 800
C) Rs 763
D) Cannot be determined
E) None of these
3.The ratio of the length and the breadth of a rectangular plot is 6 : 5 respectively; if the breadth of the
plot is 34 metre less than the length, what is the perimeter of the rectangular plot?
A) 374 metre
B) 408 metre
C) 814 metre
D) 748 metre
E) None of these
4. Ninad, Vikas and Manav enter into a partnership. Ninad invests some amount at the beginning. Vikas
invests double the amount after 6 months and Manav invests thrice the amount invested by Ninad aer 8
months. They earn a prot of Rs 45,000 at the end of the year. What is Manavs share in the prot?
A)Rs 25,000
B)Rs 15,000
C)Rs 12,000
D)Rs 9,000
E) None of these
5. The cost of 7 tables and 12 chairs is Rs 48,250. What is the cost of 21 tables and 36 chairs?
A) Rs 96,500
B) Rs l,25,500
C) Rs l,44,750
D) Cannot be determined
E) None of these
6. A can complete a piece of work in 12 days. A and B together can complete the same piece of work
in 8 days. In how many days can B alone complete the same piece of work?
A) 15 days
B) 18 days
C) 24 days
D) 28 days
E) None of these

7. What approximate value should come in place of the question mark (?) in the following question?
6523 544 l.2=?
A)21
B)33
C)14
D)8
E)28
8. Raman drove from home to a neighbouring town at the speed of 50 km/hr and on his return journey, he
drove at the speed of 45 km/hr and also took an hour longer to reach home. What distance did he cover
each way?
A)450 km
B)225 km
C)900 km
D)500 km
E) None of these
9. The average weight of a group of 75 girls was calculated as 47 kg. It was later discovered that the
weight of one of the girls was read as 45 kg, whereas her actual weight was 25 kg. What is the actual
average weight of the group of 75 girls? (Rounded off to two digits after decimal)
A)46.73 kg
B) 46.64 kg
C) 45.96 kg
D) Cannot be determined
E) None of these
10. In how many different ways can the letters of the word MIRACLE be arranged?
A) 720
B) 5040
C) 2520
D) 40320
E) None of these
11. The sum of the two digits of a two-digit number is 12 and the difference between the two-digits of the
two-digit number is 6. What is the two-digit number?
A) 39
B) 84
C) 93
D) Cannot be determined
E) None of these
12. If an amount of Rs. 74, 336 is equally divided amongst 150 people, how much approximate amount
would each person get?
A) Rs. 522
B) Rs. 485
C) Rs. 496
D) Rs. 488
E) Rs. 5 10
13. Mr. Sarang invests 6% of his monthly salary i.e. Rs 2,100 on insurance policies. Also he invests 8% of
his monthly salary on family mediclaim policies and another 9% of his salary on NSCs. What is the total
annual amount invested by Mr. Sarang?
A) Rs 11,400
B) Rs 96,600
C) Rs 8,050
D) Rs 9,500
E) None of these
14. A trader sells 145 metre of cloth for Rs 12,325 at the prot of Rs 10 per metre of cloth. What is the
cost price of 1 metre of cloth?

A) Rs. 65
B) Rs. 75
C) Rs. 95
D) Rs 85
E) None of these
15. There are 7 dozen candles kept in a box. If there are 14 such boxes, how many candles are there in all
the boxes together?
A) 1176
B) 98
C) 1216
D) 168
E) None of these

Answers:1. D
2. E
3. D
4. B
5. C
6. C
7. C
8. A
9. A
10. B
11. E
12. C
13. B
14. B
15. A
1. (D) CI = P [(1+6/100)^T-1 ]
= 41250 [(1+6/100)^3-1 ]
= 41250 ((53 53 53 - 50 50 50)/(50 50 50))
= 41250 ((148857 - 125000)/125000)
= (41250 - 23877)/125000 =Rs. 7879
2.(E) CP of the pair of trousers = (863+631)/2
= 1494/2 = kg 747
3. (D) Let the length and breadth of the plot be 6x metre and 5x metre respectively.
6x 5x = 34
X = 34
Perimeter of the plot = 2(6 * 34 + 5 * 34)
= 748 metre
4.(B) Let the investment of Ninad be Rs x
Investment of Vikas = Rs. 2x
And investment of Manav = Rs. 3x
Ratio of their investments
= x * 12 : 2x * 6 : 3x * 4
= 12x : 12x : 12x = 1 : 1 : 1
Manavs share in the profit = 1/3 * 45000

= Rs. 15000
5. (C) Cost of 21 tables and 36 chairs = 3 * (48250)
= Rs. 144750
6. (C) Work of A for 1 day = 1/12
Work of (A + B) for 1 day = 1/8
Work of B for 1 day = 1/8 1/12 = 1/24
B alone can complete the same work in 24 days.
7. (C) ? = 6523 544 * 1.2
= 6523/544 1.2=14 (App.)
8. (A) Let the distance covered by him each way be x km.
x/45 x/50 = 1
(10x-9x )/450=1
x=450
9. (A) Actual average wt. = (75 47 +25-45)/75
= (3525-20)/75=3505/75
= 46. 73 kg.
10.(B) Reqd. different ways = 7p7
= 7!
= 5040
11. (E) Let the number be 10x + y
x + y = 12
and x y = 6
x=9
and y = 3
or, x =3
and y = 9
Hence the no. may be 93 or 39.
12.(C) The amount each would get = 74336/50
= Rs. 496 (App.)
13. (B) Total amount invested = 2100 * 23/6 = Rs. 8050
Annual investment =Rs. 8050*12 = 96,600
14. (B) Let the cost of 1 metre of cloth be Rs. x.
(x + 10) * 145 = 12325
145x + 1450 = 12325
x = (12325-1450)/145=Rs.75
15. (A) Total no. of candles = 14 * 7 * 12 = 1176

To view the above post in Hindi : Click Here

bankersadda.com

http://www.bankersadda.com/2015/08/banking-pathway-2015-quant-quiz-on-time.html

Banking Pathway 2015: Quant Quiz on Time and work (Previous


Questions)
1. 12 Men can complete a piece of work in 36 days. 18 women can complete the same piece of work in 60
days. 8 men and 20 women work together for 20 days. If only women were to complete the remaining
piece of work in 4 days. How many women would be required?
a.70
b.28
c.66
d.40
e.None of these

2. 9 children can complete a piece of work in 360 days. 18 men can complete the same piece of work in 72
days and 12 women can complete the piece of work in 162 days. In how many days can 4 men, 12 women
and 10 children together complete the piece of work?
a.124
b.81
c.68
d.96
e.None of these

3. Fifty six men can complete a piece of work in 24 days. In how many days can 42 men complete the
same piece of work?
a.18
b.32
c.98
d.48
e.None of these

4. 42 women can do a piece of work in 18 days. How many women would be required to do the same work
in 21 days?
a.36
b.24
c.30
d.44
e.None of these

5. 30 men can do a piece of work in 16 days. How many men would be required to do the same work in 20
days?
a.12
b.36
c.48

d.24
e.None of these

6. 8 men can complete a piece of work in 4 days. 12 women can complete the same piece of work in 4
days whereas 8 children can complete the same piece of work in 8 days. 2 men, 8 children and 3 women
work together for 2 days. If only women were to finish the remaining work in 2 days. How many total
women would be required?
a.12
b.18
c.24
d.20
e.None of these

7. 18 children can do a piece of work in 12 days. How many children would be required to do the same
work in 8 days?
a.12
b.18
c.24
d.27
e.None of these

8. 8 men and 4 women together can complete a piece of work in 6 days. Work done by a man in one day
is double the work done by a woman in one day. If 8 men and 4 women started working and after 2 days,
4 men left and 4 new women joined, in how many more days will the work be completed?
a.5 days
b.8 days
c.6 days
d.4 days
e.9 days

9. B and C together can complete a work in 8 days, A and B together can complete the same work in 12
days and A and C together can complete the same work in 16 days. In how many days can A, B and C
together complete the same work?
a.48/13
b.96/13
c.89/12
d.41/12
e.None of these

10. 10 men can complete a piece of work in 8 days. 20 women can complete the same piece of work in 6
days. In how many days 16 men 18 women together can complete the same piece of work?
a.19/7
b.27/7
c.24/7

d.17/7
e.None of these

bankersadda.com

http://www.bankersadda.com/2015/07/banking-pathway-2015-quant-quiz-time.html

Banking pathway 2015: Quant Quiz (Time & work and Pipe &
Cistern)
Dear Readers,In continuance to the Banking Pathway 2015, we are providing you a Quant quiz which can be
helpful for all the exams including IBPS CWE-RRB,/ IBPS CWE-PO /RBI Assistant and many more.Time
management is also required to gain good marks so here we also providing time to solve this Quiz.
Time: 10 -12 minutes
1. A can do a work in 50 days and B in 40 days . They work together for 10 days and then A leaves and B
have to finish the work alone. How long will B take to finish it??
(a) 11 days
(b) 18 days
(c) 22 days
(d) 26 days
(e) None of these
2. 30 men, working 4 hrs a day can do a piece of work in 10 days. Find the number of days in which 45
men working 8 hrs a day can do twice the work. Assume that 2 men of the first group do as much work in
2 hrs as 4 men of the second group do in 1 hr.
(a) 6(1/3) days
(b) 6(2/3) days
(c) 5(3/6) days
(d) 3(1/6) days
(e) None of these
3. A alone would take 27 hrs more to complete the job than if both A and B would together. If B worked
alone, he took 3 hrs more to complete the job than A and B worked together. What time, would they take if
both A and B worked together?
(a) 8 hours
(b) 10 hours
(c) 9 hours
(d) 6 hours
(e) None of these
4. A and B together can do a piece of work in 12 days which B and C together will do in 16 days. After A
has been working on it for 5 days, and B for 7 days, C finishes it in 13 days. In how many days A,B and C
alone will do the work ?
(a) 16, 48 and 26 days respectively
(b) 16, 48 and 24 days respectively
(c) 26, 48 and 24 days respectively
(d) 16, 46 and 24 days respectively
(e) None of these
5. Two women Ganga and Jamuna, working separately can mow a field in 8 and 12 hours respectively. If
they work for an hour alternately, Ganga beginning at 9 am, when will the mowing be finished?
(a) 9:30 PM
(b) 8:30 PM
(c) 6:00 AM
(d) 7:00 PM
(e) None of these

6. A, B and C together can do a work in 12 days. A alone can do the work in 36 days and B alone can do
the same work in 54 days. Find in what time C alone can do that work?
(a) 9 days
(b) 18 days
(c) 24 days
(d) 27 days
(e) None of these
7. A, B and C together can do a work in 4 days. A alone can do the work in 12 days B alone can do the
same work in 18 days. Find in what time C alone can do the same work alone?
(a) 9 days
(b) 18 days
(c) 27 days
(d) 8 days
(e) None of these
8. A can complete a work in 35 days and B can do the same work in 28 days. If A after doing 10 days,
leaves the work , find in how many days B will do the remaining work?
(a) 15 days
(b) 10 days
(c) 27 days
(d) 24 days
(e) None of these
9. A can complete a work in 24 days and B can complete the same work in 18 days. If A after doing 4 days
leaves the work find in how many days B will complete the remaining work?
(a) 11 days
(b) 15 days
(c) 12 days
(d) 10 days
(e) None of these
10. A and B together can do a piece of work in 6 days, B alone could do it in 8 days. Supposing B works
at it for 5 days, in how many days A alone could finish the remaining work?
(a) 9 days
(b) 8 days
(c) 24 days
(d) 12 days
(e) None of these
11. A and B can do a piece of work in 20 days and 30 days. both starts the work together for some
time,but B leaves the job 5 days before the work is completed. Find the time in which work is completed.
(a) 7 days
(b) 12 days
(c) 14 days
(d) 16 days
(e) None of these
12. Two pipes A & B can fill a tank in 36 hours and 45 hours respectively. If both the pipes are open
simultaneously. How much times will be taken to fill the tank?
(a)20 hour
(b)25 hour

(c)15 hour
(d)30 hour
(e)None of these
13. If A & B two pipes can fill a tank in 10 hour, when A pipe can fill a tank in 6 hour alone ,then in how
much time will be taken to fill/empty the tank when pipe B open alone ?
(a) Empty in 15 hr
(b) Filled in 20 hr
(c) Empty in 20 hr
(d) Filled in 15 hr
(e) None of these
14. Pipe A and B can fill a tank in 10 hour and 12 hour respectively but pipe C can empty the same tank in
15 hour, In how much time it will take fill the tank when the three pipes are opened together?
(a) 8.5 hour
(b) 10 hour
(c) 12 hour
(d) 15 hour
(e) None of these
15. Two pipes A & B fill an empty tank in 40 minutes and 60 minutes respectively, If both pipes are open
simultaneously after how much time should A be closed so that tank is filled in 36 minutes?
(a) 16 min
(b) 20 min
(c) 25 min
(d) 30 min
(e) None of these.
Answers :1. c
2. b
3. c
4. b
5. e
6. d
7. a
8. e
9. b
10. a
11. c
12. a
13. a
14. a
15. a
Explanation :
1. (c) 22 days
Let the total work be 200 work
Efficiency of
A = 200/50 = 4 work/day
B = 200/40 = 5 work/day
A+Bs efficiency = 9/day
A+Bs 10 days work = 9*10 = 90

Remaining work = 200-90 = 110


Time taken by B alone to finish the remaining wrk = 110/5 = 22days
2. (b) 6(2/3) days
M1D1H1/E1W2 = M2D2H2/E2W1 (From MDH Rule)
Efficinecy of first grp : 2nd grp = 2*2 :4*1 = 1:1
Now, D2 = M1D1H1E1W2 / M2H2E2W1
D2 = 30*4*10*1*2 / 45*8*1*1
D2 = 20/3 = 6(2/3) days
3. (c) 9 Hours
Let A+B together takes X hours
A will take X+27 hrs
B will take X+3 hrs
Let the total work be (X+27)(X+3)
Efficiency of A= X+3
B = X+27
Total efficiency = 2X+30
Time working together = (X+27)(X+3) / 2X+30 = X
X^2 +30X + 81 = 2X^2 + 30X
or, X^2 = 81 or X= 9 hrs (neglecting Ve time )
4. (b) 16, 48 and 24 days respectively
Let the total work be 48
Efficiency of
A+B = 4/day (i)
B+C = 3/day..(ii)
Now, A works for 5 days, B works for 7 days and C works for 13 days and completes the total work of 48.
This can be rewritten as
A+B for 5 days + B+C for 2 days + C for 11 days completes the total work of 48
Now, A+Bs 5 days work = 20
B+Cs 2 days work = 6
Therefore, 20+6+ Cs 11 days work = 48
Cs 11 days work = 48-26 = 22
Cs efficiency = 2/day.. (iii)
From (i),(ii),(iii)
Cs efficiency = 2
Bs Efficiency = 1
As efficiency = 3
Time taken by
A= 16 days, B= 48 days, and C= 24 days
5. (e) None of these (6:30PM)
Let the total work be 24
Efficiency of Ganga = 24/8 = 3/hr
Efficiency of Jamuna= 24/12 = 2/hr
They work alternately starting from Ganga
First 2 hrs work = 3+2 = 5
First 8 hrs work = 20
Remaining = 24-20 = 4
9th hr work to be done by Ganga = 3
Remaining work = 4-3 = 1 to be done by Jamuna in 1/2 hr.
Total time = 8+1+(1/2) hrs = 9.5 hrs or 9 Hr 30 minutes
So work will be completed by 9AM + 9 hrs 30 minutes = 18 hrs 30 minutes or 6:30 PM

6. (d) 27 Days
Let the total work be 108 (Common Multiple of 12,36 and 54)
Efficiency of A+B+C =108/12= 9,
of A alone = 108/36 = 3 and
of B alone = 108/54 = 2
Therefore of C alone = 9-(3+2) = 4
Time taken by C = 108/4 = 27 days
7. (a) 9 Days
Let the total work be 36 ( Can take any value Preferably Common Multiple )
Efficiency of
A+B+C = 36/4 = 9
A alone= 36/12 = 3
B alone = 36/18 = 2
C alone = A+B+C- (A+B) = 9-(3+2) = 4
Time taken by C alone = 36/4 = 9 days
8. (e) None of these (20 days)
Let the total work be 140
Efficiency of A = 4
Efficiency of B = 5
A works for 10 days = 4*10 = 40
Remaining work = 140-40 = 100 to be done by B
B will do it in 100/5 = 20 days
9. (b) 15 days
Let the total work be 72
Efficiency of A = 3 and Of B = 4
As 4 days work = 3*4 = 12 remaining work = 72 -12 = 60
Work completed by B in 60/4 = 15 days
10. (a) 9 days
Let the total work be 24
Efficiency of A+B = 4
Efficiency of B = 3
Efficiency of A = 1 as A+B = 4 and B= 3
Work done by B in 5 day = 3*5 = 15
Remaining work = 24-15 = 9
Remaining work to be done by A in 9/1 = 9 days
11. (c) 14 days
Let the total work be 60
Efficiency of A = 3 and of B = 2
Efficiency of A+B = 3+2 = 5
Suppose B never left the work then if the time taken remains same then work done by B in those 5 days will be
added to original work.
Therefore, Now, works become = 60 + Bs 5 days work = 60+10 = 70
Time taken = 70/5 = 14 days
Other way :- B leaves the work 5 days before means A did work alone for that 5 days
Work Done by A in that 5 day = 5/20 = 1/4
Remaining work = 3/4
To complete the work together A+B would have taken 1/ (1/20+ 1/30) = 600/50 = 12 days
3/4th of the work together will be completed in 12*3/4 = 9 days
Total time = 5+9 = 14 days

12. (a) Required Time = 1/(1/45+1/36)= 20 hour


13. (a) 1/6+ 1/B= 1/10
So B = -60/4 =-15
So b will empty the tank in 15 hour
14. (a) Total Time = 1/(1/10 + 1/12-1/15)= 8.5 hour
15. (a) Pipe B will work for 36 min.
in 36 min he will fill the part of tank = 36/60 =3/5 part
Remain part = 2/5
it will filled by A in minutes = 2/5*40 = 16 min
So Tap A should be closed after 16 minutes to fill the tank in 36 minutes.

bankersadda.com

http://www.bankersadda.com/2015/08/banking-pathway-2015-quant-quiz_19.html

Banking Pathway 2015: Quant Quiz


Directions (1-5): Following table shows the total population and ratio of male and female among them of
five cities A, B, C, D and E. Table also shows the literate population and ratio of male and female among
them. Answer these questions based on this table.
1.What is the total number of literate females in all five
states?
(1) 72 lakhs
(2) 84 lakhs
(3) 96 lakhs
(4) 102 lakhs
(5) None of these
2.What is the total number of males in City D who are
not literate?
(1) 12 lakhs
(2) 15 lakhs
(3) 20 lakhs
(4) 24 lakhs
(5) None of these
3.What is the ratio of total females in City A and literate females in the same city?
(1) 2 : 1
(2) 1 : 2
(3) 3 : 2
(4) 2 : 3
(5) 1 : 1
4.The number of literate males in City C is what percentage of total number of males in City A?
(1) 72%
(2) 75%
(3) 80%
(4) 84%
(5) 96%
5.Number of total females in City B is how much percentage more than that of number of literate females
in City A?
(1) 25%
(2) 40%
(3) 60%
(4) 30%
(5) 36%
6.The sum of five consecutive odd numbers is equal to 165. What is the product of the largest and the
smallest numbers?
(1) 1514
(2) 1073
(3) 1123
(4) 1200
(5) None of these

7.Ajay got 86 marks in Hindi, 56 marks in Science, 76 marks in Maths, 48 marks in Social Science and 92
marks in English. The maximum marks of each subject are 100. How much overall percentage of marks
did he get?
(1) 73.8
(2) 72.6
(3) 71.6
(4) 84.8
(5) None of these
8.12 men can complete a piece of work in 20 days. In how many days will 15 men complete the same
piece of work?
(1) 10
(2) 8
(3) 14
(4) 16
(5) None of these
9.Find the highest number of three digits which on dividing by 10, 12 and 15 leaves remainder of 1 in each
case?
(1) 901
(2) 961
(3) 991
(4) 999
(5) None of these
10.Even after giving a discount of 20%, a shopkeeper makes a profit of 10%. Find the mark-up%?
(1) 30%
(2) 35%
(3) 37.5%
(4) 40%
(5) None of these
Answers with Explanation

bankersadda.com

http://www.bankersadda.com/2015/07/banking-pathway-2015-quant-short-tricks.html

Banking Pathway 2015: Quant Short tricks (Time & Work)


Dear Reader,
Time and Work is a very easy topic and at least 2 Questions are asked from this topic in Any Exam whether it's
Clerk or PO or SSC.What actually the Time and Work Topic is all about? In simple words It's how much time a
person will take to Complete a Given Work.
Here we are providing some cases of time and work and efficient method to solve it. Hope this would be helpful to
all aspirants.

These Tricks are shared by one of the our ardent BA reader Insomniac.We wish you good luck for your
future.
Note -: In conventional Method work is always treated as 1
Example: So if I say that a person can complete a work in 15 days that means he will do 1/15 work in one
day, It's simple maths.
Now another person does the same work in 30 days. So he will do 1/30 work in 1 day.
Now if i ask in how many days they will complete the work together. What we gonna do is Add their 1 day
of work
like 1/15 + 1/30 = (2+1)/30 = 3/30 = 1/10
Now this 1/10 we got is actually their 1 day work, So if they do 1/10 work in one day then it's simple they
will complete the whole work in 10 days.
Now that was the conventional method and I believe that you all know how to solve Questions through
Conventional method.
So now lets move on to the Faster method i.e efficiency method.
In efficiency method the Work is not treated in numerical value, Like in Conventional method the work is 1 but here
the work is treated as percentage.
So by common sense the work is always treated as 100%
So when i say a person completes a work in 15 days it means he will do 100/15 % work in 1 day i.e 6.66% work in
1 day
If another person does the work in 30 days that means he will do 3.33% work in 1 day.
And together they will do 6.66 + 3.33 = 9.99 or 10% work in one day So in how many days they will do the
complete work, that will be 100/10 = 10 days.
Now it may sound difficult That we have to convert Each value in % but don't worry you don't have to convert each
value, You just have to learn all the values till 1/30 and then it will be a cakewalk.
Now we will take Some standard Cases Of time and work and you all are free to ask any problem if you have in
any case.
Case 1 - A does a work in X days, B does a Work in Y days In how many days they will complete the work.
Question- A completes the work in 10 days and B completes the work in 15 days In how many days they
will complete the work.
Conventional Method
Work done by A in 1 day = 1/10
Work Done by B in 1 day = 1/15
Work done By A & B together in 1 day = 1/10 + 1/15 = (3+2)/30 = 5/30 = 1/6
As A & B Completes 1/6 work in one day So they will complete the whole work in 6 Days.

Efficiency method.
Efficiency of A =100/10 = 10%
Efficiency of B = 100/15 = 6.66%
Efficiency of A & B Together = 10+ 6.66 = 16.66%
So the time taken by A & B together to Complete the work will be 100/16.66 = 6 Days.
Case -2 A can do a work in X days and B can do it Y days, In how many days the work is completed if they
work alternatively Started by A.
Now in these type of question the person are not actually working together, what happens in this type of question
is that A works for 1 day and then on 2nd day B work and on 3rd again A work and on Fourth again B works and so
on till the work is completed.
For example A can do a work in 10 days B can do it 15 days and how many days they will finish it if The work is
started by A
So again work done by A in one day = 1/10
'' ''
''
''
'' B
''
''
= 1/15
Now the work done by Togther will be = 1/10 + 1/15 = 1/6 [ Note now this 1/6 work is not done by them in 1 day but
in 2 days Actually, See A worked for 1 day and did 1/10 work on the second day B worked and finished the 1/15
work So in total 2 days they did 1/6 work]
So in 2 days they did 1/6 work so in how many days they will complete the whole work, Simple 12 days.
Efficienecy Method
A's Efficiency = 10%
B's Efficiency = 6.66%
A + B Efficiency = 16.66%
Work done by A and B in 2 days [ remember 2 days because they are not working together but working
alternatively] = 16.66%
So time taken by them to complete 100% work = 100/(16.66 = 6 [ but always remember multiply this by 2,
Beacause 16.66% work is done by them in 2 days and not in 1 day.
So The answer will be 6*2 = 12 days.
Case 3: A can do a work in X days, B can do the work Y days and A leaves after working Z days.
Question- A can do a work in 10 days and B can do it in 15 days, A works for 2 days and then leaves, In
how many days will the work be completed?
Now here we can se that A leaves after 2 days that means A and B only worked for 2 days and the remaining work
is done by B alone.
So first we have to calculated the work done by A and B together in 2 days.
So work done be A in 1 day = 1/10
"" "
"
" B " " " = 1/15
Work done by A & B together is 1 day = 1/10 + 1/15 = 1/6
Work done by A & B together in 2 days = (1/6) * 2 = 1/3
So remaining work = 1 - 1/3 = 2/3
Now this 2/3 work has to be done by B alone.
So it takes 15 days for B to do Complete work, How much time it will be taken by B to do 2/3 work ? So it will be
15*(2/3) = 10 days
So the work will be completed in 2 + 10 days = 12 days
Efficiency method
A's efficiency = 10%
B's Efficiency = 6.66%
Total a+b = 16.66%
work done by A and B together in 2 days = 16.66*2 = 33.33%
Work remaining = 66.66%
time taken by B to complete 66.66% work = 66.66/6.666 = 10 days
Total time = 10+2 = 12 days

Case 4
A can do a piece of Work in 10 days and B can do it in 15 days, In how many days will the work be
completed if B leaves 2 days before the completion on work.
Now in this question B leaves before 2 days
Together they can do the work in what = 1/10 + 1/15 = 1/6
That means 6 Days.
That means Together they could have completed the work in 6 days but B works only till 4th day and The
remaining work will be done by A alone
So they worked together for 4 days in all So work done by them in 4 days = (1/6)*4 = 4/6 = 2/3
remaining work = 1/3
Now this 1/3 work will be done by A alone
Now A can do the complete work in 10 days, So the time taken by him to do 1/3 work = 10 * (1/3) = 10/3 days or
3.33 days
So total time taken = 4+ 3.33 days = 7.33 days
Efficiency method
A's efficiency = 10%
B's efficiency = 6.66%
Total = 16.66%
Work will be completed in 6 days
Work done in 4 days = 66.66%
remaining = 33.33%
A will complete the remaining in = 33.33/10 = 3.33
Total = 4+3.33 = 7.33
Case 5: A can do a Work in X days B can Do it in Y days, In how many days The work will get completed if
B leaves 2 days before the actual completion of work.
Question: A can do a work in 10 days B can do it in 15 days, In how many days The work will get
completed if B leaves 2 days before the Actual Completion of Work.what is the difference between this
Actual completion of work and Completion of Work?
See in last example the work was supposed to get completed in 6 days, So we just Solved the question taking into
consideration that B leaves 2 days before the completion of work i.e B worked for 4 days and the rest 2 days work
was don by A alone and Completes that work in 3.33 days Total 7.33 days.
So if i ask In this question If B left 2 days before the actual completion then it means B should have left on 5.33
days Got it ?
Now back to the question.
Now just imagine that the work gets completed in x days.
So A would work for x days[ A works for the whole time ]
And B would work for x-2 days[ because B left 2 days before the actual completion of work]
So now According to Question
x/10 + (x-2)/15 = 1 [ Beacuse work is always one ]
(3x+2x-4)/30 = 1
5x -4 = 30
5x = 34
x = 6.8 days.
So the work will be completed in 6.8 Days.
It can also be asked That after how many days B left, So the answer would be Simple 6.8 - 2 = 4.8 days
Efficiency Method
A's Efficiency = 10%
B's Efficiency = 6.66%
Let the no. of days be x
so Accordring to question

10x + 6.66(x-2) = 100[ Work is always 100% in efficiency method ]


10x + 6.66x - 13.33 = 100
16.66x = 113.33
x = 113.33/16.66 = 6.8
Answer = 6.8 days

Thank you Insomniac :)

bankersadda.com

http://www.bankersadda.com/2015/07/banking-pathway-2015-quant-study-notes_10.html

Banking Pathway 2015: Quant Study Notes & Quiz ( Profit &
Loss)
Dear Reader ,
In continuance to provide study notes(Short Tricks ) today we are Providing Notes on Profit & Loss. Which is very
crucial topic for upcoming competitive examinations.Here we also Providing some concept clearing quiz for
analysing yourself. Time management also very crucial part for gain good marks in exam. so use wisely your
precious time.
These Study notes are provided by one of our ardent BA reader Insomniac and Quiz is provided by Bedanshu.
We wish you both good luck for future.
Profit & Loss:
Cost Price-The price at which an article is purchased is called its cost price (C.P.)
Selling Price-The price at which the article is sold is called its selling price (S.P.)

CP = Cost Price = The price at which an object is Purchased


SP = Selling Price = The price at which the object is Sold.
When SP < CP Loss = CP - SP
When SP > CP Profit = SP - CP
Note: Loss% and Profit% both are calculated upon CP
Profit% = [Profit/CP] * 100
Loss% = [Loss/CP] * 100
Suppose Company A produces 1000 T.V in Year 2000 And 1200 T.V in Year 2001. On the other hand
Company B produces 5000 T.V in year 2000 and 5500 T.V in year 2001. Which company has the better
growth rate ?
Now Look at company A, The increase is of 200 T.V and Company B the increase if of 500 T.V So in Numerical
Sense Company B has Produced more than A.
But we are not talking here about Numerical Growth We are talking about relative Growth. i.e Growth with respect
to it's previous year production. [ Like things are compared with like thing Only, Just Like you can't compare Apple
and Oranges]
Taking That Point Into Consideration The Growth of Company A with respect to it's Previous year production will
be (200/1000)*100 = 20%
And that for company B it will be (500/5000)*100 = 10%
So clearly Company A has a better growth rate than company B.
Cost Price aka CP
In my methods I Consider CP to be an Absolute Value of 100%. So if anybody Says he made profit of 20% it
means He sells the Object at 120% or C.P 100% and Profit is 20% then it means SP = 1.2x( It will be more clear
to you when i will explain Different Case)
Profit or Loss both are calculated with respect to C.P i.e CP is Always used as a base while calculating profit and
loss.
Selling Price aka SP
I think by Common Sense you all know that If SP>CP then you will have profit whose value will be (SP - CP) In
terms Of numerical Value.
And Profit % will be [(SP-CP)/CP] * 100 or [(Profit/CP)*100] [ remember i told you that profit and loss both are
calculated on CP i.e taking CP as Base, So all you have to do is calculate Profit in terms of numerical value( SP CP) and then divide it by the base(CP) and then multiply it by 100 and you will get your profit %]

Eg. CP of a pen is 10 Rs and SP is 12 Rs. What is profit and profit % ?


Pretty easy Huh !! Just calculate profit first So it will be SP - CP = 12-10 = 2RS
And profit % [(SP-CP)/CP]*100 = [(12-10)/10]*100 = [(2/10)*100] = 20%
So now I think The difference between profit and profit % is clear to you.
Now If you know What is Profit then you all must know That what is Loss and when loss Occurs.
Loss occurs when we make some pretty bad decisions and We go out Nuts and Start selling The object
at a rate less than the purchasing price of the object.
Lets Put that in Mathematical Way. If CP>SP then there is a case of loss. to Find out the amount of loss all
you have to do is (CP - SP)
And to calculate Loss% nothing difficult just the usual stuff [(CP-SP)/CP*100] or [(Loss)/CP*100] [ Look again I
told you both Loss and profit are calculated taking CP as Base. So what i have done in the formula is That i
calculated Numerical Loss and then divided it by our BASE i.e CP and Then Multiplies it by 100 To get our Loss%]
And i don't think I have to explain again the difference between loss and Loss %.
So now Moving On to Other basic Stuff.
Marked Price aka MP= The Price at which a Product is Marked[ Like when you go to Your Local Market for
buying Some nice Sunglasses( I mean fake RAY BAN's ;) ) And the Dealer say the cost is 1000rs and It's Also
Marked on the Box but as we all know that it's just a MARKED PRICE and he will eventually sell that Ray Ban to
you at 200Rs, And well if you are good at Bargaining then he will even sell it you at 100RS, And If It's me He will
give it ME for free and even pay me 50RS back :P Well just Joking :P So that is our Marked Price]
Discount % = It's like concession on the MARKED PRICE. The dealer says I Am just giving You a discount
Man You are a regular customer and blah blah blah And you are like my relative and all the BS( But here is the
catch The Discount % is always calculated on M.P In the above example of RAY BAN if you want to calculate the
Discount % then It's Easy The MP was 1000Rs he Finally Sells you that Ray Ban at 200 So discount Given = MP SP i.e 1000-200 = 800rs
Now Discount % is calculated taking MP as BASE so Discount % will be
[(MP-SP)/MP]*100 = [(1000-200)/1000]*100 = 80%]
Discount is calculated on MP but Marking of MP is done with respect to CP.
For example if I say I bought Something for 500rs and I marked the Product 60% above the CP then It means I
marked the product 60% of 500 = 300 above CP means 500+300 = 800rs. OK
CASE 1- Simple Profit and Profit % Calculation
Mohit Purchased A watch for 1000rs and then Sold it to Nimesh for 1250rs. Calculate the Profit and Profit
%?
Most Simple Question Which You will never get in Any Exam :P( But Basics are Basics we gotta revise it at least)
So What happens here Mohit purchases a Watch( You see word Purchase And You know it's CP) at 1000rs
SO C.P = 1000rs
And then he sells it at 1250Rs( You See the Word SELL Ok that's our SP )
So SP = 1250Rs
Now Profit as I told is nothing but SP - CP So profit = 1250-1000 = 250rs
Now Profit % = [(Profit)/CP*100] So profit % = [(250/1000)*100] = 25%
CASE - 2
Now The Watch That Nimesh purchased for 1250rs Is Sold Again by Her at Rs 625. So what will be Loss and
Loss %
Again Usual Stuff
Loss = CP - SP
Loss = 1250 -625 = 625
Loss% = [(loss/CP)*100] = [(625/125)*100] = 50%
So we have a 50% loss here.

Case 3 Inversion case


Profit or loss% is Give and CP or SP is Given and you have to find SP or CP
Steve Sells an article for 1200Rs And he makes a profit of 20% in the Transaction. So What is the Cost
price?
I told You once If you don't Know about Something Then assume it as x.
So we take CP = x
Now If i sell an article at 20% profit then what will be our SP in terms of x ?
yeah it's pretty simple 1.2x [ Because is told you percentage to decimal conersion So 20% here is nothing but 0.2x
and total SP will be x+ 0.2x = 1.2x remember add in case of profit and subtract in case of loss]
And according to the Question he sold the article at 1200rs
So 1.2x = 1200rs
So x = 1000Rs.
Steve again sells an article for 1200rs but this time he suffers a loss of 20%. What will be the C.P?
Now just Take CP = x
So as i Told S.P will be ? yeah 0.8x ( As i said add in case of profit and subtact in case of loss)
and acoording to the question SP = 1200rs
So 0.8x = 1200
x = 1200/0.8 = 1500rs
So C.P is 1500rs.
If it's given that C.P is 1000rs and profit made is 20%
Then it will be much more simple.
C.P is 1000rs so profit 20% will be 200rs.
So Profit = sp-cp therefore SP = CP + profit = 1000+200=1200rs.
Or If S.P is Given and Also Discount % is given you have to calculate MP
Example S.P = 200rs
Discount % = 80%
Find MP.
Simple Let M.P be X
So S.P after 80% discount will be 0.2x
and according to question S.P = 200
So 0.2x= 200
x = 1000rs = MP
Case 4: Combination Where MP CP SP are Mixed Together.
An article was purchased for Rs. 78,350. Its price was marked up by 30%. It was sold at a discount of 20%
on the marked up price. What was the profit percent on the cost price?
Cost price = Rs. 78350
Marked price = 1.3*78350 = 101855( I told in the Start CP is Absolute 100% so If anything is Marked or Sold
Above CP by ---% You just have to add that % to 100% and Convert it into it's relative decimal value)
Selling price = 101855*0.8= 81484[ Discount is calculated on MP so here MP is Absolute 100%, 20% discount
that Means The product is being Sold at 80% of MP or 0.8MP)
Profit = S.P - CP = 81484 - 78350 = 3134
Profit% = (Profit/CP)*100 = (3134/78350)*100 = 4%
Case 5
A man purchases 11 oranges for 10rs and Sells 10 oranges for 11rs.Find profit or loss%?
1st- Basic method.
Find the C.P of 1 orange that will be 10/11
Find SP of 1 orange that will be 11/10
As it's clear SP>CP hence Profit will be SP-CP = 11/10 - 10/11 = (121 - 100)/110 = 21/110

Profit % = [(profit)/CP]*100 = [(21/110)/(10/11)*100] = [(21*11)/(110*10)*100 = 21%


2nd- LCM method.
Take The LCM The two numbers present in the Question i.e LCM of 10 and 11 = 110.
Now this Is the Amount Of oranges you will buy and sell and calculate the profit % on that.
So CP of 110 Oranges = 100rs
S.P of 110 Oranges = 121rs
Profit = 21Rs
Profit % = 21%[ calculates on the CP of 110 Oranges]
3rd - fastest Method but Confusing
Write The Purchase line First --- 11 oranges for 10
-------------------------------------------------cross multiply----Selling Line 2nd
10 oranges for 11
11*11 will be SP and 10*10 will Be CP
Profit % = (11*11 - 10*10)/(10*10)*100 = 21%
Lets See one More Example
A man buys 8 pens for rs 9 and sells 9 pens for rs 8, Find profit or loss%?
By LCM methdo
take lcm of 8 and 9 that will be 72
CP of 72 pens will be 81rs
SP of 72 Pens will be 64Rs
Clearly there is loss which will be equal to 81 - 64 = 17rs
Loss % = (17/81)*100 = 20.98 or 21%
BY Fastest Method.
Write the purchase line 8 pen for 9rs
---------------------------------cross multiply
write sale line
9 pen for 8rs
C.P will be 8*8 = 64
S.P will be 9*9 = 81
Loss% will be (17/81)*100 = 20.98%
Case 6:- Dishonest dealer using false Weight and selling at Cost price.
A Dishonest dealer Professes to Sell the product at cost price but Instead of Selling 1000gm He sells only
900gm for 1Kg Wt
You don't need any Formula to Solve This Type of Question. You just have to use your own Mind here.
Now Look what the dealer is actually Doing here.
Dealer says He sell at Cost price Means He say He Sells at the price he purchases.
Now What amount He says He Sells = 1000gm
What Actually He Sells = 900gm
Now you can See here he is only selling 900gm and he is getting the oney for 1000gm
So this money from 100 Grams is His Profit OK.
Now how we calculate profit % ? We take CP as Base and Divide the Profit by CP.
Now look in this Question he is Selling 900 Gram and getting 100gram As profit.
So profit % will be (100/900)*100 = 11.11%
One More Question
A dishonest dealer Professes to sell the goods at cost price but instead of selling 1000 gms he sells only
800 gms for 1KG WT. Find his Gain% ?
Now Just Remember what He says He is Selling or what he gets paid for, he says he sell 1000gram
But What Actually He sells ? yeah exactly 800grams.
So how much he gets Extra or how much he cheats = 200grams
So profit will be (200/800)*100 = 25%

Case 7: Dishonest Dealer and also Selling Above Cost price.


A dishonest dealer Sells his Good 20% above the cost Price and Also cheats the Customer By giving them only
800gm for 1kg wt. What's his Profit % in the whole transaction.
We have to do the same stuff here Just Imagine. If he sells 1kg then how much will get paid for but also remember
that he sells his good above 20% of CP Which means that if he sells 1000gm he gets paid for 1200gm. [ 20%
above CP ka matlab yahi hua na ?]
So he gets paid for 1200gms and What he actually Sells here is ? Yes 800gms Only
So Profit will be SP - CP = 1200 - 800 = 400gm
Profit % will be (Profit)/CP*100 = [400/800]*100 = 50%
Case 8: When two Articles are Sold at Same Price but one at profit and one at loss and % profit = % loss.
In Such Cases there will always be a loss (%)which will be equal to [(Common Gain or Loss)]/10]^2
Example - A man Sell two Wrist Watches One at a profit of 20% and one at loss of 20%. The selling price of
each watch is 200rs.
i) Find the Percentage of profit or loss.
ii) Net Amount of profit or loss.
i) As i told there will always be a loss in this case And % loss = [(common gain or loss)/100]^100
Now just put the value % loss = [(20/10)]^2 = 4%
Hence Loss % = 4%
ii) Net Amount of Loss
So His Total SP was 200 + 200 = 400rs
He Suffers a loss = 4% Which means he sells his watch at 96% of their value i.e CP
So acording to Question 96% of CP = 400rs
or 0.96CP = 400rs
CP = 400/0.96
CP = 416.6667
So Net Amount of Loss = CP - SP = 416.667 - 400 = 16.667Rs
Case 9 - Goods passing through Successive hands.
It's a Lot like the method i told you yesterday about consecutive increase or decrease.
But Let's just Check it again.
A sells a good a profit of 20% to B and B Sells That Good to C at a profit of 25% If C pays 225 For It. What was it's
Cpst Price for A.
So Just Asumme that CP for A was x
So he sells it at 20% profit That means he sells it at 1.2x
Now S.P of A = C.P of B
So Now B sells it to C at 25% profit
That means B sells it at 1.2*1.25*x
Now C pays 225rs
That mean 1.2*1.25*x = 225
so x = 150Rs
Answer CP for A = 150Rs.
Or you can Also use the formula which i told yesterday [x + y + (xy)/100]
Same way you can solve for 3 persons also.
Case 10 - CP of X articles = SP of Y Articles.
Very Simple If you know the trick behind it.
Eg - CP of 25 Articles is Equal to the SP of 20 Articles. Find the Profit or loss %.
Just write it This was 25CP = 20SP
Now Cross multiply So that Variables gets on One side of the equation and Constant gets on the Other Side.
So SP/CP = 25/20

Now you just have to take that Elements on the opposite sides of Equation represents their corresponding value.
So in Equation SP/CP = 25/20. In front of SP the value is 25( So our SP will be 25)
And in front of CP the value is 20( So out CP is 20)
Now You know CP and SP calculating profit or loss is a child's play now but still we have to play it[ Personal
Advice Always Believe in complete solution of the question, never leave the question in Mid Way ]
So as SP>CP there is profit
And profit will be [(SP-CP)/CP]*100 = (5/20)*100 = 25%
Another Example
CP of 10 articles is equal to the SP of 12 Articles Find the profit or Loss %?
Do the same stuff again
10CP=12SP
Cross multiply now.
SP/CP = 10/12
So SP = 10
and CP = 12
So clearly there is a loss And loss % = (Loss/CP)*100 = (2/12)*100 = 16.66%
Case 1: Marking Above x% and giving discount of y%, Total profit or loss.
Eg A person Marks his good above 50% of CP and Gives Discount of 20% Find his Profit %.
The easiest way to solve this type of question is to assume the CP as 100
So CP is 100
M.P will be 50% above CP that will be 150
Now he gives discount of 20%
As discount is caculated on MP so SP after deducting the discount will be 150*0.8 = 120
Now SP = 120, CP = 100 So profit % will be 20%.
Case 12 - Decrease in Price of Commodity allows A person to Buy X quantity more of an item.
EG - When the price of sugar decreases by 10%, a man could buy 1 kg more for Rs 270. Then, the
original price and final price of the sugar are ?
Now remember i told you a formula yesterday Which Goes something like this[ (How Much It is decreased)/( What
It Becomes after decrease)*100].
So this Question is Implementation of that Formula only.
Price is decreased by 10%.
So Man can purchase how much extra now ? Apply the formula ( How much decrease/ What It becomes )*100 =
(10/90)*100 = 100/9 %
So man can buy 100/9 % more sugar now.
Lets Assume that originally He used to buy x kg of sugar
And as it's given in the question He can Buy 1 KG more. So that means that 100/9 % of x = 1kg
(100/900)*x = 1
x = 9kg.
Now Original Quantity = 9kg
So Original Cost = 270/9 = 30Rs/KG
Increased Quantity = 9+1 = 10kg
So Final Price = 270/10 = 27RS/kg
Case 13
A trader allows a discount of 25% on his articles but wants to gain 50% gain. How many times the CP
should be marked on the items?
Simple way to solve this Question is By Assuming MP as X and CP as Y.
So Let MP be X, So SP after 25% discount = 0.75x
And He also Wants to Again 50% on CP, So SP in Terms of y will be = 1.5y
Now Both SP are Equal So

0.75x = 1.5y[ Now we have to find MP with respect to CP So express the equation in terms of Y]
x = 2y
or x = 200% of Y
So he Should marks his Goods 100% above the CP.
Case 14:- Successive Discount.
We all used to get Amazed when we heard deals like 50% + 49% discount, I always used to wonder how can they
sell their product at 1% price LOL. Then i studied % in class 7th and it became clear to me that's it's another way
of looting commom man.
So what actually is Successive Discount.
Successive simply means anything which is applied in succession ( ek ke baad ek apply karna )
So When Pantaloons Say 50% + 30% off It doesn't mean you will get the discount of 80%. If they wanted
to give you 80% discount( which they would never do) then they would simply have written 80% instead
of 50% + 30%.
For Example You wen to Pantaloons or levis whatever And You Like a jeans Whose MP is 1000rs, and there is a
discount of 50% + 30%. So Now You have to apply the 50% discount, By applying that New MP will become
500rs and Now On this 500Rs you will apply the next 30% discount to get the final SP which will be 350Rs.
So Lets See some Examples.
There are 2 Successive discount on Watch Whose MP is 2000rs. the first Discount is of 40% and other is
of 20%.
The Good thing with successive discount is that you can apply The discount in any way you want, that means you
can apply 20% discount first and if you want you can apply 40% discount first. The answer will remain the same.
So now Lets Apply 40% discount on 2000rs. After applying 40% discount the MP will become 1200rs and On that
1200 we apply another 20% discount So the final SP will become 960Rs.
Now Do the Other Way. First Apply 20% discount on 2000rs So new MP will be 1600 Rs Now apply 40% discount
again. And the Final SP will be 960 Rs
You can see the answer is same in both the cases.
But I will tell you simple method Just Multiply It.
I means MP is 2000 You want to apply 40% and 20% Discount Just do it like this was 2000*0.6*0.8 = 960
Sometimes It's Also Asked two successive discount of 30% and 40% is Equal to what Single Discount.
No need to worry Just do the regular Stuff. If MP was 1 after 30% discount it will become 0.7 and after 40% it will
be 0.6
So multiply the values.
0.7*0.6 = 0.42.
Now This 0.42 is The Final SP
So total Discount will be equal to 1- 0.42 = 0.58 or 58%
Lets see 1 more example.
What will the Single Equivalent discount for two Successive Discount of 40% and 50%?
Let MP = 1
Now apply discount 0.6*0.5 = 0.3 = SP
So Discount = 1- 0.3 = 0.7 or 70%.
Case-15: Equation Based Question,
it's not a single case many Question can be Made From This case But basic idea is you have to make a
mathematical Equation To Solve Such type of Questions.
A trader gets a profit of 25% on an article. If he buys the article at 10% lesser price and sells it for Rs. 2 less, he
still gets 25% profit. Find the actual CP of the article.
Let Assume the CP of the article was x. So according to Question The SP must have been 1.25x
Now He buys the article at 10% lesser price that means he buys it at 0.9x
And he sells it 2rs less which means at 1.25x - 2
He will still get 25% profit But This 25% will be calculated on 0.9x because it's the new CP
So 1.25x - 2 - 0.9x = 25% of 0.9x

0.35x -2 = 0.225x
0.35x - 0.225x = 2
0.125x = 2
x = 2/0.125 = 2000/125 = 16
So CP = 16.
Another One
A trader Sells an Article at 25% profit If he had Sold the item at 10 Rs. more the profit would have been
30%. Find The CP?
It's very simple question In this type of question just assume CP as x.
And Convert the % value of Profit into decimal and Then Solve the question Accordingly.
25% of x = 0.25x
and 30% of x= 0.3x
Now in the Question it is said The dealer would get 10rs more if the profit is 30% Or the difference between 25%
profit and 30% profit is 10Rs
So 0.3x - 0.25x = 10
0.05x = 10
x = 10/0.05 = 1000/5 = 200
One More Question, A dealer Sells an Article at 20% profit If he had sold the article at 500rs less he would
have suffered a loss of 30%. Find CP
Just Take CP as x
so 20% profit will be = 0.2x
30% loss = -0.3x[ remember loss is assigned as negative]
So according to Question the Difference between 20% profit and 30% loss is 500rs
So 0.2x - ( -0.3x) = 500
0.2x + 0.3x = 500
0.5x = 500
x = 500/0.5 = 5000/5 = 1000

Quiz :
Time: (5-6) minutes.
1. Aadesh bought a combined total of 25 monitors and printers. He marked up the monitors by 20% on CP
while each printer was marked up by Rs. 2000. He was able to sell 75% of the monitors and 2 printers and
make a profit of Rs. 49,000. The remaining monitors and 3 printers could not be sold by him. Find his
overall profit or loss if he gets no return on unsold items and it is known that a printer costs 50% of a
monitor.
(a) Loss of Rs. 48,500
(b) Loss of Rs. 21,000
(c) Loss of Rs. 41,000
(d) Data Inadequate
(e) None of these
2. A rickshaw dealer buys 30 rickshaws for Rs.4725. Of these, 8 are four seaters and rest are two seaters.
At what price must he sell the four seaters so that if he sells the two seaters at 3/4th of this price, he
makes a profit of 40% on his outlay.
(a) Rs. 180
(b) Rs. 270
(c) Rs. 360
(d) Rs. 450
(e) None of these

3. Ritesh bought 25 washing machines and microwave ovens for Rs. 2,05,000. He sold 80% of the
washing machines and 12 microwaves ovens for a profit of Rs 40,000. Each washing machine was
marked up by 20% over cost and each microwave oven was sold at a profit of Rs. 2,000. The remaining
washing machines and 3 microwave ovens could not be sold. What is Raghavs overall profit/loss?
(a) Rs. 1000 profit
(b) Rs. 2500 loss
(c) Rs. 1000 loss
(d) Cannot be determined
(e) None of these.
4. A flat and a piece of land were bought by two friends Tarun and Varun respectively at prices of Rs.
2Lakh and Rs. 2.2 Lakh. The price of the flat rises by 20% every year and that of land by 10% every year.
After two years, they decided to exchange their possessions. What is approx. percentage gain of the
gainer?
(a) 7.56%
(b) 6.36%
(c) 4.39%
(d) 3.36%
(e) None of these
5. Sunil calculates his profit percentage on the selling price whereas Sujeet calculates his profit on the
cost price. They find that the difference of their profits is Rs. 900. If the selling price of both of them are
the same, and Sunil gets 50% profit and Sujeet gets 40% profit, then find their selling price.
(a) Rs 4200
(b) Rs 4500
(c) Rs 4000
(d) Rs 4800
(e) None of these
6. A reduction of 10% in the price of salt enables a person to buy 2 kg more for Rs.180. Find the reduced
and the original price per kg of salt respectively.
(a) Rs 10, Rs 9
(b) Rs 9, Rs 10
(c) Rs 18, Rs 20
(d) Rs 20, Rs 18
(e) Rs 18, Rs 16.2
7. A person sold his watch for Rs. 24. If the percentage of his loss was equal to the cost price , then the
watch would have cost him
(a) Rs. 40
(b) Rs. 60
(c) Rs. 50
(d) Rs. 80
(e) None of these
8. A man buys two horses for Rs. 1550. He sells one so as to lose 23% and other so as to gain 27%. On
the whole transaction he neither gains nor loses. What does each horse costs?
(a) 807,743
(b) 817,733
(c) 827,723
(d) 837,713
(e) None of these
9. An orange vendor makes a profit of 20% by selling oranges at a certain price. If he charges Rs. 1.2

higher per orange he would gain 40%. Find the original price at which he sold an orange.
(a) Rs. 3
(b) Rs. 12
(c) Rs. 4.8
(d) Rs. 6.0
(e) None of these
10. After selling a watch, shyam found that he had made a loss of 10%. He also found that had he sold it
for Rs.27 more, he would have made a profit of 5%. the actual initial loss was what percentage of the
profit earned,had he sold the watch for a 5% profit?
(a) 23%
(b) 150%
(c) 200%
(d) 180%
(e) None of these.
Thank you Insomniac and Bedanshu .

Answers & Explanation;


1. a
2. b
3. c
4. e (8.189 approx. )
5. a
6. b
7. e (Either Rs 40 or Rs 60)
8. d
9. e (Rs. 3.60)
10. c
Explanation:
1.Total Number of printers = 5 (2 sold , 3 unsold)
Monitors = 20.
Profit made on Printers sold = 2000*2 = 4000.
Monitors sold = 20*75% = 15
Profit made on Monitors sold = 49000-4000 = Rs.45000.
Profit made per monitor = 45000/15 = 3000.
20% of CP of Monitor = 3000
CP of Monitor = 15000.
CP of Printer = 7500
Total CP = 15000*20 + 7500*5 = 3,37,500
Total SP = 18000*15 + 9500*2 = 2,89,000
Loss = 48,500
2. Total investement = Rs. 4725
Total SP = 1.4*4725 = 6615
Now, Let the price of 4 seater be x then price of two seater will be .75x.
8x + 22*0.75x = 6615
24.5x= 6615 or x = 270
3. Total number of Microwave ovens = 15 (12 sold +3 unsold)

Hence, Washing machines = 10


He sold 12 ovens and 8 washing machines
Hence, In total he sold 80% of both
Thus, He sells 80% of both at a profit of Rs. 40,000.
Cost of 80% of the goods = 0.8*2,05,000 = 1,64,000
Hence, Total SP = 1,64,000+40,000 = 2,04,000
CP = 2,05,000
Loss = Rs.1000
4. After 2 years :Flat would be worth = 2Lakh* 1.2*1.2 = Rs. 288000
Land would be worth= 2.2Lakh*1.1*1.1 = Rs. 266200
Profit of the Gainer = Rs. 21800
Profit % of the gainer = 21800*100/266200= 8.189(approx)
Also if loss% woudd have been asked of the loser
loss% = 21800*100/288000 = 7.56 (approx. )
5. Let SP be Rs. 100
CP for Sunil = => (SP-CP)*100/SP = 50
CP for Sunil ==> (100-CP)*100/100 = 50 or CP = Rs. 50
(Divided by SP as Profit calculated on SP)
Profit for Sunil = 100-50 = Rs 50
Now, CP for Sujeet = (SP-CP)*100/CP = 40
(100-CP)*100= 40CP or CP for Sujeet= Rs. 1000/14
Profit for Sujeet = 100-100/14 = 400/14
Now, Difference in profit when SP is 100 = 50-400/14 = 300/14.
Now, Equating difference and SP, we have
300/14 : 100 : : 900 : SP
SP = 900*100*14/300 = Rs. 4200
6 . Let originally he buy X kg for Rs. 180
Now, he will buy X+2 kg for Rs. 180.
Reduction in original price =10%
(180/X)/kg*90/100 = [180/(X+2)]/kg
90(X+2) = 100X
X = 18
Therefore, Originally he bought 18kg.
Original Price = Rs. 10/kg
Reduced Price = Rs. 9/kg
7. SP = Rs. 24
Let CP be X hence, Loss% = X
(X-SP)*100/X = X or (X-24)*100/X = X
X^2-100X+2400 = 0
(X-60)(X-40) = 0
X= 60 or 40
8. Let CP of one be X and other be Y
X+Y = 1550.(i)
ATQ:0.77X + 1.27Y = 1550(ii) (as no profit and no loss is there)
Solving both, we get
50Y = 35650 or Y = 713
Hence, X = 1550-713 = 837

Therefore, CP of each horse = 837,713


9. Let the CP be Rs. x/ orange
Profit = 20%
SP = Rs. 1.20x
Now, If case :SP =x+ Rs.1.2
Profit = 40%
Therefore, we can say
1.40x = x+1.2 or x = Rs. 3
Hence, Original SP = Rs 1.2x = Rs. 3.60/10. Profit= 5% (If case )
5% of CP ------> Rs. 27
So, CP = Rs. 540
Now, Loss% = 10
Loss =Rs. 54
Required % = 54*100/27 = 200%
To view the above quiz in hindi click here

bankersadda.com

http://www.bankersadda.com/2015/07/banking-pathway-2015-quant-study-notes.html

Banking Pathway 2015: Quant Study Notes & Quiz (Permutation


& Combination and Probability)
Dear Reader ,
In continuance to provide study notes(Short Tricks ) today we are Providing Notes on Permutation& combination
and Probability. Which is very crucial topic for upcoming competitive examinations.Here we also Providing some
concept clearing quiz for analysing yourself. Time management also very crucial part for gain good marks in exam.
so use wisely your time.
These Study notes are provided by one of our ardent BA reader Insomniac. We wish you good luck for future .
Permutation& Combination and Probability:
Permutation and Combination are not that important for the purpose of exam Because Question are rarely asked
from This Topic but We have to learn them anyway because Question of probability can't be solved without
learning permutation and combination. So will give you all a little hint about what is permutation and what is
combination and then we will move on to Probability.
But Before That Just Look at A very Important Concept Without Which You can't solve a single Question of
permutation/combination or probability.
And that Factorial Notation.
It's represented by (!) and it is read as Factorial.
So if i write 5! it will be read as Five Factorial.
And what it means ? It means to simply multiply all the numbers in decreasing order till 1.
Like if i write 6! it means 6*5*4*3*2*1 = 720
Or 7! = 7*6*5*4*3*2*1 = 5040
For Fast Calculation You all must learn the value of factorial till 10.
Just Learn these values
1! = 1
2! = 2
3!= 6
4! = 24
5! = 120
6! = 720
7! = 5040
8! = 40320
9! = 362880
10! = 3628800
Well Before I Start Explaining Permutation and combination one thing i want to tell and that is It's the easiest topic
that you will find in maths. Most people are unable to understand it and that's why people think it's complex and all
type of misconceptions but trust me it's the easiest topic in the whole mathematics and It's not actually even maths,
It's less about Calculation and more about Logical Thinking. Well We all can't Calculate Fast but we all can think
fast.
So what is permutation?
In Simple words it's arrangement or No. of ways things can be arranged.
Suppose there are 3 words ABC and if it's asked How many ways these three can be arranged then all yu or What
are the no. of permutations Possible. Then all you have to do is Arrange this things in as many ways it's Possible.
Let's try to arrange them now. SO There is ABC, ACB, BAC, BCA, CAB, CBA Are there any more ways these can
be arranged ? try it ? No These are the all possible arrangements. So The answer to the above Question will be 6.
That is ABC can be arranges in different ways.
Now there were only 3 alphabets What if there were more like You have to Arrange ABCDEFGHI. Now for 3
alphabets it was easy you easily arranged them But Arranging these 9 letters will take you days and even then you
will not be able to get a certain answer.
So what we should do here. No need to worry our mathematicians were genius they created a very simple formula

for that.
And Formula is like this.
N Different things can be arranged in N! ways.
So in above Question there were 9 alphabets so the no. of possibele arrangements will be 9! = 362880.
So that was out basic concept Now let's move on to another basic concept.
So in the above questions It was Asked in how many ways ABCDEFGHI Can be arranged. In this question they
were asking the possible arrangements of all the 9 Alphabets, They can also Ask In how many ways 4 alphabets
from above 9 alphabets can be arranged.
In such type of Questions there is another formula Which is very very very important because it will be used in
almost every question.
So the formula is Out of n things r things can be arranged in nPr ways. and
nPr = n!/(n-r)!
So in the above Question it is asked that in how many ways 4 alphabets from the total 9 alphabets can be
arranged.
So apply the formula nPr = 9P4 = 9!/(9-4)! = 9*8*7*6*5*4*3*2*1/5*4*3*2*1 =9*8*7*6 = 3024.
Now there is a trick to easily calculate nPr by which you won't have to do any division work.
Like if it say 9P3 then you just have to multiply Starting from 9 in decreasing order till the next 2 digit i.e 9P3 =
9*8*7. Why we multiply till 7 only ? that is because the value of r is 3 and total multiplication should contain the
value of r.
Another example if it 7P2 then you will just do 7*6[ 2 number because r = 2 ok]
if it's 7P4 then the answer will be 7*6*5*3[ 4 no. because value of r=4]
So If it's 10P5 then the value will be 10*9*8*7*6 [ 5 digit because value of r = 5]
I think you understand my point now. Now move on to the cases.
Actually there are infinite cases in Permutation and Combination 100's of different type of question can be formed
So i will only discuss the cases that are important for the exam, And if you have any problem in any other case
then you can ask me personally.
Case - 1 Simple Arrangement Case well all words are unique.
By UNIQUE i mean all alphabets are different
In how many ways the letters of the word ROCKET can be arrnged.
very Simple just count the no. of words in ROCKET that will be 6
So number of arrangements will be n! that will be 6!
CASE - 2 Arrangement When All the words are not UNIQUE.
That means some words are repeated.
Like No. of possible arrangements of word TITANIC
Now In this case you Just have to find the total possible ways first without even thinking about Repeated words
and then after that You will divide that with the numbers of times a Word is repeated.
So in the above Question Total alphabets = T = 2, I = 2. A= 1 C =1 N = 1Total 7 So Permutations will be 7! and
Now you will divide It by No. of times A word is repeated SO T is repeated 2 times and I is repeated two times So
divde 7! by these 2. So final Answer will be 7!/(2!*2!)
Let's See another Example. In how many ways the letters of the word RUNNING Can be arranged.
So total no. of alphabets in the above Words = 7
No. of words that are repeated = N = 3 times repeated.
So the solution will be Total permutation divided by no. of times a word is repeated and that will be 7!/3! that will be
your answer.
Case 3 - Arrangement Some Words are always together and Some Words and Never together .
No of possible arrangements of the words LAYERING When Vowels are always together.
In this case what we do Is we consider the no. of Vowels as 1 single alphabet That [AEI] is a one single alphabet
In that way they will always be together and the rest words are LYRNG.
So the total no. of alphabets will be 6 ? Why 5 Alphabets are LYRNG and [AEI] is One alhpabet remember so The
total alphabet will be 6
And no. of possible arrangements will be 6!
But but the question is not complete yet [AEI] Though considered as 1 alphabet but stil the words AEI can change
places within itself Like AEI it also can be AIE or EIA. So there are 3 words so no. of total arrangements that they
can do within itself will be 3!

So our final answer will be 6!*3![ that is because 6! is the no. of possible ways when AEI are together and And
multiplied by 3! because AEI can change places within themselves in 3! possible Ways]
If it was asked that VOWELS in LAYERING are never together that what we will do ?
This Question can't be solved directly.
In order to solve this We will have to FIND the total no. of arrangements of the word LAYERING and then Subtract
the no. of arrangemnts in which AEI are Always together.
So no. of possible arrangements of LAYERING will be 8!
And We already Solved that when AEI are always together the no. of possible ways are 6!*3!
So no. of possible ways when AEI are never together will be 8! - 6!*3!
Now i told you that there are many more cases but that are really not important I am explaining these cases
because they are important and help ypu while solving Probability.
Now We should move on to the next Topic That Is Combination. Now you know that Permutation means
Arrangement or no. of possible ways A thing can be arranged.
What is the meaning of Combination.
Combination is a simple act of Choosing or Selection.
Like When it is asked What are no. of possible ways Word TITAN can be arrange You have to find The
Permutation.
But if it is asked what are no. of possible ways You can Select 2 alphabet from the word TITAN, It means you have
to find Combination.
The act of selection or Choosing is called COMBINATION.
Now you all must know what is nPr so it's time to move towards nCr
Like nPr = n!/(n-r)!
nCr is somewhat simillar but that is just an extra r! in the denominator
So nCr = n!/[(n-r)!*r!]
nCr means r things has to be selected out of n things.
Like in the above Question No. of possible ways 2 alphabets can be selected from the word TITAN
So total no. of alphabets n = 5
no. of alphabets which we have to select r = 2
So the answer will be 5C2 = 5!/(5-2)!*2! = 5!/3!*2! = 5*4/2*1 = 10
Now i told you have to calculating nPr in a simple way Just like that we can also calculate nCr in a simple way All
you have to do is Follow the method of nPr and In division you have to also multiply in increasing order from 1
Like 6C3 = 6*5*4/1*2*3
And 9C2 = 9*8/1*2
and 10C4 = 10*9*8*7/1*2*3*4
7C5 = 7*6*5*4*3/1*2*3*4*5
This much knowledge of combination is enough for solving the Questions of Probability.
So without wasting Time just move on to our main Topic ie Probability.
Probability
So what is Probability ?
Probability is Just the chances have happening of an event. Like what are the chances that You will Become a PO
or An Income Tax Inspector or a Clerk. What are the chances that you will find the love of you life (That chance of
that is very rare)
These all chances are just the game of Probability. Our Life is Also The sum of all these chances, the chances we
take Like What are the chances that you will study after 12 instead of gossiping on whatsapp.
So how de we find the probability of happening of an event. In mathematical terms probability = Number of
favourable Outcomes/ Total outcomes
No. of favourable outcomes means the outcomes which we want.
Total outcomes Means the total possible outcomes (That's the reason we studied Permutation and Combination so
that we can find total outcomes]
Let me give you a very realistic example. What is the probability that You will Become a PO in SBI ?
So We have to find the favourable outcomes here That will be the No. of Posts in SBI[ because if you get any of
the post in the total post you will be a PO]
So total no. of Posts In SBI this time is 2000
And what are the total outcomes or What are the total no. of Applicants = 20,00,000

So what is the probability that You will be 1 of them Simple Probability of You getting selected = favourable
Outcomes/ total outcomes = 2,000/20,00,000 = 1/1000
That is Your Chances. Or in other words 1 in a thousand Aspirant can become a PO in SBI.
So i think Now you have the basic Idea what is PROBABILITY.
So now Lets Move On to Questions.
But before that.VERY VERY VERY VERY VERY IMPORTANT
AND = Multiplication(*)
OR = Addition (+)
If anywhere and I mean Anywhere you see a question which say what is the probabilty of getting X or Y, It simply
means that you have to find probabilty of X and Probability of Y and ADD them, The word OR means Addition
Always Keep in Mind that.
And if It is asked what is the prbability of getting X and Y, It simply means that you have to find the probability of X
and Y and Multiply them, The word AND means Multiplication Always remember that.
At least = Minimum We require [ Or kam se kam Kitna hone chahiye Usase jyada bhi ho sakta hai but usase kam
nahi hona chahiye]
Example If we want at least 2that measn Minimum we need 2 We can have 3 or 4 or 5 It doesn't matter but
Should not be less than 2.
At Most = Maximum We Require[ Jyada se Jyada Kitna ho sakta hai, Usase Kam ho sakta hai farak nahi padta but
usase jyada ahi hona chahiye]
Example if we want AT MOST 2 That means we can have 2 we can have 1 and we can have 0 also any less value
it doesn't But we can can't have anything greater than 2.
These cases will be more clear to you when we will solve some Questions.
Questions related to Balls.
Case 1: Normal Case
There are Total 5Red, 3Blue and 2 Green bals In a Bag, Two balls are taken out at random What is the
probability that
i)-2 Balls will be Green.
ii- 2balls will be RED
iii) - 2 balls will be BLUE.
i) What is the probability that 2 balls are taken out at random from a bag and both balls are Green.
So Calculate First the favourable Outcomes. That is how many ways 2 balls can be taken out from a bag which
have 2Green balls = 2C2 = 1
Now calculate Total Outcomes. That is how many ways 2 balls can be taken out from the bag containing total 10
balls [ 5red + 3Blue + 2 Green = Total 10] = 10C2 = 10*9/1*2 = 45
So probability = favourable outcomes/total outcomes = 1/45
ii) What is the probability that 2 balls are taken out from bag and both are RED
So Calculate First the favourable Outcomes. That is how many ways 2 balls can be taken out from a bag which
have 5Red balls = 5C2 = 5*4/1*2 = 10
Now calculate Total Outcomes. That is how many ways 2 balls can be taken out from the bag containing total 10
balls [ 5red + 3Blue + 2 Green = Total 10] = 10C2 = 10*9/1*2 = 45
So probability = favourable outcomes/total outcomes = 10/45 = 2/9
iii) What is the probability that 2 balls are taken out from bag and both are Blue.
So Calculate First the favourable Outcomes. That is how many ways 2 balls can be taken out from a bag which
have 3BLUE balls = 3C2 = 3*2/1*2 = 3
Now calculate Total Outcomes. That is how many ways 2 balls can be taken out from the bag containing total 10
balls [ 5red + 3Blue + 2 Green = Total 10] = 10C2 = 10*9/1*2 = 45
So probability = favourable outcomes/total outcomes = 3/45 = 1/15.
CASE 2 - AND Case .
There are Total 5Red, 3Blue and 2 Green bals In a Bag, Three balls are taken out at random What is the
probability that
i) 2 balls are Red and 1 ball is Green
ii) 2balls are Blue and 1 ball is Green
i) What is the probabilty that 3 balls are taken out and out of those 3 balls 2 balls are red and 1 is green.
In this questions there are 2 events i.e getting 2 red ball and getting 1 green ball

So first we have to calulate the seperate probabilities first.


So no. of ways 2 Red balls can be selected out of total 5 balls = 5C2 = 5*4/1*2 = 10
So no. of ways 1 Green ball can be selected out of total 2 balls = 2C1 = 2
So Favourable Outcomes i.e No. of ways 2 Red balls AND 1 Green Ball can Be Selected = 10*2 = 20[ Whenver
you see and Just Multiply it ]
And Total No. of Outcomes i.e Selecting 3 balls out of total 10 balls = 10C3 = 10*9*8/1*2*3 = 120
So probability of getting 2 Red and 1 green Balls = favourable outcomes/total outcomes = 20/120 = 1/6
ii) What is the probabilty that 3 balls are taken out and out of those 3 balls 2 balls are BLUE and 1 is
GREEN
So no. of ways 2 BLUE balls can be selected out of total 3 balls = 3C2 = 3*2/1*2 = 3
So no. of ways 1 Green ball can be selected out of total 2 balls = 2C1 = 2
So Favourable Outcomes i.e No. of ways 2 BLUE balls AND 1 Green Ball can Be Selected = 3*2 = 6[ Whenver
you see and Just Multiply it ]
And Total No. of Outcomes i.e Selecting 3 balls out of total 10 balls = 10C3 = 10*9*8/1*2*3 = 120
So probability of getting 2 BLUE and 1 green Balls =favourable outcomes/total outcomes = 6/120 = 1/20
Case 3: OR CASE
There are Total 5Red, 3Blue and 2 Green bals In a Bag, 2 balls are taken out at random What is the probability
that 2 balls are Red or 2 balls are Blue
In this questions there are 2 events i.e getting 2 red ball or getting 2 Blue balls
So first we have to calulate the seperate probabilities first.
So no. of ways 2 Red balls can be selected out of total 5 balls = 5C2 = 5*4/1*2 = 10
So no. of ways 2 BLUE balls can be selected out of total 3 balls = 3C2 = 3*2/1*2 = 3
So Favourable Outcomes i.e No. of ways 2 RED balls OR 2 BLUE Balls can Be Selected = 10 + 3 = 13[ Whenver
you see OR Just ADD it ]
Now calculate Total Outcomes. That is how many ways 2 balls can be taken out from the bag containing total 10
balls [ 5red + 3Blue + 2 Green = Total 10] = 10C2 = 10*9/1*2 = 45
So the probability of getting 2 Red ball or 2 Blue balls = favourable outcomes/total outcomes = 13/45
CASE 4 - AT LEAST CASE
There are Total 5Red, 3Blue and 2 Green bals In a Bag, Three balls are taken out at random What is the
probability that At least 2 Balls are RED.
Now What i Told you In At Least Case You have to select at least 2 means You can have all 3 balls red But at least
2 balls should be RED means we will have to find the probability of getting 2 red balls OR 3 red balls.
So there are 2 cases here 1st case is when we get 2 red balls and 1 ball can be of any other colour
and 2nd case is when we get all 3 balls as red.
1st case
So no. of ways 2 Red balls can be selected out of total 5 balls = 5C2 = 5*4/1*2 = 10
And No. ways 1 ball can be selected out of rest 5 balls = 5C1 = 5
Our Favourable outcomes I.e getting 2 red balls and 1 ball of any colour = 5*10 = 50 [ And case so multiply ]
And Total No. of Outcomes i.e Selecting 3 balls out of total 10 balls = 10C3 = 10*9*8/1*2*3 = 120
So Probability of getting 2 red balls and 1 ball of any other colour = favourable outcomes/total outcomes = 50/120
= 5/12
2nd case When we get all 3 balls as red.
So no of ways 3 red balls can be selected out of total 5 red balls i.e also our favourable outcome = 5C3 =
5*4*3/1*2*3 = 10
And Total No. of Outcomes i.e Selecting 3 balls out of total 10 balls = 10C3 = 10*9*8/1*2*3 = 120
So probability of getting 3 red balls =favourable outcomes/total outcomes = 10/120 = 1/12
Now Either Case One will happen OR Case 2 will happen. that means either we will get 2red balls and 1 other ball
or we will get all 3 red balls So As i already explained that In OR case Probabilities gets added so we will just add
the probability To get the final probability.
So when 3 balls are taken out at random the probability that at least 2 balls are green = 1/12 + 5/12 = 6/12 = 1/2
Case 5 At MOST CASE
There are Total 5Red, 3Blue and 2 Green bals In a Bag, Three balls are taken out at random What is the
probability that At Most 2 Ball is RED.
So as i told you all in case of AT most We can have any number less than But not greater than That means We

can Have 2 Red balls out of 3 balls and We also can 1 red ball out of 3 balls and we can also have 0 red balls but
we can't have More than 2 Red ball. That means all 3 balls can't be RED.
So we will solve same like the last case.
No. Of ways 2 red balls and 1 others balls can be selected = 5C2*5C1 = 10*5 = 50
No. Of ways 1 red balls and 2 others balls can be selected = 5C1*5C2 = 5*10 = 50
Now Of ways Balls are selected that there are NO red balls That means All three balls are of Other Colours = 5C3
= 10
Total No. Of Outcomes = 10C3 = 10*9*8/1*2*3 = 120
So Probability will be (50+10 +50)/120 = 110/120 = 11/12
Quiz :Time:- (4-5 minutes)
1. A bag has six red marbles and six blue marbles. If two marbles are drawn randomly from the bag, what
is the probability that they will both be red?
A) 1/2
B) 11/12
C) 5/12
D) 5/22
E)1/3
2. There are five students in a study group: two finance majors and three accounting majors. If two
students are chosen at random, what is the probability that they are both accounting students?
A) 3/10
B) 2/5
C) 1/5
D) 3/5
E) 4/5
3. At a certain business school, 400 students are members of the sailing club, the wine club, or both. If
200 students are members of the wine club and 50 students are members of both clubs, what is the
probability that a student chosen at random is a member of the sailing club?
A) 1/2
B) 5/8
C) 1/4
D) 3/8
E) 3/5
4. A bag contains 3 red marbles, 3 blue marbles, and 3 green marbles. If a marble is randomly drawn from
the bag and a fair, six-sided dice is tossed, what is the probability of obtaining a red marble and getting 6
from dice?
A. 1/15
B. 1/6
C. 1/3
D. 1/4
E. 1/18
5. A letter is randomly select from the word "STUDIOUS". What is the probability that the letter be a U?
A. 1/8
B. 1/4
C. 1/3
D. 1/2
E. 3/8
6. In how many different ways can the letters of the word 'MATHEMATICS' be arranged so that the vowels
always come together?
A.124045
B.20890
C. 133156
D. 120960
E. None of these

7. How many 4-letter words with can be formed out of the letters of the word, 'LOGARITHMS', if repetition
of letters is not allowed?
A. 400
B. 4050
C. 5040
D. 5773
E. None of these
8. In a group of 6 boys and 4 girls, four children are to be selected. In how many different ways can they
be selected such that at least one boy should be there?
A. 156
B. 209
C. 193
D. 245
E. None of these
9. In a bag, there are 8 red, 7 blue and 6 green balls. One ball is picked up randomly. What is the
probability that it is neither red nor green?
A. 3/91
B. 1/3
C. 3/7
D. 7/15
E. None of these
10. One card is drawn at random from a pack of 52 cards. What is the probability that the card drawn is a
face card (Jack, Queen and King only)?
A. 3/13
B. 1/13
C. 7/52
D. 9/13
E. None of these
Thank you Insomniac
Answers:1.D
Probability that both are red marbles = 6/12 x 5/11 = 5/22
2. A
Probability of first student to be accounting student =3/5
Probability of second student to be accounting student =2/4 = 1/2
Probability that both students to be accounting students =3/5 x 1/2 = 3/10
3.B
Members in sailing club = 250
Probability of choosing member from sailing club = 250/400 = 5/8
4.E
Probability getting red marble = 3/9 = 1/3
Probability of getting 6 = 1/6
Probability of getting red marble and 6 = 1/3 x 1/6 = 1/18
5. B
Probability of choosing u - 2/8 = 1/4
6.D
No. of ways = {8! /(2! * 2!)}{4!/2!}= 10080 *12 =120960
7. C
Required no. of words = 10p4 = 10*9*8*7 = 5040
8.B
For at least one boy required no. of way =(6C1*4C3)+(6C2*4C2)+(6C3*4C1)+(6C4) =209
9.B
Total no. of balls = 8+7+6 = 21
Probability to chose neither red nor green ball = 7/21= 1/3

10. A
Required Probability = 12/52 =3/13
To view the above quiz in Hindi : Click here

bankersadda.com

http://www.bankersadda.com/2015/08/banking-pathway-2015quant-quiz.html

Banking Pathway 2015:Quant Quiz


Directions (1-3): In the following number series only one number is wrong. Find out the wrong number.
1. 1 1
1) 2
2) 6
3) 30
4) 212
5) 2310

2. 1 1.4
1) 1.4
2) 1.96
3) 2.776
4) 4.1208
5) 6.0208
3.
1
1) 19
2) 43
3) 91
4) 186
5) 379

1.96

19

30

212

2.776

43

91

2310

4.1208

186

6.0208

379

4.Five alarms start to ring together and ring at the intervals of 3, 4, 6, 11 and 12 seconds respectively. How
many times will they ring together in one hour, excluding the one at the start?
1) 24 times
2) 25 times
3) 26 times
4) 27 times
5) 28 times
5.What is the number which, when added to the terms of the ratio 17 : 29 makes the new ratio
7 : 11?
1) 3
2) 4
3) 5
4) 6
5) 7
6.A person sells one house for Rs 75 lakhs, making profits of 25%. He sells another house at a loss of
20%, and on the whole he makes neither profit nor loss. What is the cost of the second house?
1) 50 lakhs
2) 60 lakhs
3) 75 lakhs
4) 90 lakhs
5) None of these
7.Ravi and Raj invested in the ratio of 4 : 3 in a business. If 16% of the total profit goes to charity and
Ravis share of profit is Rs 816, what is the amount of total profit?
1) Rs 1200

2) Rs 1400
3) Rs 1500
4) Rs 1700
5) None of these
8.The average of the first and second of three numbers is 12 more than the average of the second and the
third of these numbers. What is the difference between the first and the third of these three numbers?
1) 6
2) 12
3) 24
4) 36
5) 18
9.A person completes journey by using three different types of vehicles. He completes first part with
speed of 10 kmph, second part at the speed of 12 kmph and third part at the speed of 15 kmph. If the
distance travelled by each vehicle is equal then what is the average speed of that person throughout the
journey?
1) 12 kmph
2) 12.5 kmph
3) 13 kmph
4) 13.5 kmph
5) None of these
10.If the ratio of the ages of A and B at present is 7 : 4. After 12 years the ratio of their ages will be 10 : 7,
what is the present age of B?
1) 12 years
2) 14 years
3) 16 years
4) 18 years
5) 20 years
Answers with Explanation !!!!!!!

bankersadda.com

http://www.bankersadda.com/2015/08/banking-pathway-2015quant-quiz.html

Banking Pathway 2015:Quant Quiz


Directions (1-3): In the following number series only one number is wrong. Find out the wrong number.
1. 1 1
1) 2
2) 6
3) 30
4) 212
5) 2310

2. 1 1.4
1) 1.4
2) 1.96
3) 2.776
4) 4.1208
5) 6.0208
3.
1
1) 19
2) 43
3) 91
4) 186
5) 379

1.96

19

30

212

2.776

43

91

2310

4.1208

186

6.0208

379

4.Five alarms start to ring together and ring at the intervals of 3, 4, 6, 11 and 12 seconds respectively. How
many times will they ring together in one hour, excluding the one at the start?
1) 24 times
2) 25 times
3) 26 times
4) 27 times
5) 28 times
5.What is the number which, when added to the terms of the ratio 17 : 29 makes the new ratio
7 : 11?
1) 3
2) 4
3) 5
4) 6
5) 7
6.A person sells one house for Rs 75 lakhs, making profits of 25%. He sells another house at a loss of
20%, and on the whole he makes neither profit nor loss. What is the cost of the second house?
1) 50 lakhs
2) 60 lakhs
3) 75 lakhs
4) 90 lakhs
5) None of these
7.Ravi and Raj invested in the ratio of 4 : 3 in a business. If 16% of the total profit goes to charity and
Ravis share of profit is Rs 816, what is the amount of total profit?
1) Rs 1200

2) Rs 1400
3) Rs 1500
4) Rs 1700
5) None of these
8.The average of the first and second of three numbers is 12 more than the average of the second and the
third of these numbers. What is the difference between the first and the third of these three numbers?
1) 6
2) 12
3) 24
4) 36
5) 18
9.A person completes journey by using three different types of vehicles. He completes first part with
speed of 10 kmph, second part at the speed of 12 kmph and third part at the speed of 15 kmph. If the
distance travelled by each vehicle is equal then what is the average speed of that person throughout the
journey?
1) 12 kmph
2) 12.5 kmph
3) 13 kmph
4) 13.5 kmph
5) None of these
10.If the ratio of the ages of A and B at present is 7 : 4. After 12 years the ratio of their ages will be 10 : 7,
what is the present age of B?
1) 12 years
2) 14 years
3) 16 years
4) 18 years
5) 20 years
Answers with Explanation !!!!!!!

bankersadda.com

http://www.bankersadda.com/2015/07/banking-pathway-quant-quiz.html

Banking Pathway: Quant Quiz


Directions (Q. 1-5): Each question below is followed by two statements A and B. You have to decide
whether the data given in the statements are sufficient for answering the questions. Read both the
statement and give answer :
1) If the statement A alone is sufficient to answer the question but the statement B alone is not sufficient.
2) If the statement B alone is sufficient to answer the question but the statement A alone is not sufficient.
3) If both statements A and B together are needed to answer the question.
4) If either the statement A alone or statement B alone is sufficient to answer the question.
5) If the data even in both statements A and B together are not sufficient to answer the question.

1. What is the profit earned by selling a pen for Rs. 120 ?


A) The cost price of 5 such pens is equal to selling price of 4 such pens.
B) 25% profit is earned by selling each pen.
2. What is the salary of C, in a group of A, B, C, D and E, whose average salary is Rs. 12000 ?
A) C's salary is 2.5 times B's salary.
B) Average salary of A and B is Rs. 4000.
3. What is the area of the circle ?
A) The breadth of a rectangle is 3/4th radius of circle.
B) The radius of the circle is equal to the side of a square of area 169 sq m.
4. What is the three digit number ?
A) 3/5 of a number is less by 90 of that number.
B) 1/4 of a number is 25% of that number.
5. What is the speed of the car ?
A) The car covers a distance 120 km in 4 hours.
B) The car covers a distance 210 km in 7 hours.
6. If 6x^2 + 5x + 1 < 0 then the value of x satisfies which inequality are ?
1) -1/2 < x < -1/3
2) x < -1/2 and x > -1/3
3) x -1/2
4) x 1/2
5) None of these
Directions (Q. 7-8): In each of these questions, two equations I and II with variable x and y are given.
You have to solve both the equations to find the value x and y and give answer:
1) If x < y
2) If x y
3) If x = y
4) If x y
5) If x > y
7.

I. x^2 = 9
II. y^2 + 8y + 16 = 0
8.
I. 4x^2 + 10x 6 = 0
II. y^2 5y + 6 = 0
9. The radius of a copper sphere is 6 cm. The sphere is melted and drawn into a long wire of uniform
circular cross section. If the length of the wire is 72 cm, then the radius of wire is ?
1) 4 cm
2) 4.2 cm
3) 2 cm
4) 3 cm
5) None of these
10. The ratio of length of rhombus is 3 : 4. Then, the ratio of the area of the rhombus to the square of the
shorter diagonal is?
1) 3 : 2
2) 3 : 8
3) 1 : 2
4) 4 : 3
5) None of these

ANSWERS:
1. 4
Using statements A
Profit % = (5 - 4)/4 * 100 = 25%
Cost price = 125 * 100/125 = Rs. 96
Profit = 120 96 = Rs. 24
Similarly, we can find profit by using statement B.
2. 5
From statement A salary of C = 2.5 B
From statement B, A + B = 2 4000 = Rs. 8000
3. 2
From statement B, radius of circle = sq. root (169) m = 13m
Required area = r^2
= 22/7 * (13)^2
4. 1
From statement A, 3/5 * x = x - 90
2/5x = 90
From B, x/4 = x/4 we can't determine the value of x
5. 4
Since, speed of car = Distance covered by it/ Time taken by it
6. 1
5x^2 + 6x + 1 < 0

6x^2 + 3x + 2x + 1 < 0
(2x + 1) (3x + 1) < 0
(x + 1/2) (x + 1/3) <0
- 1/2 < x < -1/3
7. 5
From equation I:
x^2 = 9
x = +/-3
From II:
y^2 + 8y + (4)^2 = 0
(y + 4)^2 = 0
y=-4
x>y
8. 1
I: 4x^2 + 12x - 2x - 6 = 0
4x (x + 3) - 2 (x + 3) = 0
(4x - 2) ( x + 3) = 0
x = -3 or 1/2

II:
y^2 - 5y + 6 = 0
(y -3)(y - 2) = 0
y = 3 or 2
y>x
9. 3
___r^2 * 72 = 4/3 * (6)^3
r^2 = 4
r = 2 cm
10. 5
Area of rhombus = 1/2 * 3x * 4x = 6x^2
Square of the shorter diagonal = 3x^2 = 9x^2
Req. Ratio = 6x^2/9x^2 = 2/3

bankersadda.com

http://www.bankersadda.com/2015/08/banking-pathway-quant-quiz.html

Banking Pathway: Quant Quiz


Directions : In each of these questions, two equations numbered I and II with variables x and y are given.
You have to solve both the equations to find the value of x and y. Give answer
1) if x > y
2) if x >= y
3) if x < y
4) if x <= y
5) if x = y or relationship between x and y cannot be determined.
1.
I. x^2 26x + 169 = 0
II. y^2 28y + 196 = 0

2.
I. 2x^2 + 6x 36 = 0
II. 2y^2 + 25y + 78 = 0
3.
I. 7x^2 + 104x + 169 = 0
II. y^2 + 10y 39 = 0
4.
I. x^2 6x 16 = 0
II. 3y^2 + 22y + 24 = 0
Directions: Each question below is followed by two statements A and B. You have to determine whether
the data given in the statement is sufficient to answer the question. You should use the data and your
knowledge of mathematics to choose between the possible answers. Give answer:
1) if the statement A alone is sufficient to answer the question, but the statement B alone is not sufficient.
2) if the statement B alone is sufficient to answer the question, but the statement A alone is not sufficient.
3) if the both statement A and B together are needed to answer the question.
4) if either the statement A alone or B alone is sufficient to answer the question.
5) if you cannot get the answer from the statements A and B together, but needed even more data.
5. What is the principal?
A. The simple interest earned for 8 year at the rate of 10% p.a is Rs. 960.
B. The simple interest is Rs. 480 for 4 year.
6. The sum of the ages of P, Q, R and S is 86 year, what is Q's present age?
A. The average age of P, Q and S is 24 year.
B. The average age of R and S is 28 year.
7. In how many days, can Q alone complete the work?
A. P does 3/5 of a work in 21 days.
B. Q does 3/4 of a work in 18 days.
8. Find the number of ways in which 10 players out of 14 players can be selected such that 3 particular

player are always included and 2 particular players are always excluded?
1) 38
2) 36
3) 48
4) 40
5) 32
9. The ratio of the present age of a mother and daughter is 7 : 3. Four years ago the ratio of their ages was
5 : 2. What will be the mother's age five years from now? (in years)
1) 89
2) 86
3) 87
4) 84
5) 88
10. The sum of nine consecutive odd number of set A is 657. What is the sum of seven consecutive odd
numbers whose lowest number is 18 more than the lowest number of set A?
1) 620
2) 625
3) 635
4) 615
5) 623
1. 3;
I. x^2 26x + 169 = 0
(x 13)^2 = 0
x = 13
II. y^2 28y + 196 = 0
(y 14)^2 = 0
y = 14
x<y
2. 2;
I. 2x^2 + 12x 6x 36 = 0
2x (x + 6) 6 (x + 6) = 0
(x + 6) (2x 6) = 0
x = 6, 3
II. 2y^2 + 12y + 13y + 78 = 0
2y (y + 6) + 13 (y + 6) = 0
(y + 6 ) (2y + 13) = 0
y = 6, -13/2
x>y
3. 5;
I. 7x^2 + 13x + 91x + 169 = 0
(x + 13) (7x + 13) = 0
x = 13, -13/7
II. y^2 + 13y 3y 39 = 0
(y 3) (y + 13) = 0
y = 3, 13
Relationship between x and y cannot be established.

4. 5;
I. x^2 8x + 2x 16 = 0
(x + 2) (x 8) = 0
x = 2, 8
II. 3y^2 + 22y + 24 = 0
3y 2 + 4y + 18y + 24 = 0
(y + 6) (3y + 4) = 0
y = 6, -4/3
Relationship between x and y cannot be established.
5. 1;
From A, we get,
P = 100*SI/R*T = Rs. 1200
5. 5;
P + Q + R + S = 86
From A, P + Q + S = 24 3 = 72 R = 86 72 = 14
From B, R + S = 28 2 = 56
We cannot get Qs present age from both statements.
6. 2;
From statement B, work done by Q = 18 4/3 = 24 days.
7. 2;
Out of 14 players, let us element 2 particular player
which are excluded. Now, there are 12 players for selection of these 12, three have to be included in team always.
Thus remaining players are (12 3) = 9 and the required players for team (10 3) = 7. Now,
selection cannot be done in
9C7 ways.
9C7 = 9!/7!2! = 36 ways.
8. 1;
Let the present age of mother and daughter be 7x and 3x year.
7x 4/3x 4 = 5/2
14x 8 = 15x 20
x = 12
Mothers age five years from now = 12 7 + 5
= 84 + 5 = 89 year
9. 5;
In Set A, 9x + 2(1 + 2 + ..... + 8) = 657
9x + 8*9*2/2 = = 657
9x + 36 2 = 657
x = 65
lowest number of another set = 65 + 18 = 83
Sum of seven consecutive number of another set
= 7x + (1 + 2 + 3 + 4 + 5 + 6) 2
= 7x + 6*7*2/2
= 7x + 21 = 83 7 + 42 = 623

10. 4;
First arrange the 7 boys in a row in 7P7 = 7! ways.
Then, there are 8 gaps between them. Now 5 girls can be
arranged in 8 gaps in P5
8 ways.
Required number of ways of arrangement = 7! 8P5 = 7 ! * 8 !/5!

= 8! 7 6 = 42 8 !

bankersadda.com

http://www.bankersadda.com/2015/09/ibps-mission-exam-quant-quiz.html

IBPS Mission Exam :Quant Quiz


1.Neeraj buys a plot of land for Rs. 96 ,000. She sells 2/5 of it at a loss of 6%. She wants to make a profit
of 10% on the whole transaction by selling the remaining land. The gain % on the remaining land is
(1) 20
(2) 202/3
(3) 14
(4) 7
(5) none of these
2.An article is sold at a gain of 15%. Had it been sold for Rs. 27 more, the profit would have been 20%.
The cost price of the article is
(1) Rs. 500
(2) Rs. 700
(3) Rs. 540
(4) Rs. 545
(5) none of these
3.On selling 17 balls at Rs. 720, there is a loss equal to the cost price of 5 balls. The cost price (in Rs.) of
a ball is
(1) 45
(2) 50
(3) 55
(4) 60
(5) none of these
4.Meena purchased 30kg of rice at the rate of Rs. 10 per kg and 35 kg at the rate of Rs. 11 per kg. He
mixed the two. At what price per kg (in Rs.) should he sell the mixture to make a 30% profit in the
transaction?
(1) 12.5
(2) 13
(3) 13.7
(4) 14.25
(5) none of these
5.A shopkeeper purchased a chair marked at Rs 600 at two successive discount of 15 % and 20 %
respectively .He spent Rs 28 on transportation and sold the chair for Rs 545,his gain %
(1) 25
(2) 30
(3) 35
(4) 20
(5) none of these
6.Two items A and B are sold at a profit of 10% and 15% respectively. If the amount of profit received is
the same, then the cost price A and B may be
(1) Rs.1,000, Rs. 1,500
(2) Rs.5,000, Rs. 2,000
(3) Rs.3,000 ,Rs. 2,000
(4) Rs.3,000 ,Rs. 5,000
(5) none of these

7.In an examination A got 25% marks more than B, B got 10% less than C and C got 25% more than D. If
D got 320 marks out of 500, the marks obtained by A were
(1) 405
(2) 450
(3) 360
(4) 400
(5) none of these
8.Three sets of 40,50 and 60 students appeared for an examination and the pass percentage was 100, 90
and 80 respectively. The pass percentage of the whole set is
(1) 882/3
(2) 842/3
(3) 88 1/3
(4) 841/3
(5) none of these
9.A certain distance is covered by a cyclist at a certain speed. If a jogger covers half the distance in
double the time, the ratio of the speed of the jogger to that of the cyclist is
(1) 1 : 4
(2) 4 : 1
(3) 1 : 2
(4) 2 : 1
(5) none of these
10.The distance between places A and B is 999 km. An express train leaves place A at 9 am and runs at a
speed of 55.5 km/hr. The train stops on the way for 1 hour 20 minutes. It reaches B at
(1) 4:20 am
(2) 12 pm
(3) 6 pm
(4) 11 pm
(5) none of these
Answers with Explanation !!!!!

bankersadda.com

http://www.bankersadda.com/2015/09/mission-ibps-quant-quiz_7.html

Mission IBPS : Quant Quiz


1.The bus fare and train fare of a place from Kolkata were Rs. 20 and Rs. 30 respectively. Train fare has
been increased by 20% and the bus fare has been increased by 10%. The ratio of new train fare to new
bus fare is
(1) 11 : 18
(2) 18 : 11
(3) 5 : 3
(4) 3 : 5
(5) None of these

2.A money-lender borrows money at 4% per annum and pays the interest at the end of the year. He lends
it at 6% per annum compound interest compounded half yearly and receive the interest at the end of the
year. In this way, he gains Rs. 104.50 a year. The amount of money he borrows, is
(1) Rs. 6,000
(2) Rs, 5,500
(3) Rs. 5,000
(4) Rs. 4,500
(5) None of these
3.Simple interest on Rs. 500 for 4 years at 6.25% per annum is equal to the simple interest on Rs. 400 at
5% per annum for a certain period of time. The period of time is
(1) 4 years
(2) 5 years
(3) 25/4 years
(4) 26/3 years
(5) None of these
4.With a given rate of simple inerest, the ratio of principal and amount for a certain period of time is 4 : 5
After 3 years, with the same rate of interest, the ratio of the principal and amount becomes 5 : 7. The rate
of interest is
(1) 4%
(2) 6%
(3) 5%
(4) 7%
(5) None of these
5.Rs. 1,000 is invested at 5% per annum simple interest. If the interest is added to the principal after every
10 years, the amount will become Rs. 2,000 after
(1) 15 years
(2) 18 years
(3) 20 years
(4) 162/3 years
(5) None of these
6.A sum of money amounts to Rs. 5,200 in 5 years and to Rs. 5,680 in 7 years at simple interest. The rate
of interest per annum is
(1) 3%
(2) 4%

(3) 5%
(4) 6%
(5) None of these
7.A borrows Rs. 800 at the rate of 12% per annum simple interest and B borrows Rs. 910 at the rate of
10% per annum, simple interest. In how many years will their amounts of debt be equal?
(1) 18
(2) 20
(3) 22
(4) 24
(5) None of these
8.A person deposited Rs. 400 for 2 years ,Rs 550 for 4 years and Rs. 1,200 for 6 years. He received the
total simple interest of Rs. 1,020. The rate of interest per annum is
(1) 10%
(2) 5%
(3) 15%
(4) 20%
(5) None of these
9.The marked price of a shirt and trousers are in the ratio 1 : 2. The shopkeeper gives 40% discount on
the shirt. If the total discount on the set of the shirt and trousers is 30%, the discount, offered on the
trousers is
(1) 15%
(2) 20%
(3) 25%
(4) 30%
(5) None of these
10.A dealer buys an article marked at Rs. 25,000 with 20% and 5% off. He spends Rs, 1,000 for its repairs
and sells it for Rs. 25,000. What is his gain or loss per cent?
(1) Loss of 25%
(2) Gain of 25%
(3) Gain of 10 %
(4) Loss of 10%
(5) None of these
Answers with Explanation !!!!!!!!!!!!

bankersadda.com

http://www.bankersadda.com/2015/09/mission-ibps-exam-di-quiz.html

Mission IBPS Exam : DI Quiz


Directions : (Q. 1-5) Study the following graph and table carefully and answer the given
questions Percentage of Males, Females and Children Living in Various Colonies

Total number of residents in various colonies


A

1250

2050

1800

1150

1620

1.What is the total number of females in Colonies A, B and C together?


(1) 1785
(2) 1821
(3) 1479
(4) 1692
(5) None of these
2.The number of children in Colony A are approximately what per cent of the number of children in
Colony E?
(1) 121
(2) 116
(3) 75
(4) 101
(5) 98
3.What is the respective ratio of the number of males in Colony B to the number of females in the same

colony?
(1) 3 : 5
(2) 7 : 5
(3) 8 : 7
(4) 5 : 7
(5) None of these
4.What is the average number of residents from all the colonies together?
(1) 1654
(2) 1600
(3) 1580
(4) 1574
(5) None of these
5.What is the difference between the number of males in Colony D and the number of children in the
same colony?
(1) 138
(2) 126
(3) 136
(4) 135
(5) None of these
Directions : (6-10) Study the pie-chart and table carefully to answer the questions that follow
Number of Employees Working in Various Departments of an Organization and the Ratio of Men to
Women in the Same.Total Number of Employees = 4600

Ratio Of Men and Women

Departments

Men

Women

HR

Accounts

Production

IT

Marketing

Merchandising

6.What is the number of women in the Accounts department?


(1) 86
(2) 102
(3) 80
(4) 92
(5) None of these
7.What is the total number of employees working in the IT department and HR department together?
(1) 1628
(2) 1742
(3) 1766
(4) 1646
(5) None of these
8.What is the respective ratio of total number of men to the total number of women working in all the
departments together?
(1) 63 : 41
(2) 41 : 27
(3) 53 : 47
(4) 27 : 19
(5) None of these
9.Number of women in the Merchandising department form what per cent of the total number of
employees in the organization?
(1) 3
(2) 6
(3) 1
(4) 12
(5) None of these
10.What is the respective ratio of number of men in the Production department to the number of men in
the Marketing department?
(1) 7 : 13
(2) 9 : 11
(3) 13 : 7
(4) 11 : 9
(5) None of these

Answers with Explanation!!!!!!!!!!!!!!

To view the above post in hindi Click Here

bankersadda.com

http://www.bankersadda.com/2015/09/mission-ibps-exam-di-quiz_15.html

Mission IBPS Exam : DI Quiz


Directions : (Q. 1-5) Study the graph carefully to answer the questions that follow
Income (in lakhs) of three Companies over the Years

1.If the per cent profit of Company A in the year 2002 was 20, what was its expenditure in that year?
(1) Rs. 250000
(2) Rs. 275000
(3) Rs. 175000
(4) Rs. 150000
(5) None of these
2.If the expenditure of Company C in 2003 was Rs. 1.75 lakhs, what was its per cent profit in that year?
(rounded off to two digits after decimal)
(1) 38.29
(2) 42.86
(3) 53.41
(4) 58.64
(5) None of these
3.What is the average income of Company A over the years?
(1) Rs. 275000
(2) Rs. 3000000
(3) Rs. 2750000
(4) Rs. 30000
(5) None of these
4.What is the approximate per cent increase in income of Company B in the year 2006 from the previous
year?
(1) 28
(2) 11
(3) 17
(4) 8
(5) 22

5.Per cent increase/decrease in income of Company C was highest for which year?
(1) 2004
(2) 2006
(3) 2003
(4) 2002
(5) None of these
Directions : (Q. 6-10) Study the following graph carefully to answer the questions that follow
Number of Students (in thousands) in Three Schools over the Years

6.What was the average number of students in all the schools together in the year 2006?
(1) 30000
(2) 9000
(3) 3000
(4) 6000
(5) None of these
7.How many times the total number of students in all the three Schools A, B and C together was exactly
equal among the given years?
(1) 2
(2) 5
(3) 4
(4) 3
(5) None of these
8.Total number of students in School B and School C together in the year 2004 was approximately what
percentage of the total number of students in School B and School C together in the year 2007?
(1) 85
(2) 80
(3) 75
(4) 184
(5) 131
9.What was the difference between the total number of students in all the schools together in the year
2003 and number of students in School B in the year 2005?
(1) 2000
(2) 3000
(3) 3500

(4) 2500
(5) None of these
10.What was the approximate average number of students in School A over all the years together?
(1) 1990
(2) 2090
(3) 2300
(4) 1800
(5) 2700
Answers with Explanation!!!!!!

bankersadda.com

http://www.bankersadda.com/2015/09/ibps-mission-quant-quiz.html

Mission IBPS Exam : Quant Quiz


1.A man sells his table clock for Rs. 144. If his percentage of profit is equal to his cost price, then the cost
price of the watch is:
(a) Rs. 90
(b) Rs. 85
(c) Rs. 80
(d) Rs. 75
(e) Rs. 64
2.An article when sold for Rs. 200 fetches 25 percent profit, What would be the percentage profit/loss if 6
such articles are sold for Rs. 1056?
(a) 10% loss
(b) 10% profit
(c) 5% loss
(d) 5% profit
(e) None
3.Ben purchased 125 stools at the rate of Rs. 120 per stool. The transport expenditure was @ Rs. 10 for 5
stool. He paid an octroi @ Rs. 2 per stool and coolie charges were Rs. 250. What should be the selling
price of each stool, if he wants profit of 10%?
(a) Rs. 125
(b) Rs. 145
(c) Rs. 150
(d) Rs. 140
(e) None
4.If the selling price is doubled, the profit triples. The profit per cent is
(a) 662/3% gain
(b) 150%
(c) 100%
(d) 120%
(e) None
5.A person purchased an article at 9/10 th of its selling price and sold it at 8% more than its selling price.
His gain per cent is
(a) 8
(b) 12
(c) 20
(d) 15
(e) 16
6.If 4/7 of a piece of work is completed in 7/4 days, in how many days can rest of the work be completed?
(a) 7/3
(b) 3/7
(c) 21/16
(d) 21
(e) 16/21
7.Mukesh, Anil and Sumit are three civil engineers. Mukesh can design a plan of a multi-storeyed

apartment in 5 hours and Anil in 4 hours alone. Three together can do it in 2 hours. In what time can
Sumit do it alone?
(a) 7 hours
(b) 15 hours
(c) 20 hours
(d) 18 hours
(e) 12 hours
8.P can do a job in 3 days, Q can do 3 times that job in 8 days and R can do 5 times that job in 12 days. If
all of them work together, they will finish one job in:
(a) 120/47 days
(b) 8/9 days
(c) 3 days
(d) 2 (1/2) days
(e) Data insufficient
9.Miti and Aditi can do a piece of work in 45 days and 40 days respectively. They began to work together
but miti leaves after x days and Aditi finished the rest of the work in (x + 14) days. After how many days
did Miti leave?
(a) 19
(b) 11
(c) 9
(d) 12
(e) 10
10.A can do a piece of work in 2 (1/2) days which B can do in 3 (1/2) days. If As wages are Rs. 50 per
week and Bs wages are Rs. 42.50 per week, what Approximate A would have charged for doing work for
which B received Rs. 340.?
(a) Rs. 300
(b) Rs. 360
(c) Rs. 286
(d) Rs. 320
(e) Rs. 420
Answers with Explanation !!!!!!!!!!!!!

bankersadda.com

http://www.bankersadda.com/2015/08/mission-ibps-exam-quant-quiz.html

Mission IBPS Exam: Quant Quiz


1. The length of a rectangle is 20% more than its breadth. What will be the ratio of the area of this
rectangle to the area of a square whose side is equal to the breadth of the rectangle?
(1) 5 : 6
(2) 6 : 5
(3) 2 : 1
(4) Data inadequate
5) None of these
2. A well with 14m inside diameter is dug 10m deep. Earth taken out around it to a width of 21 m to form
an embankment. The height (in metres) of the embankment is:
(1) 1/2
(2) 2/3
(3) 3/4
(4) 3/4
(5) None of these
3. The area of a square of side 8 cm is equal to a rectangle. Which of the following statement/s is/are
definitely true about the rectangle?
(1) The length of rectangle is 16 times its breadth
(2) The length of rectangle is 32 times its breadth
(3) the breadth of rectangle is 1/6 of its length
(4) the breadth of rectangle is 1/9 of its length
(5) None of these
4. A man sold a wristwatch for Rs. 2.400 at a loss of twenty five per cent. At what rate should he
sold the wristwatch to earn a profit of twenty five per cent?
(1) Rs. 3,600
(2) Rs. 4.000
(3) Rs. 3,500
(4) Rs. 3.800
(5) None of these

have

5. Rehaan purchased a bike for Rs 54.000. He sold it at a loss of 8 per cent. With that money he again
purchased another bike and sold it at a profit of 10 per cent. What is his overall loss/profit?
(1) Loss of 657
(2) Profit of 567
(3) Loss of 649
(4) Profit of Rs 648
(5) None of these
6. Meera purchased an item for Rs. 62,000 and sold it at loss of 25 percent. With that amount she
purchased another item and sold it at a gain of 30 percent. What was her overall gain/loss?
(1) Loss of Rs. 1560
(2) Profit of Rs. 1560
(3) Loss of Rs. 1550
(4) Profit of Rs. 1550
(5) None of these

7. In an objective examination of 90 questions, 5 marks are allotted for every correct answer and 2 marks
are deducted for every wrong answer. After attempting all the 90 questions, a student got a total of 387
marks. The number of questions attempted wrong were:
(1) 9
(2) 18
(3) 27
(4) 36
(5) None of these
8. A man has in all Rs. 640 in the denominations of one-rupee, five-rupee and ten rupee notes. The
number of each type of notes are equal. What is the total number of notes he has?
(1) 90
(2) 100
(3) 120
(4) 150
(5) None of these
9. Sumit had 85 currency notes in all, some of which were of Rs. 100 denomination and the remaining of
Rs. 50 denomination. The total amount of all these currency notes was Rs. 5000. What is the value of 50
rupee notes?
(1) Rs. 150
(2) Rs. 700
(3) Rs. 1500
(4) Rs. 3500
(5) None of these
10. The Qutab Minar casts a shadow 150m long at the same time when the Vikas Minar casts a shadow
120m long on the ground. If the height of the Vikas Minar is 80m, what is the height of the Qutab Minar?
(1) 100m
(2) 120m
(3) 150m
(4) 180m
(5) None of these
Answers..
1. 2
2. 2
3. 1
4. 2
5. 4
6. 3
7. 1
8. 3
9. 4
10. 1

bankersadda.com

http://www.bankersadda.com/2015/09/mission-ibps-exam-quant-quiz.html

Mission IBPS Exam :Quant Quiz


1.The average age of four boys, five years ago was 9 years. On including a new boy. The present average
age of all the five is 15 years. The present age of the new boy is
(1) 14 years
(2) 6 years
(3) 15 years
(4) 19 years
(5) None of these
2.If the average of 39, 48, 51, 63, 75, 83, x and 69 is 60, then the value of x is
(1) 52
(2) 53
(3) 50
(4) 51
(5) None of these
3.The cost of a piece of diamond varies with the square of its weight. A diamond of Rs. 5,184 value is cut
into 3 pieces whose weights are in the ratio 1 : 2 : 3. Find the loss involved in the cutting.
(1) Rs. 3,068
(2) Rs. 3,088
(3) Rs. 3,175
(4) Rs. 3,168
(5) None of these
4.A discount of 30% on the marked price of toy reduces its selling price by Rs. 30. What is the new selling
price (in Rs.)?
(1) 70
(2) 21
(3) 130
(4) 100
(5) None of these
5.The capacities of two hemispherical vessels are 6.4 litres and 21.6 litres. The ratio of their inner radii is
(1) 4 : 9
(2) 16 : 81
(3) 2:3
(4) 2 : 3
(5) None of these
6.Pipe A alone can fill a tank in 8 hours. Pipe B alone can fill it in 6 hours. If both the pipes are opened and
after 2 hours pipe A is closed, then the other pipe will fill the tank in
(1) 6 hours
(2) 3.5 hours
(3) 4 hours
(4) 2.5 hours
(5) None of these
7.The population of a town is 15000. If the number of males increase by 8% and that of females by 10%,
then the population would increase to 16300. Find the number of females in the town.

(1) 4000
(2) 6000
(3) 3000
(4) 5000
(5) None of these
8.If Rs. 5,000 becomes Rs. 5,700 in a years time, what will Rs. 7,000 become at the end of 5 years at the
same rate of simple interest?
(1)Rs. 10,500
(2)Rs. 11,900
(3)Rs. 12,700
(4)Rs. 7,700
(5) None of these
9.A thief is noticed by a policeman from a distance of 200 m. the thief and the policeman run at the rate of
10 km and 11 km per hour respectively. The distance (in metres) between them after 6 minutes is
(1) 190
(2) 200
(3) 100
(4) 150
(5) None of these
10.A sells an article to B at a profit of 20% and B sells it to C at a profit of 25%. If C pays Rs 1200, the
cost price of article originally (in Rs.) is
(1) 700
(2) 600
(3) 1,000
(4) 800
(5) None of these
Answers with Explanation !!!!!!!!!!!!!!

bankersadda.com

http://www.bankersadda.com/2015/09/mission-ibps-quant-quiz_4.html

Mission IBPS :Quant Quiz


Directions (Q. 1-5): What will come in place of question mark (?) in the following number series?
1. 17 45 172 ? 5088 35602
1) 712
2) 804
3) 804
4) 850
5) 904
2. 9
1) 712
2) 589
3) 596
4) 602
5) 616

333

785

929

1029

3. 1328 1722 2188 2732 3360 ?


1) 4072
2) 4075
3) 4078
4) 4081
5) 4084
4.13
1) 84
2) 91
3) 95
4) 98
5) 102

570

2846

11376

34116

5. 34 47
41 44
55
38
76
?
1) 29
2) 27
3) 25
4) 22
5) 18
Directions (Q. 6-10): In each of these questions, two equations (I) and (II) are given. You have to solve
both the equations and give answer
1) if x < y
2) if x y
3) if x = y, or no relation can be established between x and y.
4) if x > y
5) if x y
6.
I. 9x2 = 1
II. 4y2 + 11y 3 = 0

7.
I. 3x2 + 5x -2 = 0
II. 2y2 - 7y + 5 = 0

8.
I. 6x2 + 13x + 5 = 0
II.3y2 + 11y + 10 = 0

9.
I. 7x 4y = 29
II. 5x + 3y 50 = 0

10.
I. x2 5 = 0
II.4y2 - 24y + 35 = 0
Answers with Explanation !!!!!!!!

bankersadda.com

http://www.bankersadda.com/2015/09/mission-ibps-quant-quiz.html

Mission IBPS :Quant Quiz


1.The ratio of the number of students studying in Schools A, B and C is 4 : 8 : 3 respectively. If the
number of students studying in each of the schools is increased by 80%, 20% and 60% respectively, what
will be the new ratio of the number of students in Schools A, B and C.
(1)8 : 3 : 4
(2)4 : 2 : 3
(3)2 : 3 : 4
(4)Cant say
(5)None of these
2.A person covered some distance in 24 hours. He covered half the distance by rail @ 75 km per hour and
the rest by car @ 45 km/hr. The total distance covered by him was
(1)900 km
(2)1350 km
(3)675 km
(4)2700 km
(5)None of these
3.A sum of Rs. 2135 is to be divided among A, B and C in such a way that 3 times As share, 4 times Bs
share and 7 times Cs share are all equal. The share of C is
(1)Rs. 420
(2)Rs. 735
(3) Rs. 980
(4)Rs. 1200
(5)None of these
4.A person sold 160 mangoes for the C.P. of 200 mangoes. His gain percent is
(1)10%
(2)15%
(3)12%
(4) 25%
(5)None of thes
5.In how many different ways can the letters of the word DESIGN be arranged so that the vowels are at
the two ends ?
(1) 48
(2) 72
(3) 36
(4) 24
(5) None of these
6.A certain fraction is equivalent to 3/5 . If the numerator of the fraction is increased by 1 and the
denominator is decreased by 1, the new fraction is equivalent to 2/3 . What is the sum of numerator and
denominator of the original fraction?
(1)40
(2)32
(3)48
(4)24
(5)16

7.A shopkeeper sell his articles at cost price but uses 900 gm weight for 1200 gm weight. His profit
percentage is
(1)33.33%
(2)25%
(3)16.66%
(4)20%
(5)None of these
8.A car runs at the speed of 60 kms per hour when not serviced and runs at 72 kms/hr. when serviced.
After servicing the car covers a certain distance in 12 hours. How much time will the car take to cover the
same distance when not serviced ?
(1) 16.4 hours
(2) 13 hours
(3) 16 hours
(4) 14.4 hours
(5) None of these
9.The average of five positive numbers is 64. The average of the first two numbers is 59 and the average
of last two numbers is 63. What is the third number ?
(1)76
(2)56
(3)86
(4)Cannot be determined
(5)None of these
10.The ratio of income of A and B is 5:4 and their expenditure is as 3:2. If at the end of the year, each
saves Rs. 1200, then the income of A is
(1)Rs. 2550
(2)Rs. 2700
(3)Rs. 3000
(4)Rs. 3300
(5)None of these
Answers with solution !!!!

bankersadda.com

http://www.bankersadda.com/2015/08/quant-quiz.html

MISSION IBPS: Quant Quiz


Directions (1-5): Each question below is followed by two statements I and II. You are to determine whether
the data given in the statement is sufficient to answer the question. You should use the data and your
knowledge of Mathematics to choose between the possible answers.Give answer
(1)If the statement I alone is sufficient to answer the question, but the statement II alone is not sufficient.
(2)If the statement II alone is sufficient to answer the question, but the statement I alone is not sufficient.
(3)If either the statement I alone or the statement II alone is sufficient to answer the question.
(4)If you cannot get the answer from the statements I and II together, but need more data.
(5)If both statements I and II together are needed to answer the question.

1.What is the value of A + B?


I.A = 50% of B
II.2A + 2B = 40
2.How fast can Ram row a boat in still water?
I.He takes 5 minutes to row upstream for a distance of 1200 m.
II.He takes 20 seconds to row downstream for a distance of 1200 m.
3.Find the rate of simple interest.
I.The principal triples in 9 years.
II.The principal is Rs 1000.
4.What is the area of rectangle ABCD?
I.Length of its diagonal is 16 m.
II.Perimeter is more than 20 m.
5.What is the value of 2A + 3B?
I.A + 3B = 60
II.A is twice of B.
Directions (6-10): What should come in place of question mark (?) in the following number series?
6.200
231
264
299
336
?
1) 355
2) 375
3) 365
4) 395
5) None of these
7.10
1) 20
2) 18
3) 26
4) 156
5) 28
8.46656
1) 7
2) 6
3) 5
4) 4

22

7776

24

1296

216

36

5) 3
9.27
1) 512
2) 459
3) 519
4) 521
5) 524
10.96
1) 251
2) 255
3) 261
4) 264
5) 269

64

107

125

129

216

162

Answers with Explanation !!!!

343

206

bankersadda.com

http://www.bankersadda.com/2015/08/mission-ibps-quant-quiz.html

Mission IBPS :Quant Quiz


1.What would be compound interest obtained on an amount of Rs 4000 at the rate of 5% per Year after 3
year.
1)630.5
2)612
3)578
4)525.5
5)None of these
2.Shobit marks his goods 25% above cost price, but allows 17.5% discount for cash payment. If he sells
the article for Rs 825, find the cost price of the article.
1)Rs 800
2)Rs 810
3)Rs 820
4)Rs 823
5)None of these
3.A motorist travels a distance of 10km at a speed of 50km/hr in the onward journey and 60km/hr , while
returning. His average speed is ?
1)54.55km/hr
2)55km/hr
3)556/11km/hr
4)54 km/hr
5) None of these
4.Difference between the compound interest and simple interest accrued in two years at 8% per annum is
Rs 128. What is the principle amount ?
1)Rs 18000
2)Rs 16000
3)20000
4)Cannot be determined
5)None of these
5.The number obtained by interchanging the digits of a two digit number is less than the original number
by 18.If sum of the digits is 6, what was the original two digit number?
1)51
2)24
3)42
4)15
5)None of these
6.If the positions of the digits of a two-digit are interchanged, the number obtained is smaller than the
original number by 27. If the digits of the number are in the ratio 1: 2, what is the original number?
(1)36
(2)63
(3)48
(4)Cannot be determined
(5)None of these

7.Mr. Duggal invested Rs 20,000 with rate of interest @ 20 p.c.p.a. The interest was compounded half
yearly for first one year and in the next year it was compounded yearly. What will be the total interest
earned at the end of two years?
(1)Rs 8,800
(2)Rs 9,040
(3)Rs 8,040
(4)Rs 9,800
(5)None of these
8.A man sells an article at a gain of 20%. If he had bought it at 20% less and sold it for Rs.264 less, he
would have still gained 20%. The cost price of the article is:
(1)Rs.2400
(2)Rs.1100
(3)Rs.2640
(4)Rs.2635
(5)None of these
9.The compound interest on a certain sum for 2 years at 20% per annum is Rs. 880. The simple interest
on the same sum for double the time at half the rate percent per annum is:
(1)Rs.800
(2)Rs.1000
(3)Rs.1200
(4)Rs.1600
(5)None of these
10.Ramans monthly expenses is six time Ratans monthly expenses. Ratans monthly expenses is thirty
percent more than Govinds monthly expenses. Govinds monthly expenses is Rs. 64000 /- . What is
Ramans Annual Expenses?
(1)83,200/(2)9,98,400/(3)4,99,200/(4)104,400/(5)None of these
Answers with Explanation

bankersadda.com

http://www.bankersadda.com/2015/09/mission-ibps-quant-quiz_8.html

Mission IBPS :Quant Quiz


1.In a town, the population was 8000. In one year, male population increased by 10% and female
population increased by 8% but the total population increased by 9%. The number of males in the town
was :
(1) 4000
(2) 4500
(3) 5000
(4) 6000
2.In an examination, there were 1000 boys and 800 girls. 60% of the boys and 50% of the girls passed.
Find the percent of the candidates failed?
(1) 46.4
(2) 48.4
(3) 44.4
(4) 49.6
3.If A exceeds B by 40%, B is less than C by 20%, then A : C is :
(1) 28 : 25
(2) 26 : 25
(3) 3 : 2
(4) 3 : 1
4.Price of sugar rises by 20%. By how much percent should the consumption of sugar be reduced so that
the expenditure does not change?
(1) 20
(2) 10
(3) 16.66
(4) 15
5.In a school 70% of the students are girls. The number of boys are 510. Then the total number of
students in the school is :
(1) 850
(2) 1700
(3) 1830
(4) 1900
6.Applied to a bill for Rs. 1,00,000 the difference between a discount of 40% and two successive
discounts of 36% and 4% is :
(1) Nil
(2) Rs. 1,440
(3) Rs. 2,500
(4) Rs. 4,000
7.A tradesman marks his goods 10% above his cost price. If he allows his customers 10% discount on
the marked price, how much profit or loss does he make, if any?
(1) 1% gain
(2) 1% loss
(3) 5% gain
(4) No gain, no loss

8.The angles of a pentagon are in the ratio 1 : 2: 3: 5: 9, the largest angle is :


(1) 81
(2) 135
(3) 243
(4) 249
9.An agent gets a commission of 2.5% on the sales of cloth. If on a certain day, he gets Rs. 12.50 as
commission, the cloth sold through him on that day is worth :
(1) Rs. 250
(2) Rs. 500
(3) Rs. 750
(4) Rs. 1,250
10.A sum of money doubles itself at compound interest in 15 years. In how many years it will become
eight times?
(1) 30
(2) 45
(3) 50
(4) 60
Answers with explanation !!!!!!!!!!

bankersadda.com

http://www.bankersadda.com/2015/07/night-class-data-interpretation-quiz_20.html

Night Class: Data Interpretation Quiz


Directions: Read the table carefully to answer the questions that follows :
Percentage wise share distribution is given of different companies P, Q, R, S and T out of the total shares.

1. Total shares sold by companies P, R and T in year 2001 is what percentage of the total share of
company R ?
1) 24.67
2) 21.67
3) 25.67
4) 28.67
5) None of these
2. What is the average share sold by all the companies in year 2004 ?
1) 175
2) 160
3) 165
4) 155
5) None of these
3. What is the difference of share sold by companies P, T and Q in year 2005 together and same in year
2004 ?
1) 205
2) 204
3) 206
4) 305
5) None of these
4. What is the average share sold by company R in all the years together ?
1) 298
2) 200
3) 198
4) 398
5) None of these
5. Total no. of shares sold by companies P, Q and S in 2003 together ?
1) 493.5
2) 393.5
3) 593.5

4) 429
5) None of these
Direction: Read the line graph carefully to answer the questions below :

6. What is the total number of students hired by TCS for all centres ?
1) 2150
2) 2350
3) 2450
4) 3000
5) None of these
7. Total no. of students heired by all the companies for Indore centre is ?
1) 2500
2) 2400
3) 2450
4) 2300
5) None of these
8. What is the difference between total number of students heired by all the companies for lucknow centre
and Indore centre ?
1) 950
2) 850
3) 900
4) 925
5) None of these
9. What is the average no. of students heired by TCS for all centres taken together ?
1) 530
2) 630
3) 430
4) 420
5) None of these
10.What is the difference between total no. of students hired by all companies for Indore and average no.
of students for the same ?
1) 1825
2) 1925
3) 1725
4) 2025

5) None of these

1. 2
(30 + 50 + 50)/600 * 100 = 130/6 = 21.67
2. 3
(40 + 50 + 450 + 105 + 180)/5 = 165
3. 2
(24 + 150 + 300) - (40 + 50 + 180) = 204
4. 3
(50 + 40 + 240 + 450 + 210)/5 = 198
5. 2
96 + 140 + 157.5 = 393.5
6. 1
450 + 200 + 500 + 350 + 650 = 2150
7. 4
350 + 600 + 650 + 700 = 2300
8. 3
2300 - 1400 = 900
9. 3
2150/5 = 430
10. 3
2300 - 2300/4 = 1725

bankersadda.com

http://www.bankersadda.com/2015/07/night-class-data-interpretation-quiz.html

Night Class: Data Interpretation Quiz


Directions: Study the table carefully to answer the questions that follow :
Yearly fees (in Rs. thousands) of five different courses in five different years :

1. If 30% of the yearly fees were reduced for B.E course in the year 2011, what was the rectified yearly
fees for B.E course in the year 2011 ?
1) Rs. 2460 2) Rs. 5740 3) Rs. 6260 4) Rs. 3230 5) None of these
2. Total yearly fees for MBA course overall the years together was what percentage of total yearly fees of
MCA course in the year 2010, 2011 and 2012 together ?
1) 70.73%
2) 104.46%
3) 87.26%
4) 141.38%
5) None of these
3.What was the percentage increase in yearly fees of B. A in the year 2009 as compared to the previous
year ?
1) 84 8/13%
2) 45 7/24%
3) 61 5/24%
4) 26 2/13%
5) None of these
4.What was the difference between the total yearly fees for all the courses together in the year 2012 and
the yearly fees of B.Sc course in the year 2010 ? (in Rs. )
1) 34240
2) 28300
3) 37200
4) 25350

5) None of these
5.What was the average yearly fees of MCA course overall the years together ?
1) Rs. 7120
2) Rs. 6620
3) Rs. 5940
4) Rs. 6460
5) None of these
Directions: The following pie-chart shows the hourly distribution (in degrees) of all the major activities of
a student :
6.What per cent does he spend in school comparison
to sleeping ?
1) 81 7/11%
2) 87 7/9%
3) 122 2/9 %
4) 76 2/11 %
5) None of these
7.What is the difference between the time he spent in
Games and in Others ?
1) 3 hrs 20 min
2) 3 hrs 33 min
3) 3 hrs
4) 3 hrs 55 min
5) None of these
8. The percentage of time, which he spends in homework is
1) 11 2/9 %
2) 13 2/7 %
3) 90 %
4) 13 1/9%
5) None of these
9. If he spends the time in school equal to the sleeping and remains constant in other activities, then
percentage decrease in time of games is
1) 175
2) 57 1/7
3) 61 2/7
4) 72
5) None of these
10. If he spends 1/4th time of homework in Physics, then the number of hours he spends in rest of the
subject in homework is
1) 3
2) 4
3) 1
4) 6
5) None of these

ANSWERS

1. 2
Req Fee = 8200 * 70/100 = Rs. 5740
2. 4
(4.2 + 5.4 + 6.8 +7.6 + 8.8)/ (6.6 + 7.2 + 9.4) * 100
= 32.8/23.2 * 100 = 141.38%
3. 1
= (2.4 - 1.3)/1.3 * 100 = 1.1/1.3 * 100 = 84 8/13%
4. 3
= {(6.2 + 7.4 + 9.6 +8.8 + 9.4) - (4.2)}
= Rs. 37200
5. 2
= (4.5 + 5.4 + 6.6 + 7.2 + 9.4)/5
= Rs. 6620
6. 3
= 110/90 * 100 = 122 2/9
7. 1
= (85 - 35)/360 * 24 = 3hours 20min
8. 5
= 40/360 * 100 = 11 1/9%
9. 2
If the time spends in shcool is equal to that of spent in sleeping, then angle of sleeping is increased by
20degrees. Hence, his time of games is decreased by 20degrees.
= 20/35 * 100 = 57 1/7%

10. 3/4 * 40/360 * 24 = 2 hours

bankersadda.com

http://www.bankersadda.com/2015/07/night-class-data-interpretation-quiz_15.html

Night Class: Data Interpretation Quiz


Direction : Study the following graph carefully and answer the questions below :
Total Sales of Hindi and English newspaper in five different localities of a city :

1.What is the difference between the total sale of Hindi newspaper and the total sale of English
newspaper in all the localities together ?
1) 4000
2) 5000
3) 3000
4) 2500
5) None of these
2. The sale of Hindi newspaper in locality P is approximately what percent of the total sale of Hindi
newspaper in all the localities together ?
1) 20 %
2) 15 %
3) 18 %
4) 24 %
5) None of these
3.What is the ratio of the sale of Hindi newspaper in locality R to the sale of English newspaper in locality
T?
1) 12 : 7
2) 11 : 7
3) 7 : 12
4) 8 : 12
5) None of these
4. The sale of English and Hindi newspaper is locality P is what percent of the sale of English and Hindi
newspaper in locality S ?
1) 220
2) 125

3) 120
4) 135
5) None of these
5.What is the total sale of English newspaper in all the localities together ?
1) 22000
2) 20000
3) 25000
4) 21000
5) None of these
6.What is the average sale of English and Hindi newspaper in locality R and S ?
1) 9000
2) 14000
3) 12000
4) 8000
5) None of these
Direction: Study the following information carefully and answer the question given below it :
Percentage proportion of income of seven companies in a state during financial year 2013 14 :

7. If the expenditure of company D is 750 crore, what was the total income of all the given companies
together ?
1) Rs. 4135 crore
2) Rs. 4115 crore

3) Rs. 4125 crore


4) Rs. 4025 crore
5) None of these
8. If in the given financial year income of company A was 440 crore. What was the approximately
expenditure of company C in that year ?
1) Rs. 45 crore
2) Rs. 55 crore
3) Rs. 46 crore
4) Rs. 35 crore
5) None of these
9.What is the ratio of the expenditure of company D to company C ?
1) 80 : 11
2) 11 : 80
3) 12 : 70
4) 70 : 12
5) None of these
10. If the difference between the income of company F and company A is Rs. 750 crore, what is
the expenditure of both the companies together ?
1) Rs. 6400 crore
2) Rs. 7400 crore
3) Rs. 8400 crore
4) Rs. 5400 crore
5) None of these
11.Which of the following companies has recorded the maximum profit in this financial year if the
combined income of these companies is Rs. 4500 crore ?
1) A
2) B
3) F
4) C
5) None of these
12. If the income of company D and B together is Rs. 270 crore then fined the profit of company B in that
year ?
1) Rs. 14.25 crore
2) Rs. 15.5 crore
3) Rs. 11.75 crore
4) Rs. 12.5 crore
5) None of these
Answers:
1. 3
2300 - 20000 = 3000
2. 4
5500/23000 * 100 = 23.91%
3. 1
6000/3500 = 12/7 = 12:7
4. 3
9000/7500 * 100 = 120

5. 2
6. 4
(8500 + 7500)/2 = 16000/2 = 8000
7. 3
20% * 100/110 * total = 750
Total = 4125
8. 2
20% = 440
1% = 440/20 = 22
3% = 66 crore
exp = 66 * 100/120 = 55 crore
9. 1
10. 4
5% = 750
1% = 150
(45*150)/125 * 100 = 5400 crore
11. 3
12. 3
(20% + 10%) = 270
10% = 90
Ib = 90
Eb = (90*100)/115 = 78.26
Pb = 11.739

bankersadda.com

http://www.bankersadda.com/2015/07/night-class-data-interpretation-quiz_14.html

Night Class: Data Interpretation Quiz


Direction : Study the table carefully to answer the questions that follow :
Percentage of marks obtained by different students in different subjects :

1.What is the total marks obtained by Nisha in all the subject together ?
1) 256
2) 406
3) 356
4) 425
5) None of these
2.What is the approximate average marks obtained by all the students together in Geography ?
1) 48
2) 38
3) 54
4) 28
5) None of these
3. The number of students who got 70 % marks or above in all the subjects are
1) 4
2) 3
3) None
4) 6
5) None of these
4. How many students have scored more than 120 marks in Maths ?
1) 3
2) 2
3) 1
4) 4
5) None of these
5. If 120 marks are required to pass in Maths, How many students pass in the Maths exams
1) 4
2) 5

3) 4
4) 3
5) None of these
6. In which subject the overall percentage is the highest
1) Maths
2) Physics
3) Hindi
4) History
5) None of these
7.What is the ratio of total number of marks obtained by Rakesh in Physics and Chemistry together to
Chinu in the same subjects?
1) 17 : 23
2) 23 : 17
3) 23 : 18
4) 23 : 27
5) None of these
Direction : Study the following information to answer the given questions :
Percentage of students in various courses of Engineering like Electronics, IT, CSE, Mechanical, Mining
and Civil Engineering.

8. How many girls are there in course Mechanical Engineering and Computer Science Engineering
together ?
1) 440
2) 640
3) 540
4) 240
5) None of these
9. For which pair of courses is the number of boys same ?
1) IT and Electronics
2) Civil and Mechanical
3) IT and Civil
4) CSE and IT
5) None of these
10. For which courses the number of boys are the minimum ?
1) Civil Engineering
2) Electronics Engineering
3) Mining Engineering
4) Computer Science Engineering
5) None of these

11. For course I.T. What is the ratio of the boys to girl ?
1) 122 : 102
2) 102 : 122
3) 32 : 42
4) 20 : 25
5) None of these
12.What is the total number of student in mining and Computer Science Engineering together ?
1) 760
2) 690
3) 640
4) 670
5) None of these
13.What is the ratio between boys in Mechanical and Civil Engineering together to girls in the same
subject ?
1) 134 : 45
2) 136 : 45
3) 45 : 136
4) 38 : 136
5) None of these
14.What is the average number of girls in course I.T Mechanical and Civil Engineering together ?
1) 98
2) 108
3) 105
4) 118
5) None of these

Answers:
1. 2
150 + 75 + 50 + 48 + 48 + 35 = 406
2. 3
(60 + 92 + 76 + 72 + 64 + 60 + 80)/7 * 75/100 = 54
3. 3
4. 2
120 marks out of 150 marks = 120/150 * 100 = 80%
Auhutosh and Nisha
5. 4
120 marks out of 150 marks = 120/150 * 100 = 80%
Ashutosh, Rakesh and Nisha
6. 1
7. 2

8. 3
C = 1200 * (35% + 10%) = 1200 * 45% = 540
9. 3
10. 2
11. 2
12. 3
13. 1
14. 2
(144 + 120 + 60)/3 = 324/3 = 108

bankersadda.com

http://www.bankersadda.com/2015/07/night-class-data-interpretation-quiz_13.html

Night Class: Data Interpretation Quiz


Direction : The following table gives the percentage of population of Five States UP, MP, Rajasthan, Bihar
and Punjab which are below poverty line and also the ratio of male to female among them :
Study the table and answer the questions based on it

1.What will be the male population above poverty line for state U.P if the female population below poverty
line for state U.P is 18 million ?
1) 29.6 million
2) 39.6 million
3) 49.6 million
4) 19.6 million
5) None of these
2.What will be the number of females below poverty line in state Rajasthan if the total population of state
is 20 million ?
1) 5 million
2) 6 million
3) 3 million
4) 4 million
5) None of these
3. If the male population above poverty line in state Bihar is 21 million, then find the total population of
the state is
1) 48 million
2) 131.5 million
3) 75 million
4) 121.5 million
5) None of these
4. If the population of males below poverty line for state M.P is 15 million and that for state Punjab is 18
million , then the ratio of total population of state M. P to Punjab is
1) 40 : 27
2) 30 : 27
3) 50 : 27
4) 27 : 50

5) None of these
5. If the below poverty line population of Rajasthan is 35 million, Find the number of males in Rajasthan,
who are above poverty line
1) 35.45 million
2) 36 million
3) 34.94 million
4) 38.94 million
5) None of these
6. If the above poverty line population of state Punjab is 75 million find the below poverty line population
of female in state Punjab ?
1) 40 million
2) 15 million
3) 25 million
4) 20 million
5) None of these
Direction : Study the following table carefully and answer the questions given below :
Number of students enrolled with five colleges over the years :

7. If from college Q in 2009, 90 % of the students enrolled appeared in a competitive examination, out of
which 75 % students passed, how many students passed the examination ?
1) 280
2) 216
3) 243
4) 270
5) None of these
8. In 2007, from all the colleges together an overall 40 % of the students enrolled for a computer course
total how many students enrolled for these course ?
1) 700
2) 600
3) 800
4) 900
5) None of these
9.What is the ratio of the average number of student enrolled with colleges together in 2009 to that in
2010 ?

1) 117 : 108
2) 111 : 113
3) 110 : 113
4) 105 : 108
5) None of these
10. The Average number of students enrolled from college Q for all the years together is approximately
what percent of the average number of students enrolled from college R for all the years together ?
1) 83
2) 85
3) 87
4) 80
5) None of these
11. In 2008, from all colleges together 10 % students enrolled went abroad. Approximately how many
students went abroad?
1) 311
2) 211
3) 321
4) 521
5) None of these
12.What is the average number of students enrolled for college R for all the years is
1) 302
2) 502
3) 402
4) 602
5) None of these

Answers:
1. 2;
Female population below poverty line = 18 million
5 : 6 = x : 18
x = 15
Total population (below poverty line) = 15+18 = 33mn
Since 20% population in up is below poverty line
Therefore 80% population will be above poverty line
20 : 40 = 33 : y (Male population above poverty line)
1 : 4 = 33 : y y = 132
Male population above poverty line
= (132 * 3)/10 = 39.6 million
2. 3;
Total population (below poverty line)
= 20 35% = 7 million
Female below poverty line = 7/7 * 3 = 3 million
3. 3;
Since 7 ratio = 21
1 ratio = 3
Total ( 7 + 8) 3 = 45 million

60 % = 45
1 = 45/60 * 10 million
4. 3;
50 : 27
5. 1;
6. 2;
7. 3; 360 * 9/10 * 3/4 = 243
8. 4;
(480 + 350 + 380 + 500 + 540) 40 % = 900
9. 3;
10. 1;
(320 + 350 + 300 + 360 + 340)/(400 + 380 + 410 + 430 + 390) * 100
= 1670/2010 * 100 = 83.08%
11. 2;
(420 + 380 + 410 + 520 + 460) 10 %
= 2110 * 10/100 = 211
12. 3;
(400 + 380 + 410 + 430 + 390)/ 5 = 2010/5 = 402

bankersadda.com

http://www.bankersadda.com/2015/07/night-class-data-interpretation-quiz_9.html

Night Class: Data Interpretation Quiz


Directions: Study the following table to answer these questions.
Percentage of marks obtained by six students in six different subjects. Number given in small brackets
indicates the full marks of the subject.

1. What is the average percentage of marks obtained by all the three girls (Radha, Pratigya and Renuka) in
English?
1) 63.67
2) 67.33
3) 66.66
4) 65.33
5) None of these
2. Marks obtained by all the three boys together in Hindi is how much more than that by all the three girls
together in Hindi?
1) 9
2) 12
3) 25
4) 21
5) None of these
3. In which subject has Pratigya got the least marks?
1) Computer Sc
2) Economics
3) Hindi
4) English
5) None of these
4. What is the total marks obtained by Renuka in all the subjects?
1) 378
2) 398
3) 368
4) 408
5) None of these
5. What is the average marks obtained by these six students in Economics?
1) 52
2) 52.5
3) 55.5

4) 53.5
5) None of these
Directions: Following line graph shows sugar produced by India and Sri Lanka during the period of 2007
2012.

6. What is the percentage decrease in production from 20072008 for India?


1) 6%
2) 8 4/7%
3) 1 4/7%
4) 9%
5) None of the
7. What is the average production of Sri Lanka from 2007 to 2010?
1) 67
2) 64
3) 63
4) 59
5) 57
8. What is the ratio of the maximum production to average production for India?
1) 73 : 57
2) 73 : 59
3) 73 : 62
4) 73 : 64
5) None of these
9. What is the difference of total production of these two countries during 20072012?
1) 18
2) 20
3) 22
4) 24
5) 26
10. For Sri Lanka during which period its production increases maximum from the previous year?
1) 20072008
2) 20082009
3) 20092010
4) 20102011
5) 20112012
ANSWERS:
1. 4
Average percentage of marks (of girls)

= (72 + 76 + 48)/3 = 65.33


2. 5
Marks obtained by the boys (in Hindi)
= (75% of 60) + (65% of 60) + (70% of 60)
= 45 + 39 + 42 = 126
Marks obtained by girls
= (65% of 60) + (75% of 60) + (45% of 60)
= 27 + 39 + 45 = 111
Reqd Difference = 126 - 111 = 15
3. 3
In Hindi, she has got only 39 marks.
4. 2
Total marks gained by Rani
= (90% of 120 + 48% of 75 + 75% of 60 + 68% of 75 + 76% of 150 + 88% of 50
= 108 + 36 + 45 + 51 + 114 + 44 = 398
5. 3
Avg in Eco =
(92 + 80 + 52 + 64 + 88 + 68)/6 of 75
= 444/ 6*100 * 75 = 5505
6. 2
(70 - 64)/70 * 100 = 60/7 = 8 4/7%
7. 5
Average = (55 + 48 + 75 + 50)/4 = 228/4 = 57
8. 3
= (70 + 64 + 45 + 60 + 6 0 + 73)/6 = 372/6 = 62
Maximum = 73
Ratio = 73:62
9. 3
Production of India = 372
Production of Sri Lanka = 350
Diference = 372 - 350 = 22
10. 2

bankersadda.com

http://www.bankersadda.com/2015/07/night-class-data-interpretation-quiz_8.html

Night Class: Data Interpretation Quiz


Directions : Study the table carefully to answer the questions that follow:
Percentage of students in MBA class, who sought employment in the area of Finance, Marketing and
Software :
Average starting salaries of the students per month
(in ` thousands) in these areas :
Number of Students passed out :
1. In 2007, students seeking jobs in finance earned
more than those opting for software (per annum) by

1) Rs. 1.764 lakhs


2) Rs. 17.64 lakhs
3) Rs. 21.168 lakhs
4) Rs. 12 lakh
5) None of these
2.What is the percentage increase in the average
monthly salary offered to a software student over the
given five years ?
1) 70
2) 41
3) 67
4) 43
5) None of these
3. The number of students, who get jobs in finance is
less than the students getting software Jobs, in the
five years by
1) 4
2) 42
3) 9
4) 226
5) None of these
4.What is the percentage increase in the average
salary of finance from 20042008 ?
1) 26%
2) 29%
3) 36%
4) 32%
5) None of these
5. The average annual rate at which the initial salary
offered in marketing increase is
1) 31.5 %
2) 22.4 %
3) 71.32 %
4) 29 %

5) None of these
Directions: Study the graph carefully to answer the questions that follow :
Profit (In Rs. lakhs) made by 3 companies over the years :

6.What is the percent increase in profit made by company A over the years ?
1) 150
2) 60
3) 120
4) 82
5) None of these
7. Profit made by company C in the year 2009 was what percent of total profit made by all the three
companies in that year?
1) 111 1/2
2) 51 1/7
3) 46 2/7
4) 42 6/7
5) None of these
8. If the income of company B in the year 2010 was ` 14 lakhs, what was its expenditure in that year ?
1) Rs. 20 lakh
2) Rs. 13 lakh
3) Rs. 16 lakh
4) Rs. 7 lakh
5) None of these
9.What is the percent increase in profit of company A in the year 2009 from the previous year ?
1) 50
2) 33 1/3
3) 66 1/6
4) 42
5) None of these
10. If the expenditure of company C in the year 2013 was ` 14.44 lakhs, what was its income in that year ?
1) Rs. 22.44 lakh
2) Rs. 17.5 lakh
3) Rs. 16.22 lakh
4) Can't be determined

5) None of these

Answers
1. 3
Req difference in the amount earned = (16/100 * 1000 - 14/100 * 1000 * 7200) * 12
= Rs. 21.168 lakh
2. 5
Req% increase = (7280 - 4240)/4240 * 100 = 72% approx
3. 1
Req Difference = (0.30 * 400 + 0.34 * 600 + 0.32 * 200 + 0.14 * 1000 + 0.16 * 800) - (0.24 * 400 + 0.18 * 600 +
0.32 * 200 + 0.16 * 1000 + 0.28 * 800) = 4
4. 3
Req % increase = (8440 - 6210)/6210 * 100 = 36% approx

5. 2
Req annual rate = {(9250 - 4880)/4880 * 100} * 1/4
= 22.3873 = 22.4% approx
6. 1
Req % increase = (5 - 2)/2 * 100 = 150%
7. 4
Req % = 6/(3 + 5 + 6) * 100 = 42 6/7%
8. 5
Req expenditure = 14 - 6 = Rs. 8 lakhs
9. 1
Req % increase = (3 - 2)/2 * 100 = 50
10. 4
Profit made by company in the year 2013 is not given.

bankersadda.com

http://www.bankersadda.com/2015/07/night-class-data-interpretation-quiz_27.html

Night Class: Data Interpretation Quiz


Directions : Study the following Grpah carefully and answer the questions given below:
Number of Students Appearing for Aptitude Test from Various Towns (Number in thousands)

1. What is the average number of students appearing for Aptitude test from all the Towns together?
1) 33500
2) 3350
3) 17500
4) 33.5
5) None of these

2. The number of students appearing for the Aptitude test from Town D is approximately what percent of
the number of students appearing for the Aptitude test from Town C?
1) 243
2) 413
3) 134
4) 341
5) 143
3. What is the respective ratio of the number of students appearing for the Aptitude test from Town C and
D together to the number of students appearing for the Aptitude test from Town A, D and E together?
1) 11 : 13
2) 20 : 43
3) 20 : 47
4) 37 : 20
5) None of these
4. What is the respective ratio of the number of students appearing for the aptitude test from Town B to
Town A?
1) 3 : 4
2) 13 : 16
3) 11 : 16
4) 2 : 3
5) None of these
5. The number of students appearing for the aptitude test from Town E is approximately what percent of
total number of students appearing for the Aptitude test from all the Towns together?
1) 15

2) 17
3) 19
4) 21
5) 23
Directions: In the given pie -charts the distribution of passed students from six different colleges (A, B,
C, D, E and F) of a city during the year 2012 and 2013 is given. In the line graph the percentage of boys
passed in year 2012 and the percentage of girls passed in 2013 is shown. Answer the following questions
based on these graphs.
Total students passed in 2012 = 30000
Total students passed in 2013 = 40000

6. What is the number of female students passing in 2012 from College C?


1) 2000
2) 2300
3) 2400
4) 2500
5) 2700
7. What is the number of male students passing in year-2013 from College F?
1) 5760
2) 5750
3) 5740
4) 5730

5) 5720
8. What is the total number of females passing in 2012 from all colleges?
1) 14645
2) 15645
3) 16645
4) 17645
5) 18645
9. What is the difference between the number of male students passing in year 2013 and that of female
students passing in the same year?
1) 1690
2) 1680
3) 1670
4) 1660
5) 1650
10. Number of boys passing from College E in 2012 is what percentage of the number of boys passing
from College C in year 2013?
1) 70%
2) 80%
3) 90%
4) 100%
5) 110%
Answers:
1. 1
Reqd avg marks = (40 + 32.5 + 17.5 + 42.5 + 35)/5 thousand
= 16.5 * 1000/5 = 33500
2. 1
Reqd pecentage = (42.5 * 1000)/(17.5 * 1000) * 100%
= 243% approx
3. 1
Required ratio = (17.5 + 42.5)thousand : (40 + 42.5 + 35) thousands
= 60 : 117.5
= 20 : 39.16
= 20 : 39
4. 2
Reqd Ratio = 32500 : 40000
5. 4
Reqd percentage = (35000 * 100)/(40 + 32.5 + 17.5 + 42.5 + 35) * 1000 * 100%
= 35000/1675% = 21% approx
6. 5
50% of (64.8/360 * 30000)
= 50/100 * 5400 = 2700
7. 60% of (86.4/360 * 40000) = 60/100 * 9600
= 5760
8. 2
Total = 30000/(360 * 100) [54 * 40 + 90 * 55 + 64.8 * 50 + 50.4 * 60 + 43.2 * 45 + 57.6 * 60]

30000/(360 * 100) [2160 + 4950 + 3240 + 1944 + 3456]


30000/36000 * 18774 = 15645
9. 2
Total boys = 4000 + 4800 + 1800 + 1600 + 2880 + 5760 = 20840
Total girls = 40000 - 20840 = 19160
Diff = 20840 - 19160 = 1680
10. 5
E-Boys2012 = 43.2/360 * 30000 * 55/100 = 1980
C-Boys2013 = 36/360 * 40000 * 45/100 =1800
Req% = 1980 * 100/1800 = 110%

bankersadda.com

http://www.bankersadda.com/2015/09/night-class-quant-quiz_16.html

Night Class: Quant Quiz


Direction : Following line graph shows the ratio of imports to exports of two companies over the years :

1. In how many of the given years were the imports more than the exports in case of company - A ?
1) 5
2) 3
3) 1
4) 2
5) 4

2. If the imports of company-B in year 2011 were ` 51.688 lakh, then what were the exports of company-B
in the same year?
1) 64.26 lakh
2) 67.54 lakh
3) 71.36 lakh
4) 73.84 lakh
5) None of these
3. If the exports of company-A and Company-B were equal to 84 lakh in year 2012 then what will be the
difference between imports of company-B and imports of company-A in that year ?
1) 9.6 lakh
2) 8.4 lakh
3) 7.2 lakh
4) 6.8 lakh
5) 5.4 lakh
4. If the exports of company-A in year 2009 and exports of company-B in year 2012 were equal then the
imports of company- B in 2012 is approximately what percentage of imports of company-A in 2009 ?
1) 60%
2) 75%
3) 84%
4) 96%
5) 133.3%
5. In year 2009, If the export of company-B is increased by 100% and import is increased by 200%. Then

what will be the new ratio of import to export of company-B in that year ?
1) 0.8
2) 1.0
3) 1.2
4) 0.6
5) 1.5
6. A cycle is sold at a profit of 15%. If both the cost price and selling price is decreased by Rs. 200 the
profit would be 5% more what is the original cost price of cycle ?
1) Rs. 1200
2) Rs. 1000
3) Rs. 800
4) Rs. 600
5) None of these
7. A person lends Rs. 4200 to 'A' for 3 years and Rs. 6500 to 'B' for 7 years. If the person gets Rs. 8134 as
interest then what is the rate of interest per annum ?
1) 6%
2) 8%
3) 10%
4) 12%
5) 14%
8. A sum amounts to Rs. 98010 in 2 years and to Rs. 107811 in 3 years compounded annually. What is the
sum ?
1) Rs. 81000
2) Rs. 84000
3) Rs. 87000
4) Rs. 90000
5) Rs. 94000
9. How many different words can be formed with the letters of the word ''BREAKING". So that the words
begin with 'B' and end with 'G' ?
1) 120
2) 720
3) 5040
4) 2520
5) 1440
10. A bag contains 9 white and 7 black balls. Three balls are drawn at random. Find the probability that all
the three balls are black ?
1) 1/8
2) 1/12
3) 1/16
4) 1/21
5) None of these
1. 4
Imports > Exports
Import/ Exports > 1
2. 4
I/E = 0.7
E = I/0.7 = 51.688/0.7 = 73.84 lakh

3. 2
Ia/Ea = 0.8
Ia = 0.8 * 84 = 67.2 lakh
Ib/Eb = 0.9
Ib = 0.9 * 84 = 75.6 lakh
Diff = 75.6 - 67.2 = 8.4 lakh
4. 2
Ia/Ea = 1.2
Ib/Eb = 0.9
Let export of A and B are X
Ia = 1.2x
Ib = 0.9x
Req% = 0.9x/1.2x * 100 = 75%
5. 3
I/E = 0.8
I1 = I + 200I/100 = 3I
E1 = E + 100E/100 = 2E
Ratio = I1/ E1 = 3I/2E = 3/2 * 0.8 = 1.2
6. 3;
Let the original cost price is x, so selling price is
115 x/100 = 23 x/ 20
Now CP = x 200,
S.P. = 23x/20 - 200
Profit = 23x/20 200 (x 200) = 23x/20 x = 3x/20
% profit = 3x/20 * 1/(x - 200) * 100 = 20
3x/(x - 200) = 4
3 x = 4 x 800
x = 800
7. 5;
Let the rate of interest is r% p.a.
4200 * r * 3/100 + 6500 * r * 7/100 = 8134
126 r + 455 r = 8134
r = 8134/581 = 14% pa
8. 1
Let the principal is P and rate of interest is r
P (1 + R/100)^3 = 107811
P (1 + R/100) ^2 = 98010
Dividing eq I by II
1 + R/100 = 107811/98010
R/100 = 9801/98010
R = 980100/98010 = 10% pa

P = (1 + 10/100)^2 = 98010
p = 98010 * 100/121 = 81000
9. 2
Now 6 letters are left for six different place
Req No. of wages = 6P6 = 6! = 720
10. 3
n(S) = 16C3 = 560
n (E) = Number of ways of selecting 3 balls out of 7 =
7C3 = 7!/3!4! = 35
P(E) = 35/560 = 1/16

bankersadda.com

http://www.bankersadda.com/2015/08/night-class-quant-quiz_27.html

Night Class: Quant Quiz


1. A dishonest shopkeeper announces to sell rice at the cost price, but he uses a false weight of 920 gm
instead of a kilogram. What is the gain percentage earned by the shopkeeper?
1) 8.69%
2) 7.12%
3) 6.25%
4) 4.62%
5) 9.14%

2. A man was engaged on a job for 40 days on the condition that he will get a wage of Rs. 180 for the
day he works, but he will have to pay a fine of Rs. 20 for each day of his absence. If he gets Rs. 520 at the
end of the month, then, he was absent for how many days?
1) 12 days
2) 10 days
3) 6 days
4) 8 days
5) 14 days
3. Find the least number which when divided by 25, 30, 35 and 40 leaves the remainder 18, 23, 28 and 33
respectively.
1) 4186
2) 4193
3) 4196
4) 4183
5) 4207
4. Excluding stoppages, the speed of a bus is 54 kmph and including stoppages, it is 36 kmph. For how
many minutes does the bus stop per hour?
1) 20 min
2) 25 min
3) 30 min
4) 18 min
5) 32 min
5. At present Manorama is six times older than her son's age. Seven years hence, the ratio of their ages
will be 11:3. What is Manorama's present age?
1) 42 years
2) 54 years
3) 63 years
4) 36 years
5) 48 years
6. How many kg of pure salt must be added to 40 kg of 4% solution of salt and water to increase it to 12%
solution?
1) 2.04 kg
2) 2.64 kg
3) 3.14 kg
4) 3.64 kg

5) 5.64 kg
7. The average of the test scores of a class of 'x' students is 80 and that of 'y' student is 94. When the
scores of both the classes are combined, the average becomes 86. What is the ratio of x to y?
1) 6 : 5
2) 5 : 4
3) 4 : 3
4) 7 : 6
5) 3 : 2
8. In a triangle, two sides of 4 cm and 3 cm are at right angle. If the triangle is revolved along the 4 cm
side, the curved surface area of the cone so formed will be
1) 62.8 cm^2
2) 52.6 cm^2
3) 64. 8 cm^2
4) 67.6 cm^2
5) 66.6 cm^2
9. If 35% of a number is subtracted from the second number the second number reduces to its four
fifths. What is the ratio the first number to the second number?
1) 4 : 7
2) 4 : 9
3) 5 : 9
4) 5 : 7
5) 6 : 7
10. The ages of Anshu, Bharti and Charita together are 57 years. Bharti is thrice as old as Anshu and
Charita is 12 years older than Anshu. Then, the respective age of Anshu, Bharti and Charita is:
1) 8, 26, 23 (years)
2) 9, 27, 21 (years)
3) 8, 25, 24 (years)
4) 9, 26, 22 (years)
5) 9, 28, 20 (years)
Answers:
1. 1
He sells 920 grams of rice and gains 80 grams
Gain % = 80/920 * 100 = 8.69%
2. 2
Let the number of days he was absent be x days.
180 (40 x) 20 x = 5200
7200 180x 20x = 5200
7200 200x = 5200
x = 2000/20 = 10 days
3. 2
Required number = LCM of (25, 30, 35, 40) 7
= 4200 7 = 4193
4. 3
Time stopage rest per hour
Difference of speed/ Speed without stoppage = 60 minute

Now, 54-36/36 * 60 = 30 min per hour


5. 5
Let Manorama sons age be x years
Manoramas age = 6x
(6x + 7)/(x + 7) = 11/3
18x + 21 = 11x + 77
7x = 56
x=8
Manoramas present age = 6x = 48 years
6. 4;
Amount of salt in 40 kg of solution = 4/100 * 40 = 1.6 kg
Let x kg of pure salt be added.
Then, (1.6 + x)/ (40 + x) = 12/100
160 + 100x = 480 + 12x
88x = 320
x = 3.64 kg
7. 3
(80x + 94y)/(x + y) = 86
80x + 94y = 86x + 86y
8y = 6x
x/y = 8/6 = 4/3 = 4 : 3
8. 1;
Radius of formed cone = 4 cm
Slant height = 5 cm
Surface area of formed cone = rl = 22/7 * 4 * 5 = 62.8 cm^2
9. 1
N2 - 35% of N1 = 4/5 N2
N2 - 4/5N2 = 35/100N1
N2/5 = 35/100N1
N1/N2 = 1/5 * 100/35 = 4 : 7
10. 2
Let the age of Anshu be x years
Bhartis age = 3x
Charitas age = x + 12
x + 3x + x + 12 = 57
or, 5x + 12 = 57
x=9
Anshu, Bharti and Charitas age are 9, 27, 21 years

bankersadda.com

http://www.bankersadda.com/2015/08/night-class-quant-quiz_26.html

Night Class: Quant Quiz


1. ? % of 824 + 244 = 1480
1) 140
2) 150
3) 100
4) 180
5) 120
2. 24.5 8.4 16 = ?
1) 3292.8
2) 3492.8
3) 3294.8
4) 3192.8
5) 3094.8
Directions: In each of these questions, two equations numbered I and II with variables x and y are given.
You have to solve both the equations to find the value of x and y. Give answer:
1) if x > y
2) if x >= y
3) if x < y
4) if x <= y
5) if x = y or relationship between x and y cannot be determined.
3.
I. 2x^2 + 13x + 20 = 0
II. 2y^2 3y 35 = 0
4.
I. 12x^2 41x + 35 = 0
II. 4y^2 17y +15 = 0
5.
I. 4x^2 4 = 60
II. 3y^2 + 3 = 51
6.
I. 28x^2 9x 9 = 0
II. 7y^2 + 24y + 9 = 0
Directions: In the following number series one number is wrong. Find out the wrong number.
7. 16, 40, 100, 250, 630
1) 630
2) 100
3) 250
4) 40
5) 16
8. 21, 48, 162, 969, 7748
1) 7748
2) 21

3) 969
4) 48
5) 162
9. 118, 239, 96, 264, 68, 293
1) 293
2) 264
3) 96
4) 68
5) 239
10. 128, 320, 1120, 5040, 27720, 180190
1) 27720
2) 5040
3) 320
4) 1120
5) 180190
Answers:
1. 2
2. 1
3. 5
I. 2x^2 + 8x + 5x + 20 = 0
(2x + 5) (x + 4) = 0 x = -5/2, 4
II. 2y^2 10y + 7y 35 = 0
(2y + 7) (y 5) = 0 y = -7/2, 5
Relationship between x and y does not exist.
4. 5;
I. 12x^2 20x 21x + 35 = 0
(4x 7) (3x 5) = 0
x = 7/4, 5/3
II. 4y^2 12y 5y + 15 = 0 (4y 5) (y 3) = 0
y = 5/4, 3
Relationship between x and y does not exist
5.5;
I. 4(x^2 1) = 60
x^2 1 = 15
x = +/- 4
II. 3(y^2 + 1) = 51
y^2 + 1 = 17
y = +/- 4
Hence x = y
6. 2;
I. 28x^2 21x + 12x 9 = 0
(7x + 3) (4x 3) = 0
x = -3/7, 3/4
II. 7y^2 + 21y + 3y + 9 = 0
(7y + 3) (y + 3) = 0
y = -3/7,3

7. 1
16 * 5/2 = 40
40 * 5/2 = 250
250 * 5/2 = 625 not equal to 630
Hence there should be 625 in place of 7630
8. 4:
The series is:
21 2 1 = 41 not equal to 48
41 4 2 = 162
162 6 3 = 969
969 8 4 = 7748
Hence, there should 41 in place of 48.
9. 3;
The series is:
118 + (11)^2 = 239
239 (12)^2 = 95 not equal to 96
95 + (13)^2 = 264
264 (14)^2 = 68
68 + (15)^2 = 293
Hence there should be 95 in place of 96.
10. 5
The series is
128 * 5/2 = 320
320 * 7/2 = 1120
1120 * 9/2 = 5040
5040 * 11/2 = 27720
27720 * 13/2 = 180180 not equal to 180190

bankersadda.com

http://www.bankersadda.com/2015/08/night-class-quant-quiz_25.html

Night Class: Quant Quiz


Directions: Study the following pie-charts carefully and answer the questions given below:
Number of students studying in different colleges in the year 2013 and 2014 from state 'A'

1. What was the percentage increase in the number of students in college C5 in the year 2014 as
compared to the year 2013?
1) 45%
2) 40%
3) 48%
4) 50%
5) 42%
2. In which of the following colleges was the per cent increase in the number of students the minimum in
2014 from 2013?
1) C2
2) C6
3) C4
4) C7
5) C1
3. In which college there was decrease in the number of students from 2013 to 2014?
1) C6
2) C1
3) C4
4) C2
5) C3
4. In the year 2013, the number of students studying in college C1 and C2 together is approximately what
percent of the number of students studying in these colleges together in 2014?
1) 78%
2) 75%
3) 72%
4) 81%
5) 88%

5. What was the ratio of the number of students studying in college C6 in year 2013 to that in 2014 in the
same college?
1) 25 : 26
2) 25 : 27
3) 23 : 26
4) 22 : 27
5) 23 : 25
Directions : Each question below is followed by two statements A and B. You have to determine whether
the data given in the statement is sufficient to answer the question. You should use the data and your
knowledge of mathematics to choose between the possible answers. Give answer:
1) if the statement A alone is sufficient to answer the question, but the statement B alone is not sufficient.
2) if the statement B alone is sufficient to answer the question, but the statement A alone is not sufficient.
3) if the both statement A and B together are needed to answer the question.
4) if either the statement A alone or B alone is sufficient to answer the question.
5) if you cannot get the answer from the statements A and B together, but needed even more data.
6. What is the perimeter of a semicircle?
A. The radius of the semi-circle is half of the length of the rectangle.
B. Area of rectangle is 234 cm2.
7. What is the ratio of two number N1 and N2?
A. 30% of N1 is 40% of 90.
B. 20% of N2 is 36% of 50.
8. What are the marks scored by Atul in Science?
A. Marks scored by Atul in Science is 40% of the marks obtained by him in Maths i.e 80.
B. Marks scored by Atul in Science and English is 180.
1. 4; Number of students in college C5
In 2014 = 15/100 * 30000 = 4500
In 2013, 12/100 * 25000 = 3000
Required % Increase = (4500 - 3000)/3000 * 100 = 50%
2.
For 2013:
C1 ---- 17 250 = 4250
C2 ---- 20 250 = 5000
C3 ---- 21 250 = 5250
C4 ---- 10 250 = 2500
C5 ---- 12 250 = 3000
C6 ---- 15 250 = 3750
C7 ---- 5 250 = 1250
For 2014:
C1 ---- 16 * 300 = 4800
C2 ---- 22 * 300 = 6600
C3 ---- 20 * 300 = 6000
C4 ---- 8 * 300 = 2400
C5 ---- 15 * 300 = 4500
C6 ---- 13 * 300 = 3900

C7 ---- 6 * 300 = 1800


Percentage Increase
C1 ---- 12.94%
C2 ---- 32%
C3 ---- 14.28%
C4 ---- Decrease
C5 ---- 50%
C6 ---- 4%
C7 ---- 44%
Hence, minimum increase is in the college C6
3. 3
Hence, decrease is in the college C4.
4. 4
Required % = [(17 + 20) * 250]/[(16 + 22) * 300] * 100 = 81% approx
5. 1
Required ratio = 15 250 : 13 300 = 25 : 26
6. 5
Combining the both statements A and B will not give the perimeter of semicircle.
7. 3
From A,
30/100 N1 = 40/100 * 90
N1 = 120
From B,
20/100 N2 = 36/100 * 50
N2 = 90
Combining both statements A & B:
N1/N2 = 120/90 = 4/3 = 4:3
8. 1
From statement A,
Required answer = 40/100 * 82 = 32

bankersadda.com

http://www.bankersadda.com/2015/08/night-class-quant-quiz_20.html

Night Class: Quant Quiz


Directions (Q. 14): In each of these questions, two equations numbered I and II with variables x and y
are given. You have to solve both the equations to find the value of x and y. Give answer
1) if x > y
2) if x >= y
3) if x < y
4) if x <= y
5) if x = y or relationship between x and y cannot be determined.
1.
I. x^2 + x 20 = 0
II. y^2 + 13y + 40 = 0

2.
I. x^2 11x + 30 = 0
II. y^2 13y + 40 = 0
3.
I. x^2 + 10x + 25 = 0
II. 5y^2 2 15 y + 9 = 0
4.
I. 10x^2 29x 21 = 0
II. y^2 + 13y 68 = 0
Directions (Q. 57): What value should come in the place of question mark (?) in the following number
series?
5. 362, 452, 550, 656, ?
1) 770
2) 772
3) 670
4) 870
5) 790
6. 25, 28, 26, ?, 27, 30
1) 28
2) 32
3) 34
4) 29
5) 36
7. 9, 12, 30, 99, ?
1) 406
2) 418
3) 408
4) 416

5) 424
Directions (Q. 810): What value should come in the place of question mark (?) in the following
questions?
8. 168.781 112.412 8.409 1.150 = ?
1) 44.81
2) 46.81
3) 40.81
4) 47.81
5) 46.61
9. [(327) + 227) * (2727 + 2627)] 15927 = ?
1) 1431
2) 1331
3) 1421
4) 1441
5) 1231
10. 8.5% of 160 0.42% of 750 = ?
1) 11.45
2) 12.45
3) 13.45
4) 9.45
5) 10.45

1.2
I. x^2 + 5x 4x 20 = 0
(x 4) (x + 5) = 0
x = 4, 5
II. y^2 + 8y + 5y + 40 = 0
(y + 8) (y + 5) = 0
y = 8, 5
x >= y
2.5
I. x^2 6x 5x + 30 = 0
(x 5) (x 6) = 0
x = 5, 6
II. y^2 8y 5y + 40 = 0
(y 8) (y 5) = 0
y = 8, 5
No relationship between x and y exits.
3. 3;
I. (x + 5)^2 = 0
x = 5
II. (5y 3)^2 = 0
x = 3/5

4. 5
I. 10x^2 35x + 6x 21 = 0
(5x + 3) (2x 7) = 0
x = -3/5, 7/2
II. y^2 + 17y 4y 68 = 0
(y 4) (y + 17) = 0
y = 4, 17
No relationship between x and y exists.
5. 1;
6. 4
7. 3
8. 2
? = 46.81
9. 1
? = (3 + 27)(5327) - 15927
= 15927 + 53 * 27 - 15927 = 1431
10. 5
? = 8.5/100 * 160 - 0.42/100 * 750
= 13.6 - 3.15 = 10.45

bankersadda.com

http://www.bankersadda.com/2015/08/night-class-quant-quiz_19.html

Night Class: Quant Quiz


Directions : What value should come in the place of question mark (?) in the following questions?
1. 112 44096 268 = ? 9 + 241
1) 51
2) 45
3) 55
4) 42
5) 60

2. 81^2 49^2 = ? 26
1) 150
2) 145
3) 158
4) 160
5) 165
3. [(1382 + 3007) / (513 + 114)] of 4/7 + 161 = ?
1) 165
2) 160
3) 155
4) 169
5) 164
Directions: In each of these questions, two equations numbered I and II with variables x and y are given.
You have to solve both the equations to find the value of x and y. Give answer
1) if x > y
2) if x >= y
3) if x < y
4) if x <= y
5) if x = y or relationship between x and y cannot be determined.
4.
I. 18x^2 9x + 1 = 0
II. 48y^2 58y + 15 = 0
5.
I. x^2 5x 36 = 0
II. y^2 17y + 72 = 0
6.
I. 169x^2 = 49
II. 13y = sqroot(81) sqroot(4)
7.
I. x^2 21x + 104 = 0
II. y^2 29y + 210 = 0

Directions: Each question below is followed by two statements A and B. You have to determine whether
the data given in the statement is sufficient to answer the question. You should use the data and your
knowledge of mathematics to choose between the possible answers. Give answer:
1) if the statement A alone is sufficient to answer the question, but the statement B alone is not sufficient.
2) if the statement B alone is sufficient to answer the question, but the statement A alone is not sufficient.
3) if the both statement A and B together are needed to answer the question.
4) if either the statement A alone or B alone is sufficient to answer the question.
5) if you cannot get the answer from the statements A and B together, but needed even more data.
8. What is the number?
A. The sum of the two digits is 10. The ratio of the two digits is 4:1.
B. The product of two digits of a number is 16. The quotient of two digits is 4.
9. Rs. 4800 is divided amongst P, Q and R. Find the share of P.
A. P gets onethird of what Q and R together get.
B. Q get half as much as P and R together get.
10. What is the total number of votes polled?
A. The looser get 18% of the votes polled.
B. The winner get 82% of the votes and is elected by a majority of 1488 votes.

1. 1
121 * 8 - 268 = ? * 9 + 241
? = (700 - 241)/9 = 51
2. 4
? = [(81 + 49)(81 - 49)]/26 = 160
3. 1
? = (4389 / 627) * 4/7 + 161 = 165
4. 3;
I. (6x 1) (3x 1) = 0
x = 1/6, 1/3
II. (8y 3) (6y 5) = 0
y = 3/8,5/6
x<y
5. 5
I. (x + 4) (x 9) = 0
x = 4, 9
II. (y 8) (y 9) = 0
y = 8, 9
No relationship between x and y
6. 4:
I: x^2 = 49/169
x = +/- 7/13
y = 9-2/13 = 7/13
x <= y

7. 3;
I. (x 8) (x 13) = 0
x = 8, 13
II. (y 14) (y 15) = 0
y = 14, 15
x<y
8. 4;
Let the number be 10x + y.
From A, x + y = 10 and x/y = 4/1
5y = 10
y = 2, x = 8
From B, xy = 16 and x/y = 4
y=2;x=8
Either from (A) or (B) ; the number is 82.
9. 1;
P+Q+R=4
From A, P = 1/3 (Q + R)
3P = Q + R
4P = 4800
P = Rs. 1200
From B, Q = 1/2 (P+R)
2Q = P+R
Thus, we cant find the share of P from the statement B
10. 2
Let the total number of votes polled be x.
From statement B,
82% of x (100 82) % of x = 1488
or, (82% 18%) of x = 1488
64/100X = 1488
X = 1488 * 100/64 = 2325

bankersadda.com

http://www.bankersadda.com/2015/08/night-class-quant-quiz_18.html

Night Class: Quant Quiz


Directions: Study the following line graph and answer the following questions.
Percentage profit earned by two Companies over the given years.
% profit = (Income Expenditure)/Expenditure 100

1. Expenditure of company B in year 2013 and 2014 were in the ratio 13 : 7 respectively. What was the
respective ratio of their income?
1) 168 : 105
2) 171 : 105
3) 178 : 105
4) 165 : 106
5) 141 : 102
2. If the expenditure of companies A and B in year 2014 were in the ratio 7 : 11 and total expenditure in the
year 2014 of the both company was Rs. 90 lakhs, then what was the income of company B in that year?
1) Rs. 10735000
2) Rs. 11725000
3) Rs. 10835000
4) Rs. 10725000
5) Rs. 10625000
3. If the income of both companies in 2012 was equal. Find the ratio of their expenditure.
1) 181 : 172
2) 189 : 173
3) 188 : 173
4) 180 : 173
5) 189 : 172
4. If company A's income increases by 25% in 2014 as compared to 2012, but expenditure rises by 29%.
What will be percentage profit growth of A from last year?
1) 10%
2) 14%
3) 18%
4) 25%
5) Data inadequate

5. If the amount of profit remains the same over the years for company A, in which of the following years
the amount of expenditure is the minimum for company A?
1) 2015
2) 2011
3) 2012
4) 2014
5) 2013

Directions: In the following number series one number is wrong. Find out the wrong number.
6. 15, 69, 235, 939, 3755
1) 69
2) 939
3) 3755
4) 15
5) 235
7. 22, 23, 31, 58, 121
1) 22
2) 23
3) 31
4) 121
5) 58
8. 120, 220, 380, 740, 1400
1) 740
2) 220
3) 120
4) 380
5) 1400

Answers:
1. 3
E(B2013)/E(B2014) = 13/4
[I(B2013) - E(B2013)]/E(B2013) = 78/100
100 I(B2013) - 100 E(B2013) = 78 E (B2013)
I(B2013) = E(B2013) * 178/100
Similarly, I(B2014) = E(B2014) * 195/100
E(B2013)/E(B2014) = [100/178 I(B2013)]/[100/196 I(B2014)] = 13/7
I(B2013)/I(B2014) = 13/7 * 178/195 = 178/105
2. 4
E(A2014) : E(B2014) = 7 : 11
7x + 11x = Rs. 90,00,000
18x = Rs. 90,00,000
x = Rs. 5,00,000
E(B2014) = 11x = Rs. 55,00,000
I(B2014) = E(B2014) * 195/100
55,00,000 * 195/100 = 107,25,000

3. 5
E(A2012) = 100/172 I(A2012)
E(B2012) = 100/189 I(B2012)
Since, I(A2012) = I(B2012)
E(A2012) : E(B2012) = 189 : 172
4. 5
5. 1
I = E(1 + P/100)
I/E - 1 = P/100
E = 100(I-E)/P
(I E) is nothing but amount of profit 1. As (I E) will remain constant for over the years, the value of E will be
minimum in that year when the percentage profit is
maximum of company A i.e. 2015.
6. 1; The series is :
15 4 1 = 59 not equal to 69
59 4 1 = 235
235 4 1 = 939
939 4 1 = 3755
7. 4; The series is :
22 + 13 = 23
23 + 23 = 31
31 + 33 = 58
58 + 43 = 122 not equal to 121
8. 4; The series is :
120 2 20 = 220
220 2 40 = 400 not equal to 380
400 2 60 = 740
740 2 80 = 1400

bankersadda.com

http://www.bankersadda.com/2015/08/night-class-quant-quiz_17.html

Night Class: Quant Quiz


1. In how many ways 10 mangoes can be divided among 4 children?
1) 280
2) 286
3) 284
4) 278
5) 282

Directions: 7 boys and 7 girls sit in a row randomly. Find the probability that
2. The girls sit together.
1) 1/429
2) 2/429
3)3/421
4) 1/421
5) 1/427
3. The boys and girls sit alternatively.
1) 1/1715
2) 2/1715
3) 8/1715
4) 5/1714
5) 1/1716
4. What will be the length of the diagonal of a square park whose area is equal to the area of a rectangular
park of length 160 m and breadth 80 m?
1) 180 m
2) 185 m
3) 172 m
4) 175 m
5) 160 m
5. A train left from station A for station B at 4 pm at an average speed of 80 km/h. At 8 pm, another train
also had left from station A for station B on a parallel track at an average speed of 120 km/h. How far from
A, is the another train expected to overtake the first train?
1) 900 km
2) 940 km
3) 960 km
4) 980 km
5) 920 km
6. 8 men can complete a piece of work in 12 days. 10 women can do the same piece of work in 15 days
where as 18 children can complete in 10 days. 8 men, 25 women and 30 children work together for 2 days.
If only men were to complete the remaining work in 1 day, how many men would be required totally?
1) 25
2) 16
3) 18
4) 30

5) 20
7. A man buys a T.V valued at Rs. 36000. He pays Rs. 12000 at once and rest 18 months later on which he
is charged simple interest at the rate of 6% p.a. Find the total amount he pays for t.v.
1) Rs. 38180
2) Rs. 38460
3) Rs. 38860
4) Rs. 38160
5) Rs. 38260
8. A shopkeeper sells note book at the rate of Rs. 65 each and earns a commission of 5%. He also sells
pen stand at Rs. 90 each and earns a commission of 15%. How much commission will he earn in two
weeks, if he sells 12 note books and 8 pen stands a day?
1) Rs. 2058
2) Rs. 2158
3) Rs. 2258
4) Rs. 2048
5) Rs. 2068
9. Two partners invest Rs. 1,20,000 and Rs. 80,000 respectively in a business and agree that 30% of the
profit should be divided between them and the remaining profit is to be treated as interest on capital. If
one partner gets Rs. 1860 more than the other, find the total profit made in the business.
1) Rs. 31000
2) Rs. 30000
3) Rs. 28000
4) Rs. 32000
5) Rs. 29000
10. A trader has 130 kg of pulses, part of which he sells at 12% profit and the rest at 25% profit. He gains
18% on whole. What is the quantity sold at 25% profit?
1) 65 kg
2) 60 kg
3) 70 kg
4) 64 kg
5) 68 kg

1. 2; Required number of ways


= n + r 1 Cr 1
= 10 + 4 1 C41
= 13C3
= 13! / 3! 10!
= (13 * 12 * 11)/ (3*2)
= 286
2. 1;
When 7 girls sit together, regarding them as one group
(7 boys + 1 group of girls) = 8 objects.
Required probability = 8! * 7!/14!
= 1/429
3. 5;
If we begin with boy, then 7 boys can sit in 7 odd places in 7! ways and 7 girls in 7 even places in 7! ways,

Therefore total ways = 7! 7!. Similarly, if we begin with girls then 7! 7!


Required probability = 2 * 7! * 7!/ 14!
= 1/1716
4. 5
1/2 * Diagonal^2 = 160*80
D = sqroot(2*160*80) = 160m
5. 3
Distance travelled by first train in 4 h = 80 4 = 320 km
Relative speed = (120 80) = 40 km/hr
Let x be the time taken by the second train to overtake
the first train.
Then, 40 = 320/x
x = 8h
Distance from A = 120 * 8 = 960km
6. 2; (8 12) men = (10 15) women = (18 10) children
16 men = 25 women = 30 children
Now, 8M + 25W + 30C = 8M + 16M + 16M = 40 men
8 mens 1 day work = 1/12
40 men's 1 day work = 1/12 * 40/8 = 5/12
40 men's 2 day work = 5/12 * 2 = 5/6
Remaining Work = 1 - 5/6 = 1/6
Remaining work will be completed by 40 men in 2/5 days.
To complete in one day man required = 40 * 2/5 = 16 men
7. 4
Interest charge on (36000 12000 = 24000)
= (24000 * 18 * 6)/(100 * 12) = 2160
The total amount paid for T.V
= 12000 + 24000 + 2160 = Rs. 38,160
8. 1;
12 notebooks in a day, then in two weeks (i.e. 14 days)
= 14 12 = 168 notebook
Commission earned = 65 168 5/100 = Rs. 1546
8 pen stands in a day, then in two weeks (i.e. 14 days)
= 8 14 = 112 pen stands
Commission earned = 112 90 15/100 = Rs. 1512
Total commission earned = 546 + 1512 = Rs. 2058
9. 1; The ratio of profit = 12 : 8 = 3 : 2
Total Profit = 1860 (100/30)(3+2/3-2)
= 1860 * 100/30 * 5 = Rs. 31000
10. 2
The quantity sold at 25% profit

Where, N = total quantity, P = % gain on whole


x = part sold at x% profit
y = part sold at y% profit
= (p-x/y-x)N
= (18-12)/(25-12) * 130
6/13 * 130 = 60kg

bankersadda.com

http://www.bankersadda.com/2015/08/night-class-quant-quiz_17.html

Night Class: Quant Quiz


1. In how many ways 10 mangoes can be divided among 4 children?
1) 280
2) 286
3) 284
4) 278
5) 282

Directions: 7 boys and 7 girls sit in a row randomly. Find the probability that
2. The girls sit together.
1) 1/429
2) 2/429
3)3/421
4) 1/421
5) 1/427
3. The boys and girls sit alternatively.
1) 1/1715
2) 2/1715
3) 8/1715
4) 5/1714
5) 1/1716
4. What will be the length of the diagonal of a square park whose area is equal to the area of a rectangular
park of length 160 m and breadth 80 m?
1) 180 m
2) 185 m
3) 172 m
4) 175 m
5) 160 m
5. A train left from station A for station B at 4 pm at an average speed of 80 km/h. At 8 pm, another train
also had left from station A for station B on a parallel track at an average speed of 120 km/h. How far from
A, is the another train expected to overtake the first train?
1) 900 km
2) 940 km
3) 960 km
4) 980 km
5) 920 km
6. 8 men can complete a piece of work in 12 days. 10 women can do the same piece of work in 15 days
where as 18 children can complete in 10 days. 8 men, 25 women and 30 children work together for 2 days.
If only men were to complete the remaining work in 1 day, how many men would be required totally?
1) 25
2) 16
3) 18
4) 30

5) 20
7. A man buys a T.V valued at Rs. 36000. He pays Rs. 12000 at once and rest 18 months later on which he
is charged simple interest at the rate of 6% p.a. Find the total amount he pays for t.v.
1) Rs. 38180
2) Rs. 38460
3) Rs. 38860
4) Rs. 38160
5) Rs. 38260
8. A shopkeeper sells note book at the rate of Rs. 65 each and earns a commission of 5%. He also sells
pen stand at Rs. 90 each and earns a commission of 15%. How much commission will he earn in two
weeks, if he sells 12 note books and 8 pen stands a day?
1) Rs. 2058
2) Rs. 2158
3) Rs. 2258
4) Rs. 2048
5) Rs. 2068
9. Two partners invest Rs. 1,20,000 and Rs. 80,000 respectively in a business and agree that 30% of the
profit should be divided between them and the remaining profit is to be treated as interest on capital. If
one partner gets Rs. 1860 more than the other, find the total profit made in the business.
1) Rs. 31000
2) Rs. 30000
3) Rs. 28000
4) Rs. 32000
5) Rs. 29000
10. A trader has 130 kg of pulses, part of which he sells at 12% profit and the rest at 25% profit. He gains
18% on whole. What is the quantity sold at 25% profit?
1) 65 kg
2) 60 kg
3) 70 kg
4) 64 kg
5) 68 kg

1. 2; Required number of ways


= n + r 1 Cr 1
= 10 + 4 1 C41
= 13C3
= 13! / 3! 10!
= (13 * 12 * 11)/ (3*2)
= 286
2. 1;
When 7 girls sit together, regarding them as one group
(7 boys + 1 group of girls) = 8 objects.
Required probability = 8! * 7!/14!
= 1/429
3. 5;
If we begin with boy, then 7 boys can sit in 7 odd places in 7! ways and 7 girls in 7 even places in 7! ways,

Therefore total ways = 7! 7!. Similarly, if we begin with girls then 7! 7!


Required probability = 2 * 7! * 7!/ 14!
= 1/1716
4. 5
1/2 * Diagonal^2 = 160*80
D = sqroot(2*160*80) = 160m
5. 3
Distance travelled by first train in 4 h = 80 4 = 320 km
Relative speed = (120 80) = 40 km/hr
Let x be the time taken by the second train to overtake
the first train.
Then, 40 = 320/x
x = 8h
Distance from A = 120 * 8 = 960km
6. 2; (8 12) men = (10 15) women = (18 10) children
16 men = 25 women = 30 children
Now, 8M + 25W + 30C = 8M + 16M + 16M = 40 men
8 mens 1 day work = 1/12
40 men's 1 day work = 1/12 * 40/8 = 5/12
40 men's 2 day work = 5/12 * 2 = 5/6
Remaining Work = 1 - 5/6 = 1/6
Remaining work will be completed by 40 men in 2/5 days.
To complete in one day man required = 40 * 2/5 = 16 men
7. 4
Interest charge on (36000 12000 = 24000)
= (24000 * 18 * 6)/(100 * 12) = 2160
The total amount paid for T.V
= 12000 + 24000 + 2160 = Rs. 38,160
8. 1;
12 notebooks in a day, then in two weeks (i.e. 14 days)
= 14 12 = 168 notebook
Commission earned = 65 168 5/100 = Rs. 1546
8 pen stands in a day, then in two weeks (i.e. 14 days)
= 8 14 = 112 pen stands
Commission earned = 112 90 15/100 = Rs. 1512
Total commission earned = 546 + 1512 = Rs. 2058
9. 1; The ratio of profit = 12 : 8 = 3 : 2
Total Profit = 1860 (100/30)(3+2/3-2)
= 1860 * 100/30 * 5 = Rs. 31000
10. 2
The quantity sold at 25% profit

Where, N = total quantity, P = % gain on whole


x = part sold at x% profit
y = part sold at y% profit
= (p-x/y-x)N
= (18-12)/(25-12) * 130
6/13 * 130 = 60kg

bankersadda.com

http://www.bankersadda.com/2015/08/night-class-quant-quiz_13.html

Night Class: Quant Quiz


1. Rahul purchased a house for Rs. 10 lakh. He sold it at 6% loss. With that money he again purchased another
house and sold it at a profit of 12%. What is his overall loss/profit?
1) Rs. 54, 600 profit
2) Rs. 51, 800 loss
3) Rs. 52, 800 profit
4) Rs. 51, 900 loss
5) Rs. 52, 600 loss

Directions: What approximate value should come in the place of question mark (?) in the following questions?
2. 28. 948 + 37.008 + 46.998 195.99 784.01 = ?
1) 71
2) 79
3) 85
4) 55
5) None of these
3. (31.98)^2 (35.02)^2 + (19.03)^2 = ?^2 + sqroot(255)
1) +/- 13
2) +/- 18
3) +/- 9
4) +/- 15
5) +/- 12
Directions: What value should come in the place of question mark (?) in the following questions?
4. (779 + 553)/ 33. 3 + 528 = ?/9
1) 5012
2) 5112
3) 5462
4) 5222
5) 5812
5. 71 1596 57 4 = ?
1) 62
2) 54
3) 68
4) 64
5) 66
6. (3496 1002) 43 12 = 456 + ?
1) 220
2) 260
3) 240
4) 250
5) 280
Directions: What value should come in the place of question mark (?) in the following number series?

7. 4, 16, 52, 160, ?


1) 472
2) 474
3) 482
4) 468
5) 484
8. 9, 10, 24, 81, ?
1) 340
2) 320
3) 345
4) 360
5) 348
9. 11, 6, 10.5, 28.75, ?
1) 108.125
2) 100.125
3) 104
4) 104.125
5) 108
1.3; SP of house = 10,00,000 94100
= Rs. 9,40,000
Now, CP of another house = Rs. 9,40,000
And SP of another house = 9,40,000 100
112
= Rs. 10,52,800
Overall profit = Rs. 10,52,800 Rs. 10,00,000 = Rs. 52,800
2. 5
3. 5;
4. 2;
5. 4;
6.3;
7. 5
8. 1
9. 4

bankersadda.com

http://www.bankersadda.com/2015/08/night-class-quant-quiz_12.html

Night Class: Quant Quiz


1. The ages of Anjali and Renu are in the ratio of 6 : 7 respectively. After 6 years, the ratio of their ages
will be 15 : 17. What is the age of Renu ? (In years)
1) 28
2) 24
3) 32
4) 42
5) None of these

2. If the length and breadth of a rectangle are changed by +20% and 10% respectively. What is the %
change in area of rectangle ?
1) 8% decrease
2) 12 % increase
3) 8 % increase
4) 4 % increase
5) None of these
3. Ram purchased a computer for Rs. 47000. He had to sell it for Rs. 45800. Find his loss % ?
1) 3 %
2) 4 13/29 %
3) 2 4/9 %
4) 2 26/47%
5) None of these
4. A can complete a piece of work in 12 days. A and B together can complete the same piece of work in 8
days. In how many days can B alone complete the same piece of work ?
1) 18
2) 16
3) 22
4) 17
5) None of these
5. The compound interest on Rs. 10000 at 20 % p.a. in 4 years is ? (In Rs)
1) Rs. 7280
2) Rs. 10736
3) Rs. 20736
4) Rs. 14142
5) None of these
6. Rs. 12000 amounts to Rs. 20736 in 3 years at r % p.a. of compound interest. What is the value of r ?
1) 20 %
2) 12 %
3) 25 %
4) 18 %
5) None of these
7. In how many different ways can the letters of the word 'MIRACLE' be arranged ?
1) 40320

2) 5040
3) 7080
4) 720
5) None of these
8. In an election between two candidates one got 72% of total valid votes. 25% of the total votes are
invalid. If the total votes are 8200, what is the number of valid votes the other candidate got ?
1) 2050
2) 1876
3) 4324
4) 1722
5) None of these
9. Shweta when increases her speed from 25 km/hr to 30 km/hr. She takes one hours less than the usual
time to cover a certain distance. What is the distance usually covered by Shweta ? (in km)
1) 150 km
2) 120 km
3) 90 km
4) 180 km
5) None of these
10. The sphere has the same curved surface as a cone of slant height 16 cm and base radius 4 cm. Find
the radius in cm ?
1) 4
2) 6
3) 12
4) 10
5) None of these

1. 1
(6x + 6)/(7x + 6) = 15/17
2. 3
1 * b = Area
1*1=1
After changes, 1.2 * 0.9 = 1.08
There is 8% increase in the area.
3. 4
Loss% = (47000 - 45800)/47000 * 100
4. 5
Req no. of days - 1/ (1/8 - 1/12)
5. 2
Req CI = 10000 * (1.2)^4 - 10000
10000 * (2.0736 - 1)
6. 1
(20736/12000) = (1 + R/100)^3

7. 2
Required = 7P7
8. 4
No. of valid votes other candidate got = (100 - 72)/100 * (8200 - 8200*25/100)
9. 1
25 * t = 30 * (t-1) = Distance
10. 1

4 * 22/7 * r^2 = 22/7 * 4 * 16

bankersadda.com

http://www.bankersadda.com/2015/09/night-class-quant-quiz_16.html

Night Class: Quant Quiz


Direction : Following line graph shows the ratio of imports to exports of two companies over the years :

1. In how many of the given years were the imports more than the exports in case of company - A ?
1) 5
2) 3
3) 1
4) 2
5) 4

2. If the imports of company-B in year 2011 were ` 51.688 lakh, then what were the exports of company-B
in the same year?
1) 64.26 lakh
2) 67.54 lakh
3) 71.36 lakh
4) 73.84 lakh
5) None of these
3. If the exports of company-A and Company-B were equal to 84 lakh in year 2012 then what will be the
difference between imports of company-B and imports of company-A in that year ?
1) 9.6 lakh
2) 8.4 lakh
3) 7.2 lakh
4) 6.8 lakh
5) 5.4 lakh
4. If the exports of company-A in year 2009 and exports of company-B in year 2012 were equal then the
imports of company- B in 2012 is approximately what percentage of imports of company-A in 2009 ?
1) 60%
2) 75%
3) 84%
4) 96%
5) 133.3%
5. In year 2009, If the export of company-B is increased by 100% and import is increased by 200%. Then

what will be the new ratio of import to export of company-B in that year ?
1) 0.8
2) 1.0
3) 1.2
4) 0.6
5) 1.5
6. A cycle is sold at a profit of 15%. If both the cost price and selling price is decreased by Rs. 200 the
profit would be 5% more what is the original cost price of cycle ?
1) Rs. 1200
2) Rs. 1000
3) Rs. 800
4) Rs. 600
5) None of these
7. A person lends Rs. 4200 to 'A' for 3 years and Rs. 6500 to 'B' for 7 years. If the person gets Rs. 8134 as
interest then what is the rate of interest per annum ?
1) 6%
2) 8%
3) 10%
4) 12%
5) 14%
8. A sum amounts to Rs. 98010 in 2 years and to Rs. 107811 in 3 years compounded annually. What is the
sum ?
1) Rs. 81000
2) Rs. 84000
3) Rs. 87000
4) Rs. 90000
5) Rs. 94000
9. How many different words can be formed with the letters of the word ''BREAKING". So that the words
begin with 'B' and end with 'G' ?
1) 120
2) 720
3) 5040
4) 2520
5) 1440
10. A bag contains 9 white and 7 black balls. Three balls are drawn at random. Find the probability that all
the three balls are black ?
1) 1/8
2) 1/12
3) 1/16
4) 1/21
5) None of these
1. 4
Imports > Exports
Import/ Exports > 1
2. 4
I/E = 0.7
E = I/0.7 = 51.688/0.7 = 73.84 lakh

3. 2
Ia/Ea = 0.8
Ia = 0.8 * 84 = 67.2 lakh
Ib/Eb = 0.9
Ib = 0.9 * 84 = 75.6 lakh
Diff = 75.6 - 67.2 = 8.4 lakh
4. 2
Ia/Ea = 1.2
Ib/Eb = 0.9
Let export of A and B are X
Ia = 1.2x
Ib = 0.9x
Req% = 0.9x/1.2x * 100 = 75%
5. 3
I/E = 0.8
I1 = I + 200I/100 = 3I
E1 = E + 100E/100 = 2E
Ratio = I1/ E1 = 3I/2E = 3/2 * 0.8 = 1.2
6. 3;
Let the original cost price is x, so selling price is
115 x/100 = 23 x/ 20
Now CP = x 200,
S.P. = 23x/20 - 200
Profit = 23x/20 200 (x 200) = 23x/20 x = 3x/20
% profit = 3x/20 * 1/(x - 200) * 100 = 20
3x/(x - 200) = 4
3 x = 4 x 800
x = 800
7. 5;
Let the rate of interest is r% p.a.
4200 * r * 3/100 + 6500 * r * 7/100 = 8134
126 r + 455 r = 8134
r = 8134/581 = 14% pa
8. 1
Let the principal is P and rate of interest is r
P (1 + R/100)^3 = 107811
P (1 + R/100) ^2 = 98010
Dividing eq I by II
1 + R/100 = 107811/98010
R/100 = 9801/98010
R = 980100/98010 = 10% pa

P = (1 + 10/100)^2 = 98010
p = 98010 * 100/121 = 81000
9. 2
Now 6 letters are left for six different place
Req No. of wages = 6P6 = 6! = 720
10. 3
n(S) = 16C3 = 560
n (E) = Number of ways of selecting 3 balls out of 7 =
7C3 = 7!/3!4! = 35
P(E) = 35/560 = 1/16

bankersadda.com

http://www.bankersadda.com/2015/08/night-class-quant-quiz_11.html

Night Class: Quant Quiz


1. 1728 / cuberoot(262144) * ? 288 = 4491
1) 164
2) 167
3) 173
4) 187
5) None of these
2. (47 588) (28 120) = ?
1) 8.225
2) 8.160
3) 7.5
4) 7.115
5) None of these
3. 45 % of 224 (?) % of 120 = 8104.32
1) 63
2) 87
3) 103
4) 67
5) None of these
4. 11960 ? = 65 23 2
1) 8
2) 3
3) 6
4) 4
5) None of these
5. 6 1/6 + 16 1/5 = ?
1) 22 11/30
2) 22 2/5
3) 222/11
4) 22 13/30
5) None of these
Directions: What should come in the place of question mark(?) in the following numbers series :
6. 14, 12, 21, 59, 231, ?
1) 812
2) 979
3) 1149
4) 1098
5) None of these
7. 121, 144, 190, 259, 351, ?
1) 466
2) 486
3) 473
4) 484

5) None of these
8. 12, 12, 18, 45, 180, 1170, ?
1) 9475
2) 11522
3) 11425
4) 12285
5) None of these
9. 380, 465, 557, 656, 762, 875, ?
1) 975
2) 968
3) 987
4) 990
5) None of these
10. 16, 24, 36, 54, ?, 121.5
1) 86
2) 81
3) 79
4) 92
5) None of these
1. 5
2. 1
3. 4
4. 4
5. 1
6. 3
7. 1
8. 4
9. 5
10. 2

bankersadda.com

http://www.bankersadda.com/2015/08/night-class-quant-quiz_10.html

Night Class: Quant Quiz


Directions: What should come in place of question mark (?) in the following questions:
1. 4 + 4.44 + 0.4 + 44.04 + 444 = ?
1) 486.48
2) 496.88
3) 486.88
4) 496.84
5) None of these
2. (?)^2 + (65)^2 = (160)^2 (90)^2 7191
1) 72
2) 68
3) 6084
4) 78
5) 82
3. 0.07 % of 1250 0.02 % of 650 = ?
1) 0.745
2) 0.875
3) 0.545
4) 0.695
5) None of these
4. sqroot(6.25) / 0.5 = ?/10
1) 5
2) 500
3) 50
4) 2.5
5) None of these
5. 1/8 * (223 + ?) = 73
1) 361
2) 371
3) 341
4) 391
5) None of these
6. cuberoot(12167) * sqroot(16384) = ? * 11.5
1) 204
2) 234
3) 286
4) 256
5) None of these
7. [18 * 14 - 6 * 8] / [488 / 4 - 20] = ?
1) 3
2) 2.5
3) 4
4) 8
5) None of these

8. sqroot(7225) * 1/5 + (45)^2 = ?


1) 2058
2) 2042
3) 2040
4) 2038
5) None of these
9. 25.6 % of 250 + sqroot(?) =119
1) 3125
2) 3025
3) 55
4) 65
5) 4225
10. 7428 * 3/4 * 2/9 * ? = 619
1) 0.2
2) 0.8
3) 0.5
4) 2.4
5) 1.5
Answers:
1. 2
2. 4
3. 1
4. 3
5. 1
6. 4
7. 5
8. 2
9. 2
10. 3

bankersadda.com

http://www.bankersadda.com/2015/08/night-class-quant-quiz_10.html

Night Class: Quant Quiz


Directions: What should come in place of question mark (?) in the following questions:
1. 4 + 4.44 + 0.4 + 44.04 + 444 = ?
1) 486.48
2) 496.88
3) 486.88
4) 496.84
5) None of these
2. (?)^2 + (65)^2 = (160)^2 (90)^2 7191
1) 72
2) 68
3) 6084
4) 78
5) 82
3. 0.07 % of 1250 0.02 % of 650 = ?
1) 0.745
2) 0.875
3) 0.545
4) 0.695
5) None of these
4. sqroot(6.25) / 0.5 = ?/10
1) 5
2) 500
3) 50
4) 2.5
5) None of these
5. 1/8 * (223 + ?) = 73
1) 361
2) 371
3) 341
4) 391
5) None of these
6. cuberoot(12167) * sqroot(16384) = ? * 11.5
1) 204
2) 234
3) 286
4) 256
5) None of these
7. [18 * 14 - 6 * 8] / [488 / 4 - 20] = ?
1) 3
2) 2.5
3) 4
4) 8
5) None of these

8. sqroot(7225) * 1/5 + (45)^2 = ?


1) 2058
2) 2042
3) 2040
4) 2038
5) None of these
9. 25.6 % of 250 + sqroot(?) =119
1) 3125
2) 3025
3) 55
4) 65
5) 4225
10. 7428 * 3/4 * 2/9 * ? = 619
1) 0.2
2) 0.8
3) 0.5
4) 2.4
5) 1.5
Answers:
1. 2
2. 4
3. 1
4. 3
5. 1
6. 4
7. 5
8. 2
9. 2
10. 3

bankersadda.com

http://www.bankersadda.com/2015/08/night-class-quant-quiz_6.html

Night Class: Quant Quiz


1. A tradesman marks his goods 40% above the cost price. If he allows a discount of 25%, then his gain %
is1) 5%
2) 7 1/2%
3) 12 1/2%
4) 6%
5) None of these

2. 10 kg of superior quality of sugar is mixed with 50 of inferior quality of Sugar. The price of Superior
quality and inferior quality of sugar is `18 and `12 respectively. The average price per kg of mixture is 1) Rs. 13
2) Rs. 15
3) Rs. 14
4) Rs. 16
5) None of these
3. Salaries of A, B and C are in the ratio 1 : 2 : 3. Salary of B and C together is ` 9000. By what % is salary
of C more than that of A?
1) 100
2) 150
3) 125
4) 200
5) None of these
4. A contractor employed 50 men to complete the project in 100 days. But later on he realised that just
after 25 days only 20% of the work had been completed. How many extra days, than the scheduled time
are required?
1) 25
2) 30
3) 22
4) 28
5) None of these
5. The base radius of a cylinder is 7 cm and its height is 40 cm. Find curved surface area. (in cm2)
1) 2460
2) 1760
3) 1640
4) 1720
5) 1580
6. If A is 90% of B. What % of A is B.
1) 101.1%
2) 190%
3) 111.1%
4) 119%
5) None of these

7. A student has to secure 40% marks to pass. He gets 144 marks and fails by 16 marks. The maximum
marks are1) 400
2) 360
3) 300
4) 480
5) 500
8. The difference between simple and compound interest on a sum of money at 10% per annum is Rs. 40.
What is the sum?
1) Rs. 5000
2) Rs. 12000
3) Rs. 8000
4) Data insufficient
5) None of these
9. A sum of money becomes 3/2 of itself in 5 years at a certain rate of simple interest. Find the rate of
interest.
1) 10%
2) 12.5%
3) 15%
4) 8%
5) None of these
10. Anjali goes to office at a speed of 12 km/h and returns to her home at a speed of 6 km/h. If she takes 8
hours in all, what is the distance between her office and her home?
1) 36 km
2) 32 km
3) 56 km
4) 24 km
5) None of these
1. 1
2. 2
3. 4
4. 1
5. 2
6. 3
7. 1
8. 4
9. 1
10. 2

bankersadda.com

http://www.bankersadda.com/2015/08/night-class-quant-quiz_5.html

Night Class: Quant Quiz


1/6 of 4/11 of 1/13 of 1/12 of 164736 = ?
1) 48
2) 32
3) 72
4) 64
5) None of these
2. 64% of 275 14% of ? = 113
1) 550
2) 475
3) 450
4) 515
5) 525
3. (12)^2 + (13)^2 = (?)^2 + 24
1) 17
2) 19
3) 15
4) 16
5) None of these
4. 15625 5 5 5 2 = ?
1) 250
2) 1250
3) 50
4) 600 5) None of these
5. 7 1/2 + 8 1/4 + 6 2/3 = ?
1) 21 5/2
2) 22 5/12
3) 21 1/4
4) 22 7/12
5) 22 1/4
Directions: What should come in the place of question mark(?) in the following numbers series?
6. 17, 22, 37, 72, 137, ?
1) 242
2) 202
3) 212
4) 232
5) None of these
7. 0, 4, 13, 29, 54, ?
1) 93
2) 103
3) 90
4) 98
5) None of these

8. 4096, 1024, 266, 86.5, ?


1) 48.25
2) 44.775
3) 46.625
4) 45.525
5) None of these
9. 20, 120, 264, 460, ?, 1040
1) 686
2) 716
3) 724
4) 698
5) None of these
10. 4, 35, 252, ?, 12404
1) 1771
2) 1653
3) 1687
4) 1724
5) 1684

1. 4
2. 3
3. 1
4. 1
5. 2

bankersadda.com

Night Class: Quant Quiz


1. 243.67 + 420.72 + 567.88 602.32 = ?
1) 631.65
2) 629.95
3) 637. 67
4) 641.85
5) None of these
2. (57)^2 + (12)^3 = (?)^2 + 77
1) 60
2) 67
3) 70
4) 80
5) None of these
3. 128% of 375 116% of 325 = ?
1) 103
2) 107
3) 123
4) 117
5) None of these
4. (144)^2 (1728)^2 (20736)^(1/4) = (12)^?
1) 11
2) 14
3) 13
4) 9
5) None of these
5. 212% of 30 + 25% of 862 = ? % of 1000
1) 27.64
2) 41.71
3) 31.27
4) 27.91
5) None of these
6. 999 1001 = ? + 26
1) 99973
2) 999999
3) 999963
4) 99983
5) None of these
7. 23.82 7.5 24.72 = ?
1) 172 .68
2) 162.73
3) 153.93
4) 181.77
5) None of these
8. (7921)^(1/2) + (14641)^(1/4) = (?)^2

http://www.bankersadda.com/2015/08/night-class-quant-quiz.html

1) 10
2) 12
3) 20
4) 14
5) None of these
9. sqroot(2116) + {cuberoot(19683)}/13.5 = ? 6
1) 12
2) 8
3) 14
4) 6
5) None of these
10. ? 26 28 = 728
1) 576
2) 336
3) 544
4) 676
5) 654

Answers:
1. 2
2. 3
3. 1
4. 1
5. 4
6. 5
7. 3
8. 1
9. 2
10. 4

bankersadda.com

http://www.bankersadda.com/2015/07/night-class-quant-quiz_30.html

Night Class: Quant Quiz


1. Find remainder of (7^99)/6
1) 2
2) 1
3) 4
4) 3
5) None of these

2. What is the greatest number of 4 digits that when divided by any of the numbers 6, 9, 12, 17 leaves a
remainder of 2?
1) 9792
2) 9982
3) 9794
4) 9906
5) None of these
3. In a fraction, numerator is increased by 20% and the denominator is diminished by 30%. The new
fraction obtained is 2/3. The original fraction is:
1) 7/18
2) 5/9
3) 6/11
4) 3/5
5) None of these
4. Find a single discount equivalent to a discount series of 25%, 20%, 10%1) 23%
2) 46%
3) 48%
4) 55%
5) 50%
5. By selling pen for ` 81, a shopkeeper gain 35%. At what price should he sell the pen to gain 20% on the
cost price?
1) Rs. 72
2) Rs. 66
3) Rs. 92
4) Rs. 76
5) None of these
Answers
1. 2
2. 3
LCM of 6, 9, 12, 17 = 612
Greatest Number of 4 digit divisible by 612 is 9792, to get remainder 2 number should be = 9792 + 2 = 9794
3. 1

4. 2
5. 1

bankersadda.com

http://www.bankersadda.com/2015/07/night-class-quant-quiz_29.html

Night Class: Quant Quiz


1. The price of an article is cut by 25%. To restore it to its original price, the new price must be increased
by1) 20%
2) 40%
3) 33 1/3 %
4) 37 1/3 %
5) None of these

2. 40% of a number when subtracted from 112, given the number itself. Find the number1) 120
2) 80
3) 150
4) 90
5) None of these
3. Find the difference between the simple and the compound interest at 5% per annum for 2 years on a
principal of Rs. 4000.
1) Rs. 8
2) Rs. 10
3) Rs. 25
4) Rs. 20
5) None of these
4. How many number of 3 digit can be formed with the digits 4, 5, 6, 7, 8 (repetition of digits not allowed)?
1) 60
2) 12
3) 240
4) 48
5) None of these
5. Mohan and Sohan travel the same distance at the rate of 4 km/hr and 6 km/hr respectively. If Mohan
takes 30 minutes longer than Sohan, the distance travelled by each is1) 8 km.
2) 6 km
3) 18 km
4) 30 km
5) None of these
6. A bag contains 4 black and 6 red balls. If three balls from the bag are chosen at random, what is the
chance that they are all red?
1) 1/6
2) 2/23
3) 1/4
4) 21/23
5) 1/9
7. The average of 40 numbers is 36. If two numbers, namely 50 and 60 are discarded, the average of the

remaining numbers is1) 34


2) 33.5
3) 35
4) 32.5
5) 34.5
8. A sum of Rs. 800 amounts to 960 in 4 years at simple interest. What will it amount to if the rate of
interest is increased by 2%?
1) Rs. 864
2) Rs. 932
3) Rs. 908
4) Rs. 1024
5) None of these
9. Pepsi and coke, there are two companies, selling the packs of cold-drinks. For the same selling price
pepsi gives two successive discounts of 10% and 20%. While coke sells it by giving two successive
discounts of 5% and 25%. What is the ratio of their marked price?
1) 95 : 96
2) 136 : 135
3) 19 : 11
4) 73 : 77
5) None of these
10. If 20 men can do a piece of work in 24 days working 5 hours a day, then how many men are required
to complete the work working 8 hours a day in 10 days?
1) 30
2) 40
3) 35
4) 36
5) None of these
Answers:
1. 3
2. 2
3. 2
4. 1
5. 2
6. 1
7. 3
8. 4
9. 1
10. 1

bankersadda.com

http://www.bankersadda.com/2015/07/night-class-quant-quiz_28.html

Night Class: Quant Quiz


Directions: Read the following information to answer the questions.
In a sports event there are three categories of race (100 m, 200 m, 400 m). Total 200 athletes participated
in that event. The number of athletes who participated only in 100m race is 30% of total number of
athletes, and among them 1/3rd are females. Number of athletes who participated in 200m race only is
15% of total number of athletes and among them 40% are females. Number of athletes who participated
only in 400m race is 1/4 of total number of athletes and among them half are females. Number of athletes
who participated in 100m and 200m race but not in 400m race is 1/10 of total number of athletes and
among them 1/4 are females. Number of athletes who participated in 100m and 400 m race but not in 200
m is 7.5% of total number of athletes and among them 8/15 are females. Number of athletes who
participated in all three categories is 1/20 of total number of athletes and among them 1/5 are females.
Number of female atheletes who participate 200m and 400 m race but not in 100m race is 8/15.

1. What is the number of female athletes who participated in exactly two categories of race?
1) 20
2) 21
3) 23
4) 24
5) 25
2. What is the difference between the total number of male athletes and the number of female athletes
who participated in exactly one category?
1) 61
2) 63
3) 65
4) 67
5) 69
3. What is the ratio of the total number of athletes who participated in 200m and 400m race but not in
100m race to the male athletes among them?
1) 15 : 11
2) 15 : 8
3) 15 : 7
4) 15 : 13
5) 8 : 7
4. What is the number of male athletes who participated in at most two categories of race?
1) 104
2) 106
3) 108
4) 110
5) 112
5. The number of male athletes who participated in all three categories of race is what percentage of total
number of female athletes?
1) 10%
2) 20%

3) 30%
4) 40%
5) None of these
Directions: What approximate value should come in the place of question mark (?) in the
following questions?
6. (72)^2 2.85
1) 12825
2) 13625
3) 14775
4) 15625
5) 16875
7. (475% of 884) 35 = ?
1) 116
2) 120
3) 124
4) 127
5) 132
8. (0.000428 0.00040) 55.05 = ?
1) 53
2) 56
3) 59
4) 63
5) 73
9. 1/(4)^2 * 175.956 + 1/(2)^3 * 351.59 - 4.989 = ?
1) 45
2) 50
3) 55
4) 60
5) 65
10. cube root(10650) = ?
1) 22
2) 26
3) 18
4) 28
5) 24

1. 2
Females = 5 + 8 + 8 = 21
2. 2;
Total males = 120, Females (one cate) = 20 + 12 + 25 = 57
Diff = 120 - 57 = 63
3. 3;
M + F = 15, F = 8

M = 7,
Ratio = 15 : 7
4. 5;
Total males = 40 + 18 + 25 + 15 + 7 + 7 = 112
5. 1;
Female(total) = 80,
Male (alt-3 cate) = 10 - 2 = 8
% = 8/80 * 100 = 10%
6. 3
7. 2
8. 3
9. 2
10. 1

bankersadda.com

http://www.bankersadda.com/2015/07/night-class-quant-quiz_25.html

Night Class: Quant Quiz


Directions : Each of the questions below consists of a question and two statements numbered I and II
given below it. You have to decide whether the data provided in the statements are sufficient to answer
the question. Read both the statements and Give answer
1) if the data in Statement I alone is sufficient to answer the question, while the data in Statement II alone
is not sufficient to answer the question.
2) if the data in Statement II alone is sufficient to answer the question, while the data in Statement I alone
is not sufficient to answer the question.
3) if the data either in Statement I alone or in Statement II alone is sufficient to answer the question.
4) if the data in both the Statements I and II even together are not sufficient to answer the question.
5) if the data in both the Statements I and II together are necessary to answer the question.

1. What is the area of a square?


I. The measure of the diagonal of the square is 80 cm.
II. The perimeter of the square is equal to the circumference of a circle.
2. What is Ashas present salary?
I. Her salary increases every year by 15 per cent.
II. She joined the organisation seven years ago.
3. What is the rate of interest p.c.p.a.?
I. The compound interest accrued on an amount of Rs 1,500 at the end of 2 years is Rs 660.
II. An amount doubles itself in 5 years at simple interest.
4. What is the total staff strength of the organisation?
I. 75% of the staff consists of male employees.
II. The ratio of female to male employees in the organisation is 1 : 3.
5. What is a two-digit number?
I. The sum of the two digits of the number is 8.
II. The number obtained by interchanging the two digits of the number is lesser than the original number by 18.
Directions: Study the table carefully to answer the following questions.
Percentage of marks obtained by six students in six different subjects

6. What is the approximate average marks obtained by all students together in Hindi?
1) 45
2) 51
3) 62

4) 59
5) 66
7. What is the ratio of the marks obtained by students A, B and C together in Maths to the marks obtained
by students D, E and F together in Marathi?
1) 7 : 22
2) 23 : 7
3) 23 : 4
4) 4 : 21
5) None of these
8. Which student has scored the second highest in all the subjects together?
1) D
2) F
3) A
4) B
5) None of these
9. In order to pass the exam, if the students need minimum 75 marks in Social Studies and minimum 85
marks in Science, how many students have passed the exam?
1) None
2) One
3) Two
4) Three
5) None of these
10. What is the percentage of marks obtained by B in all the subjects together? (rounded off to two digits
after decimal)
1) 68.62
2) 66.83
3) 69.24
4) 64.84
5) None of these

Answers
1. 1
I alone is sufficient because area of a square
= [diagonal of square/ sqroot(2)]^2
Since we have information regarding the diagonal of the square, it is easy to get the square. II alone is not
sufficient because the circumference of the circle is not known.
2. 4; We need some more information, such as the salary she was getting seven years ago.
3. 3;
Form I: We have 1500 (1 + r/100)^2 = 1500 + 660
Where r = rate of interest per annum)
1 + r/100 = sqroot(2160/1500)
1 + r/100 = 1 + 20/100
r = 20%
From II: Rate of interest per annum = 100/2 = 20%
4. 4
Neither of the two informations give clues regarding absolute number of employees.
5. 5
Suppose the two-digit number is 10x + y.
Now,

From I: We have x + y = 8
From II: We have x - y = 2
Now,
From I and II: we get
x = 5 and y = 3
Hence, the number is 53 (because 53 > 35)
6. 2
Required average = (65 + 75 + 60 + 70 + 55 + 60)/6 = 385/6 = 64 1/6
Aprox average marks = 64 1/6% of 80 = 51 approx
7. 5
= [150 * (84 + 70 + 66)%]/{50 * (60 + 74 + 76)] = (150 * 220)/(50 * 210)
8. 2

109. 3; Look at the chart given in the answer to the last question.
It is obvious that among the six students only A and C have passed the exam according to the given condition.
10. 1; The required per cent marks = 446/650 * 100 = 68.62%

bankersadda.com

http://www.bankersadda.com/2015/09/night-class-quant-quiz_10.html

Night Class: Quant Quiz


Direction: In the following questions two equations numbered (I) and (II) are given. You have to solve
both the equations and give answer :
1) If x > y
2) If x >= y
3) If x < y
4) If x <= y
5) If x = y or the relationship can't be established.
1.
I. x = sqroot(1369)
II. y = cuberoot(29791)

2.
I. 8x - 3y = 31
II. 5x + 4y = 84
3.
I. 20x^2 - 79x + 77 = 0
II. 4y^2 + 9y - 28 = 0
4.
I. 6x^2 + 29x + 28 = 0
II. 6y^2 + 11y + 4 = 0
5.
I. x^2 + 3x - 54 = 0
II. y^2 + 4y - 77 = 0
Direction (Q. 6 -10) : Following table shows the population of six different cities, ratio of male and female
among them and ratio of adult and minor among them. Answer given questions based on this table.

P.S. - Kindly click on the image for better view.


6.What is the total male population of city - B ?
1) 307420
2) 311850

3) 322450
4) 336280
5) None of these
7.What is the total adult population of city F ?
1) 187460
2) 173420
3) 184380
4) 178610
5) None of these
8. Total male population of city - C is approximately what percentage of total population of city-D ?
1) 57.8%
2) 61.4%
3) 63.7%
4) 65%
5) 67.5%
9.What is the difference between total number of adult and total number of minor of city - E ?
1) 76450
2) 80410
3) 81380
4) 82800
5) 83500
10. Total number of female of city-A is approximately what percentage of total number of female of city-B
?
1) 48.48%
2) 51.84%
3) 57.65%
4) 61.48%
5) 65.76%

1. 1;
x = sqroot(1369) = 37.................... (I)
y = cuberoot(29791) = 31 ................... (II)
x>y
2. 3;
Equn. (I) 4 + equn (II) 3
32 x 12 y = 124
15 x + 12 y = 252
_____________
47 x = 376
4
x = 8 and from this y = 11
x<y
3. 2;
20 x^2 35 x 44 x + 77 = 0
5 x (4 x 7) 11 (4 x 7) = 0
(4x 7) (5 x 11) = 0
x = 7/4, 11/5

4 y^2 + 16 y 7 y 28 = 0
4 y (y + 4) 7 (y + 4) = 0
(4 y 7) (y + 4) = 0
y = 4, 7/4
x >= y
4. 4;
6 x^2 + 8 x + 21 x + 28 = 0
2x (3 x + 4) + 7 (3x + 4) = 0
(3x + 4) (2 x + 7) = 0
x = -4/3, -7/2
6 y^2 + 3 y + 8 y + 4 = 0
3 y (2 y + 1) + 4 (2 y + 1) = 0
(3 y + 4) (2 y + 1) = 0
y = -4/3, -1/2
x <= y

5. 5;
x^2 + 9x 6 x - 54 = 0
x (x + 9) 6 ( x + 9) = 0
x = 6, 9
y^2 + 11 y 7 y 77 = 0
y (y + 11) 7 (y + 11) = 0
(y 7) (y + 11) = 0
y = 7, 11
i.e. No relation between x & y
6. 2;
B male = 554400/16 * 9 = 311850
7. 1
F(Ad) = 302820/21 * 13 = 187460
8. 3
Cmale = 369900/9 * 4 = 164400
Req% = 164400/258000 * 100 = 63.72%
9. 4
Diff = 281520/17 * (11 - 6) = 16560 * 5 = 82800
10. 5
A(Fe) = 333500/23 * 11 = 159500
B(Fe) = 554400/16 * 7 = 242550
Req% = 159500/242550 * 100 = 65.76%

bankersadda.com

http://www.bankersadda.com/2015/07/night-class-quant-quiz_24.html

Night Class: Quant Quiz


1. 1/37 * 5106 + 9/23 * 5681 = ? * 3
1. 518
2. 642
3. 787
4. 896
5. 936
2. cube root(12167) + cube root(4096) = sqroot(?)
1. 1369
2. 1521
3. 1681
4. 1849
5. 2203
3. 17.16 / 0.37 + 24.32 / 0.38 = ?
1. 112
2. 136
3. 13.806
4. 14.208
5. 15.688
4. 0.32% of 1658 + 0.48% of 2145 =?
1. 11.264
2. 12.718
3. 13.806
4. 14.208
5. 15.688
5. 7164 / 59.7 * 7.85 = ?
1. 648
2. 712
3. 884
4. 942
5. 1024
Directions: What will be the next number in the following number series6. 3, 8, 34, 206, 1650, ?
1) 16502
2) 17124
3) 17316
4) 18212
5) 18946
7. 4, 10, 18, 28, 40, ?
1) 48
2) 52
3) 54
4) 56
5) 58

8. 7, 61, 211, 505, 991, ?


1) 1512
2) 1587
3) 1643
4) 1717
5) 1751
9. 842, 425, 147, 42.75, ?
1) 12.15
2) 14.95
3) 16.35
4) 18.75
5) 20.25
10. 8.8, 35.2, 76.8, 134.4, 208, ?
1) 297.6
2) 312.4
3) 320.8
4) 324
5) 336.2
Answers
1. 3
2. 2
3. 1
4. 5
5. 4
6. 1; Series is 2 + 2, 4 + 2, 6 + 2
7. 3; Series is + 6, +8, +10, +12 --------8. 4; Series is 23 - 1, 43 -3, 63
- 5, 83 - 7
9. 2; Series is + 8 2, +16 3, +24 4 -----10. 1; Series is 8 1.1, 16 2.2, 24 3.2, 32 4.2

bankersadda.com

http://www.bankersadda.com/2015/07/night-class-quant-quiz_23.html

Night Class: Quant Quiz


Direction: Following line graph shows the percentage growth in population of six cities A, B, C, D, E and
F from 2011 to 2012 and 2012 to 2013.

1. If the population of City A is 5.4 lakh in year 2011 then what will be its population in year 2013.
1) 7.842 lakh
2) 8.424 lakh
3) 8.765 lakh
4) 9.168 lakh
5) None of these
2. What is the percentage increase in population of city C from year 2011 to year 2013.
1) 15%
2) 22.5%
3) 30%
4) 32.25%
5) 33.5%
3. If the population of city E and City D are equal in year 2012 then the population of city E is
approximately what percentage of population of city D in 2013.
1) 86.2%
2) 96%
3) 104%
4) 116%
5) 124%
4. If the population of city-F in year 2013, is 16.848 lakh then what was its population in year 2011 ?
1) 7.8 lakh
2) 8.4 lakh
3) 9.6 lakh
4) 10.2 lakh
5) 11.4 lakh
5. If the population of city-B and city-D in year 2011 are equal to 6 lakh then what will be the difference
between population of city-D in 2013 and population of city-B in 2013 ?
1) 75500

2) 97400
3) 112500
4) 137600
5) 148500
Directions : What will be the next number in the following number series6. 3, 8, 34, 206, 1650, ?
1) 16502
2) 17124
3) 17316
4) 18212
5) 18946
7. 4, 10, 18, 28, 40, ?
1) 48
2) 52
3) 54
4) 56
5) 58
8. 7, 61, 211, 505, 991, ?
1) 1512
2) 1587
3) 1643
4) 1717
5) 1751
9. 842, 425, 147, 42.75, ?
1) 12.15
2) 14.95
3) 16.35
4) 18.75
5) 20.25
10. 8.8, 35.2, 76.8, 134.4, 208, ?
1) 297.6
2) 312.4
3) 320.8
4) 324
5) 336.2
Answers
1. 2
A2013 = 540000 * 120/100 * 130/100 = 842400
2. 4
Let the population in year 2011 = 100
Population 2013 = 100 * 115/100 * 115/100 = 132.25
% increase = 32.25%
3. 4
Let the population of D & E by x in year 2012
D2013 = x * 125/100 = 1.25x
E2013 = x * 145/100 = 1.45x

Req % = 1.45x/1.25x * 100 = 116%


4. 3
F2011 = 1684800 * 100/130 * 100/135 = 960000
5. 3
D2013 = 600000 * 135/100 * 125/100 = 1012500
B2013 = 600000 * 125/100 * 120/100 = 900000
Diff = 1012500 - 900000 = 112500
6. 1; Series is 2 + 2, 4 + 2, 6 + 2
7. 3; Series is + 6, +8, +10, +12 --------8. 4; Series is 23 - 1, 43 -3, 63
- 5, 83 - 7
9. 2; Series is + 8 2, +16 3, +24 4 -----10. 1; Series is 8 1.1, 16 2.2, 24 3.2, 32 4.2

bankersadda.com

http://www.bankersadda.com/2015/07/night-class-quant-quiz_23.html

Night Class: Quant Quiz


Direction: Following line graph shows the percentage growth in population of six cities A, B, C, D, E and
F from 2011 to 2012 and 2012 to 2013.

1. If the population of City A is 5.4 lakh in year 2011 then what will be its population in year 2013.
1) 7.842 lakh
2) 8.424 lakh
3) 8.765 lakh
4) 9.168 lakh
5) None of these
2. What is the percentage increase in population of city C from year 2011 to year 2013.
1) 15%
2) 22.5%
3) 30%
4) 32.25%
5) 33.5%
3. If the population of city E and City D are equal in year 2012 then the population of city E is
approximately what percentage of population of city D in 2013.
1) 86.2%
2) 96%
3) 104%
4) 116%
5) 124%
4. If the population of city-F in year 2013, is 16.848 lakh then what was its population in year 2011 ?
1) 7.8 lakh
2) 8.4 lakh
3) 9.6 lakh
4) 10.2 lakh
5) 11.4 lakh
5. If the population of city-B and city-D in year 2011 are equal to 6 lakh then what will be the difference
between population of city-D in 2013 and population of city-B in 2013 ?
1) 75500

2) 97400
3) 112500
4) 137600
5) 148500
Directions : What will be the next number in the following number series6. 3, 8, 34, 206, 1650, ?
1) 16502
2) 17124
3) 17316
4) 18212
5) 18946
7. 4, 10, 18, 28, 40, ?
1) 48
2) 52
3) 54
4) 56
5) 58
8. 7, 61, 211, 505, 991, ?
1) 1512
2) 1587
3) 1643
4) 1717
5) 1751
9. 842, 425, 147, 42.75, ?
1) 12.15
2) 14.95
3) 16.35
4) 18.75
5) 20.25
10. 8.8, 35.2, 76.8, 134.4, 208, ?
1) 297.6
2) 312.4
3) 320.8
4) 324
5) 336.2
Answers
1. 2
A2013 = 540000 * 120/100 * 130/100 = 842400
2. 4
Let the population in year 2011 = 100
Population 2013 = 100 * 115/100 * 115/100 = 132.25
% increase = 32.25%
3. 4
Let the population of D & E by x in year 2012
D2013 = x * 125/100 = 1.25x
E2013 = x * 145/100 = 1.45x

Req % = 1.45x/1.25x * 100 = 116%


4. 3
F2011 = 1684800 * 100/130 * 100/135 = 960000
5. 3
D2013 = 600000 * 135/100 * 125/100 = 1012500
B2013 = 600000 * 125/100 * 120/100 = 900000
Diff = 1012500 - 900000 = 112500
6. 1; Series is 2 + 2, 4 + 2, 6 + 2
7. 3; Series is + 6, +8, +10, +12 --------8. 4; Series is 23 - 1, 43 -3, 63
- 5, 83 - 7
9. 2; Series is + 8 2, +16 3, +24 4 -----10. 1; Series is 8 1.1, 16 2.2, 24 3.2, 32 4.2

bankersadda.com

http://www.bankersadda.com/2015/07/night-class-quant-quiz_22.html

Night Class: Quant Quiz


1. In an examination a student attempted all 60 questions. He gets 2 marks for every right answer and
minus 1 marks for each wrong answer. Find how many wrong questions were he attempted if got total 72
marks ?
1) 48
2) 58
3) 16
4) 20
5) None of these
7. Average monthly salary of 8 technicians of a workshop was Rs. 1200 and average monthly salary of
non-technician was Rs. 300. Find total number of employee of this workshop if average monthly salary of
all the employee was Rs. 500 ?
1) 16
2) 36
3) 56
4) 40
5) None of these
8. A shopkeeper buys some quantity of lemon at the rate 3 in Rs. 2 and buys same quantity of lemon at
the rate 2 in Rs. 3. He sold all of them at the rate 5 in Rs. 5. If during the transaction the shopkeeper
losses Rs. 30. Find the quantity of the first types of lemon that he mixed ?
1) 360
2) 280
3) 180
4) 240
5) None of these
9. A shopkeeper gives two successive discount of 50% and 40% respectively. If second discount is Rs.
180. Find selling price of this article ?
1) 900
2) 270
3) 370
4) 570
5) None of these
10. A man travels 25% part of the Journey at the speed of 50 km/hr, next 60 % part of the journey at the
speed of 40 km/hr. and rest part of the journey at the speed of 20 km/hr. Find average speed of the men
during whole journey ?
1) 34 4/11 km/hr
2) 37 4/11 km/hr
3) 36 4/11 km/hr
4) 80 km/hr
5) None of these

Answers:
6. 3

Let he attempted x correct and y wrong question


2x - y = 72-------- (I) and x + y = 60 -------- (II)
Solving (I) and (II) x = 44, y = 16
7. 2
Let number of non technician = x
1200 8 + 300 x = ( 8 + x) 500
9600 + 300 x = 500 x + 4000
x = 28
Total employee = 8 + 28 = 36
8. 3
Let no. of Ist type lemon = No. of IInd type lemon = x
CP = 2x/3 + 3x/2 = 13x/6
SP = 2x * 5/5 = 2x
13x/6 - 2x = 30
13x - 12x = 180
x = 180
9. 2
Let cost of article after Ist discount is x
x * 40/100 = 180
x = 450
SP after IInd discount = 450 - 180 = 270
10. 3
Let total distance is 100KM
T1 = 25/50 = 1/2 hour
T2 = 60/40 = 3/2 hour
T3 = 15/20 = 3/4 hour
T = 1/2 + 3/2 + 3/4 = 11/4 hour
Avg speed = 100/(11/4) = 36 4/11 km/hr

bankersadda.com

http://www.bankersadda.com/2015/07/night-class-quant-quiz_21.html

Night Class: Quant Quiz


Direction: What will come in place of the question mark :
1. 124, 228, 436, ?, 1684, 3348
1) 944
2) 852
3) 872
4) 444
5) None of these
2. 1, 3, 24, 360, 8640, ?, 14515200
1) 282400
2) 292400
3) 302400
4) 25426
5) None of these
3. 10, 17, 48, 165, ?, 3475, 20892
1) 688
2) 712
3) 848
4) 918
5) None of these
4. 15, 17, 38, 120, 488, ?, 14712
1) 2450
2) 2650
3) 2850
4) 2950
5) None of these
5. 1, 6, 19, ?, 85, 146, 231
1) 46
2) 44
3) 48
4) 65
5) None of these
6. A hemisphere a cylinder and a cone stand on the same base and have the same height. Then find the
ratio of their volumes respectively ?
1) 1 : 2 : 3
2) 3 : 2 : 1
3) 2 : 3 : 1
4) 2 : 1 : 3
5) None of these
7. 1/3rd of a tank is filled with milk. After taking out 8 litres of milk 1/4th of the is still filled. Find the
capacity of the tank ? (in litre)
1) 84 litre
2) 96 litre
3) 156 litre

4) 80 litre
5) None of these
8. If the ratio of heights and radius of two cones are respectively 2 : 7 and 4 : 3, the ratio of their volume
is ?
1) 35 : 63
2) 32 : 63
3) 63 : 32
4) 3 : 49
5) None of these
9. If 27 is subtracted from a number then it is reduced to 40% of its original. The 1/3rd of the number is ?
1) 15
2) 30
3) 60
4) 75
5) None of these
10. In a mixture of 90 litres, the ratio of milk and water is 4 : 2. How much water should be added to the
mixture so that the ratio of milk and water becomes 6 : 4 ?
1) 50
2) 10
3) 40
4) 20
5) None of these
1. 2;
2 - 20, 2 - 20, 2 - 20, -----------------2. 3;
(22 - 1), (32 - 1), (42 - 1), (52 -1), (62 -1) ---------3. 1;
1 + 7, 2 + 14, 3 + 21, 4 + 28 ---------------4. 1;
1 + 2, 2+ 4 , 3 + 6, 4 + 8, 5 + 10 -----------5. 2
6 .3
2/3 * 22/7 * r^3 : 22/7 r^2 h : 1/3 * 22/7 * r^2 h
Since r = h
2:3:1
7. 2
1/3x - 1/4x = 8
x = 96litres
8. 2
2 * 4^2 : 7 * 3^2
32:63
9. 1
x - 27 = 2/5x
3x = 27 * 5

x = 15
10. 2
Let x litres water us added
60/(30 + x) = 6/4 = 3/2
120 = 90 + 3x
x = 30/3 = 10 litres

bankersadda.com

http://www.bankersadda.com/2015/07/night-class-quant-quiz_18.html

Night Class: Quant Quiz


Direction : Following line graph shows the ratio of imports to exports of two companies over the years:

1. In how many of the given years were the imports more than the exports in case of company - A ?
1) 5
2) 3
3) 1
4) 2
5) 4
2. If the imports of company-B in year 2011 were Rs. 51.688 lakh, then what were the exports of company-B in the
same year?
1) 64.26 lakh
2) 67.54 lakh
3) 71.36 lakh
4) 73.84 lakh
5) None of these
3. If the exports of company-A and Company-B were equal to 84 lakh in year 2012 then what will be the difference
between
imports of company-B and imports of company-A in that year ?
1) 9.6 lakh
2) 8.4 lakh
3) 7.2 lakh
4) 6.8 lakh
5) 5.4 lakh
4. If the exports of company-A in year 2009 and exports of company-B in year 2012 were equal then the imports of
companyB in 2012 is approximately what percentage of imports of company-A in 2009 ?
1) 60%
2) 75%
3) 84%
4) 96%
5) 133.3%
5. In year 2009, If the export of company-B is increased by 100% and import is increased by 200%. Then what will
be the new ratio of import to export of company-B in that year ?
1) 0.8

2) 1.0
3) 1.2
4) 0.6
5) 1.5
6. A cycle is sold at a profit of 15%. If both the cost price and selling price is decreased by Rs. 200 the profit would
be 5% more what is the original cost price of cycle ?
1) Rs. 1200
2) Rs. 1000
3) Rs. 800
4) Rs. 600
5) None of these
7. A person lends Rs. 4200 to 'A' for 3 years and Rs. 6500 to 'B' for 7 years. If the person gets Rs. 8134 as interest
then what is the rate of interest per annum ?
1) 6%
2) 8%
3) 10%
4) 12%
5) 14%
8. A sum amounts to Rs. 98010 in 2 years and to Rs. 107811 in 3 years compounded annually. What is the sum ?
1) Rs. 81000
2) Rs. 84000
3) Rs. 87000
4) Rs. 90000
5) Rs. 94000
9. How many different words can be formed with the letters of the word ''BREAKING". So that the words begin with
'B' and end with 'G' ?
1) 120
2) 720
3) 5040
4) 2520
5) 1440
10. A bag contains 9 white and 7 black balls. Three balls are drawn at random. Find the probability that all the
three balls are black ?
1) 1/8
2) 1/12
3) 1/16
4) 1/21
5) None of these
ANSWERS
1. 4
Imports > Exports
Imports/Exports >1
2. 4
I/E = 0.7
E = I/0.7
= 51.688/0.7
73.84lakh

3. 2
Ia/Ea = 0.8
Ia = 0.8 * 84 = 67.2lakh
Ib/Eb = 0.9
Ib = 0.9 * 84 = 75.6lakh
Diff = 8.4 lakh
4. 2
Ia/Ea = 1.2
Ib/Eb = 0.9
Let export of A and B are X
Ia = 1.2x
Ib = 0.9x
Req% = 0.9x/1.2x * 100 = 75%
5. 3
I/E = 0.8
I1 = I +200I/100 = 3I
E1 = E + 100E/100 = 2E
Ratio = I1/E1 = 3I/2E = 3/2 * 0.8 = 1.2
6. 3
Let the original cost proce is x, so SP is
115x/100 = 23x/20
Now CP = x - 200
SP = 23x/20 - 200
Profit = 23x/20 - 200 - (x - 200) = 23x/20 - x = 3x/20
% Profit = 3x/20 * 1/(x - 200) * 100 = 20
3x/(x - 200) = 4
3x = 4x - 800
x = 800
7. 5
Let the rate of interest is r% p.a.
4200*r*3/100 + 6500*r*7/100 = 8134
126r + 455r = 8134
r = 8134/581 = 14% p.a.
8. 1
P (1 + R/100)^3 = 107811
p(1 + r/100)^2 = 98010
1 + r/100 = 107811/98010
r/100 = 9801/98010
r = 10%
P = (1 + 10/100)^2 = 98010
p = 98010 * 100/121 = 81000
9. 2
B______G
Now 6 letters are left for six different place
Req No. of wages = 6P6 = 6! = 720
10. 3

n(S) = Number of ways of selecting 3 balls out of 7 = 7C3 = 7!/3!4! = 35


p(E) = 35/560 = 1/16

bankersadda.com

http://www.bankersadda.com/2015/07/night-class-quant-quiz_17.html

Night Class: Quant Quiz


Direction: In the following questions two equations numbered (I) and (II) are given. You have to solve
both the equations and give answer :
1) If x > y
2) If x y
3) If x < y
4) If x y
5) If x = y or the relationship can't be established.
1.
I. x = sq root(1369)
II. y = cube root(29791)
2.
I. 8x - 3y = 31
II. 5x + 4y = 84
3.
I. 20x^2 - 79x + 77 = 0
II. 4y^2 + 9y - 28 = 0
4.
I. 6x^2 + 29x + 28 = 0
II. 6y^2 + 11y + 4 = 0
5.
I. x^2 + 3x - 54 = 0
II. y^2 + 4y - 77 = 0
Direction: Following table shows the population of six different cities, ratio of male and female among
them and ratio of adult and minor among them. Answer given questions based on this table.

6.What is the total male population of city - B ?


1) 307420
2) 311850
3) 322450
4) 336280
5) None of these
7.What is the total adult population of city F ?

1) 187460
2) 173420
3) 184380
4) 178610
5) None of these
8. Total male population of city - C is approximately what percentage of total population of city-D ?
1) 57.8%
2) 61.4%
3) 63.7%
4) 65%
5) 67.5%
9.What is the difference between total number of adult and total number of minor of city - E ?
1) 76450
2) 80410
3) 81380
4) 82800
5) 83500
10. Total number of female of city-A is approximately what percentage of total number of female of city-B ?
1) 48.48%
2) 51.84%
3) 57.65%
4) 61.48%
5) 65.76%
Answers:
1.
x = sq root(1369) = 37
y = cube root(29791) = 31
x>y
2. 3
Eq. I * 4 + Eq. II * 3
32x - 12y = 124
15x + 12y = 252
47x = 376
x = 8 & y = 11
x<y
3. 2
20x^2 - 35x - 44x + 77 = 0
5x(4x - 7) - 11(4x -7) = 0
(4x-7)(5x-11)=0
x = 7/4, 11/5
4y^2 + 16y - 7y - 28 = 0
4y (y + 4) - 7(y + 4)= 0
(4y - 7)(y + 4) = 0
y= -4, 7/4
4. 4
6x^2 + 8x + 21x + 28 = 0
2x(3x + 4) + 7(3x + 4) = 0
(3x + 4)(2x + 7) = 0

x = -4/3, -7/2
6y^2 + 3y + 8y + 4 = 0
6y(2y + 1) + 4(2y + 1) = 0
(3y+4)(2y+1) = 0
y = -4/3, -1/2
5. 5
x^2 + 9x - 6x - 54 = 0
x(x + 9) - 6(x + 9) = 0
x = 6, -9
y^2 + 11y - 7y - 77 = 0
y(y + 11) - 7(y + 11) = 0
(y - 7)(y + 11) = 0
y = 7, -11
6. 2
Bmale = 554400/16 * 9 = 311850
7. 1
Fad = 302820/21 * 13 = 187460
8. 3
Cmale = 369900/9 * 4 = 164400
Req% = 164400/258000 * 100 = 63.72%
9. Diff = 281520/17 * (11-6)
= 16560 * 5 = 82800
10. Afe = 333500/23 * 11 = 159500
Bfe = 554400/16 * 7 = 242550

Req% = 159500/242550 * 100 = 65.759 = 65.76% approx

bankersadda.com

http://www.bankersadda.com/2015/07/night-class-quant-quiz_16.html

Night Class: Quant Quiz


1. (4.193)^2/3 / 2.89 * (8.3521)^(-1.2) = (1.7)^?
1) 3.2
2) -4.8
3) 1.2
4) -4.2
5) 2.4
2. {21/34 of (4624)} / 0.6 = ?
1) 60
2) 64
3) 70
4) 72
5) 84
3. 0.78 + ? * 72 = 13.74
1) 1.2
2) 0.96
3) 0.8
4) 0.24
5) 0.18
4. [(546)^2 / 91] / 12 = ? / 8
1) 2104
2) 2136
3) 2168
4) 2184
5) 2196
5. 3/7 of 5/11 2772 = 30% of ?
1) 1200
2) 1500
3) 1600
4) 1800
5) 2100
Direction : What value should come in the place of question mark (?) in the following number series :
6. 6, 11, 32, 111, 464, ?
1) 2165
2) 2205
3) 2285
4) 2345
5) 2375
7. 3, 14, 39, 84, 155, ?
1) 236
2) 258
3) 264
4) 272

5) 278
8. 8, 12, 24, 60, 180, ?
1) 480
2) 510
3) 630
4) 720
5) 780
9. 168, 171, 178, 191, 212, ?
1) 243
2) 247
3) 251
4) 254
5) 257
10. 4, 7, 10, 11, 22, 19, 46, ?
1) 33
2) 35
3) 37
4) 39
5) 41
Answers:
1. 2
(1.7)^2/3 / (1.7)^2 * (1.7^4)^(-1.2)
(1.7)^2 / (1.7)^2 * (1.7)^(-4.8)
(1.7)^(2-2-4.8) = (1.7)^(-4.8)
? = -4.8
2. 3
(21/34 * 68) / 0.6 = 42 / 0.6 = 70
3. 5
? * 72 = 13.74 - 0.78 = 12.96
= = 12.96/72 = 0.18
4. 4
? / 8 = (546 * 549/91) / 12
3276 / 12 = 273
? = 273 * 8 = 2184
5. 4
(30 * ?)/100 = (3 * 5 * 2772)/ (7 * 11) = 540
? = 1800
6. 4
Series is 1+5, 2 + 10, 3 + 15, 4 + 20
7. 2
Series is 1+1^2+1^3, 2+2^2+2^3, 3+3^2+3^3
8. 3
Series is *0.5+8, *1.0+12, *1.5+24
9. 1
Seres is +1+1^2, +2^2+3, +3^2+4....

10. 2
Series 1 - 4, 4+6 = 10, 10 + 12 = 22, 22 + 24 = 26
Series 2 - 7, 7 + 4 = 11, 11 + 8= 19, 19 + 16 = 35

bankersadda.com

http://www.bankersadda.com/2015/07/night-class-quant-quiz_11.html

Night Class: Quant Quiz


Directions : From a class total 360 students appeared in an exam. Three fourth of these are boys. 40 %
boys failed in the exam, two third of the girls are failed in the exam and remaining passed :
1.What is the ratio of the total number of girls to the number of boys who failed in the exam ?
1) 4 : 9
2) 9 : 5
3) 5 : 6
4) 6 : 3
5) None of these

2.What is the sum of the number of boys who failed and number of girls who passed in the exam ?
1) 148
2) 158
3) 128
4) 138
5) None of these
3.What is the difference between the number of boys who are passed and number of girls who are failed
?
1) 102
2) 202
3) 52
4) 302
5) None of these
4.What is the ratio of the total number of boys and girls who are passed to the number of boys and girls
who are failed ?
1) 7 : 8
2) 8 : 7
3) 9 : 17
4) 5 : 7
5) None of these
5. Total number of boys passed is approximately what percent of total number of girls in the class ?
1) 60 %
2) 120 %
3) 150 %
4) 180 %
5) 270 %
Direction: A bag contains 4 red, 5 brown and 6 white balls. Three balls are drawn randomly :
6.What is the probability that balls drawn contains exactly two red balls ?
1) 54/145
2) 66/455
3) 46/105
4) 31/455

5) None of these
7.What is the probability that the balls drawn contains no brown ball ?
I) 11/65
2) 31/91
3) 24/91
4) 31/455
5) None of these
8.What is the probability that the balls drawn are not of the same colour ?
1) 34/455
2) 31/105
3) 74/105
4) 421/455
5) None of these
9. If two balls are drawn randomly then what is the probability that both the balls are of same colour ?
1) 31/105
2) 74/105
3) 34/455
4) 421/455
5) None of these
10.What is the probability that the three balls drawn are of different colour ?
1) 21/91
2) 31/105
3) 24/105
4) 24/91
5) None of these

ANSWERS:
1. 3
Ratio = 90/108 = 5/6
2. 4
Sum = 108 + 30 = 138
3. 1
Diff = 162 - 60 = 102
4. 2
Ratio = 192/168 = 8/7
5. 4
Req% = 162/90 * 100 = 180%
6. 2
n(S) = 15C3 = (15 * 14 * 13)/6 = 455
2 red balls can be selected in 4c2 = 6 ways
And remaining one ball can be selected in 11C1 = 11 ways
P(E) = (6 * 11)/455 = 66/455

7. 3
n(S) = 15C3 = 455
Three balls have to be selected from 4 red and 6 white balls
No. of ways = 10C3 = 120
P(E) = 120/455 = 24/91
8. 4
n(S) =15C3 = 455
If all three ball are of same colour, number of ways
= 4C3 + 5C3 + 6C3 = 4 + 10 + 20 = 34
P(E) = 34/455
For being different colour P(E) = 1 - 34/455 = 421/455
9. 1
n(S) = 15C2 = 105
n(e) = 4C2 + 5C2 + 6C2 = 6 + 10 + 15 = 31
P(E) = 31/105
10. 4
n(S) = 15C3 = 455
n(E) = 4C1 + 5C1 + 6C1 = 4 * 5 * 6 = 120
P(E) = 120/455 = 24/91

bankersadda.com

http://www.bankersadda.com/2015/07/night-class-quant-quiz.html

Night Class: Quant Quiz


Directions: Following pie chart shows the monthly expenditure of a man on different items. His income is
Rs 8,000 monthly.

1. If his salary is Rs 8,000 then how much he pays for food?


1) Rs 1800
2) Rs 1920
3) Rs 2040
4) Rs 2200
5) None of these
2. What is the angle shown by medical expenditure?
1) 22
2) 21 1/3
3) 34
4) 29 1/5
5) 57 3/5
3. What is his monthly saving?
1) Rs 1920
2) Rs 1720
3) Rs 1120
4) Rs 1260
5) None of these
4. What is the difference between money he spent on education and medical purposes?
1) Rs 360
2) Rs 260
3) Rs 160
4) Rs 60
5) None of these
5. If the salary is raised to Rs 12,000 and distribution on various items remains the same, then how much he
spends on medical charges?
1) Rs 1800
2) Rs 2040
3) Rs 2200
4) Rs 1920
5) None of thes
Directions : Each question below is followed by two statements I and II. You are to determine whether the data
given in the statement is sufficient to answer the question. You should use the data and your knowledge of
Mathematics to choose between the possible answers. Give answer
1) If the statement I alone is sufficient to answer the question, but the statement II alone is not sufficient.
2) If the statement II alone is sufficient to answer the question, but the statement I alone is not sufficient.
3) If both statements I and II together are needed to answer the question.
4) If either the statement I alone or the statement II alone is sufficient to answer the question.
5) If you cannot get the answer from the statements I and II together, but need even more data.

6. What is the area of rectangle?


I. Perimeter is 20 meter.
II. Sum of two adjacent sides is 10 meters.
7. How many letters can 12 typists type in one day?
I. A working day consists of 8 hours?
II. 5 typists can type 200 letters in 2 days.
8. If x and y are integers, is x + y an even number?
I. 5 < x < 10
II. 6 < y < 9
9. What is the value of P?
I. The average of x, y and P is 10.
II. x = y
10. If a man wants to cover a square room with a carpet, how much will it cost?
I. If the service charge is Rs 15 and each square yard cost is Rs 20.
II. If the room were 1/2 as large in area, it would cost Rs 150 less
1. 2
(24*8000)/100 = 1920
2. 5
Angle = 16/100*360 = 288/5 = 57 3/5
3. 3
14/100 * 8000 = 1120
4. 3
Education = 18% of 8000
= 8000 * 18/100 = 1440
Medical = 16% of 8000
8000 * 16/100 = 1280
Difference = 1440 - 1280 = 160
5. 4
12000 * 16/100 = 1920
6. 5; Using Statement I:
2(L + B) = 20
L + B = 10
Using Statement II:
L + B = 10
So, even by using both the statements together we cannot
find the area of the rectangle.
7. 2; Using Statement II:
In 2 days 5 typists type 200 letters.
In 1 day 5 typists type 100 letters.

In 1 day 1 typist types 20 letters.


In 1 day 12 typists type 240 letters.
8. 5; Using Statement I:
x = 6, 7, 8, 9
Using Statement II:
y = 7, 8
So, even by using both the statements together we cannot
find the answer, as x + y can be odd as well as even.
9. 3;
From statement I:
(x + y + P)/3 = 10
x + y + P = 30
From statement II:
x = y
x + y = 0
x + y + P = 30
10. 2; Using II alone
Cost to cover the room with carpet will be 300.

bankersadda.com

http://www.bankersadda.com/2015/09/night-class-quant-quiz_9.html

Night Class: Quant Quiz


Directions: In each of these questions, two equations numbered I and II with variables x and y are given.
You have to solve both the equations to find the value of x and y. Give answer:
1) if x > y
2) if x >= y
3) if x < y
4) if x <= y
5) if x = y or relationship between x and y cannot be determined.
1.
I. 20x^2 + 73x + 63 = 0
II. 16y^2 + 16y + 3 = 0

2.
I. x^2 26x + 165 = 0
II. y^2 36y + 315 = 0
3.
I. 3x^2 32 x + 77 = 0
II. 2y^2 + 23y + 56 = 0
4.
I. 63x^2 + 71x + 20 = 0
II. 45y^2 + 52y + 15 = 0
5.
I. x^2 + 16x + 63 = 0
II. 6y^2 37y + 52 = 0
6. A fruit seller had some pineapples. He sells 30% of pineapples and still has 350 pineapples. Originally
he had how many pineapples?
1) 480
2) 500
3) 400
4) 460
5) 450
7. P, Q and R hire a penthouse for Rs. 2250. If they used it for 8 hr, 10 hr and 12 hr respectively, the rent
paid by R will be:
1) Rs. 800
2) Rs. 950
3) Rs. 900
4) Rs. 850
5) Rs. 1000
8. Two pipes can fill a tank in 20 hrs and 24 hrs respectively. A third pipe can empty the tank in 40 hours.
If all the three pipes are opened and function simultaneously. Then, in how much time the tank will be

full?
1) 10 hrs
2) 15 hrs
3) 16 hrs
4) 12 hrs
5) 18hrs
9. 15 times A is equal to 10 times B. If 10 is subtracted from B it is 22 less than 3 times of A. What is the
sum of A and B?
1) 10
2) 20
3) 24
4) 18
5) 28
10. A train covers 450 km in 6 hours. The speed of bike is half of the speed of train. How much time will
the bike take to cover the distance of 300km?
1) 6 hr
2) 7.5 hr
3) 7 hr
4) 10 hr
5) 8 hr

1. 3;
I. 20x^2 + 45x + 28x + 63 = 0
(5x + 7) (4x + 9) = 0
x = -7/5, 9/4
II. 16y^2 + 12y + 4y + 3 = 0
(4y +1) (4y +3) = 0
y = -1/4, -3/4
x<y
2. 4;
I. x^2 15x 11x + 165 = 0
or, (x 11) (x 15) = 0
or, x = 11, 15
II. y2 21y 15y + 315 = 0
or, (y 15) (y 21) = 0
or, y = 15, 21
x <= y
3. 1;
I. 3x^2 21x 11x + 77 = 0
or, (3x 11) (x 7) = 0
x = 11/3, 7
II. 2y^2 + 16y + 7y + 56 = 0
or, (2y + 7) ( y + 8) = 0

y = -7/2,8
x>y
4. 5;
I. 63x^2 + 35x + 36x + 20 = 0
(7x +4) (9x + 5) = 0
x = -4/7, -5/9
II. 45y^2 + 27y + 25y + 15 = 0
(9y + 5) (5y + 3) = 0
y = -5/9, -3/5
No relationship between x and y exists
5. 3;
I. x^2 + 9x + 7x + 63 = 0
(x + 7) (x + 9) = 0
x = 7, 9
II. 6y^2 24y 13y + 52 = 0
(6y 13) ( y 4 ) = 0
y = 13/6, 4
x<y
6. 2;
Let the total number of pineapple be x
70/100 x = 350
x = 350 * 100/70 = 500
7. 3;
Ratio of their uses = 8 : 10 : 12 = 4 : 5 : 6
Rent paid by R = 6/15 * 2250 = Rs. 900
8. 2;
Part of tank filled in 1 hour
1/20 + 1/24 - 1/40 = 15
Thus, required time for the tank to be filled = 15 hrs.
9. 2;
15 A = 10 B
3A = 2B
Now, B 10 = 3A 22
or, B 10 = 2B 22 or, (Put 3A = 2B)
or, B = 12
A = 2*12/3 = 8
Required sum = 20
10. 5;
Speed of train = 450/6 = 75 km/hr
Speed of bike = 75/2 = 37.5 km/hr
Time taken by bike = 300/37.5 = 8 hour

bankersadda.com

http://www.bankersadda.com/2015/09/night-class-quant-quiz_8.html

Night Class: Quant Quiz


Directions: What value should come in the place of question mark (?) in the following questions?
1. (72)^2 + (8)^3 (9)^2 = ?
1) 5815
2) 5425
3) 5615
4) 5695
5) 5605
2. 7/31 of 6231 + 14.5% of 740 = 30% of 900 + ?
1) 1044.4
2) 1028.5
3) 1245.8
4) 1244.3
5) 1298.6
Directions: What approximate value should come in the place of question mark (?) in the following questions?
3. (24.26)^3 (8.2)^3 = 5 + ?
1) 15
2) 36
3) 32
4) 29
5) 22
4. 2799 27.86 6.15 125.98 245.27 = ?
1) 219
2) 242
3) 229
4) 234
5) 248
5. sq root(5244.10)/53.10 * sqroot(44943.99) + (132651.01)^1/3 / (?)^1/2 = 1018
1) 1/4
2) 1/9
3) 1/25
4) 1/3
5) 1/16
Directions (Q. 36 40): What value should come in the place of question mark (?) in the following number series?
6. 17, 21, 37, 73, ?
1) 96
2) 94
3) 90
4) 137
5) 99
7. 11, 18.5, 44.5, 141, ?
1) 501.5

2) 517.5
3) 571.5
4) 561.5
5) 525.5
8. 474, 486, 510, 546,?
1) 574
2) 594
3) 548
4) 547
5) 584
9. 9, 43, 212, 1056, ?
1) 5275
2) 5175
3) 5075
4) 5285
5) 5365
10. 73, 77, 85, 99, ?
1) 123
2) 120
3) 119
4) 121
5) 124
Answers:
1. 3
2. 4
3. 5
4. 3
5. 1
6. 4
7. 3
8. 2
9. 1
10. 4

bankersadda.com

http://www.bankersadda.com/2015/09/night-class-quant-quiz_7.html

Night Class: Quant Quiz


Direction : Study the following graph carefully and answer the questions below :
Total Sales of Hindi and English newspaper in five different localities of a city :

1.What is the difference between the total sale of Hindi newspaper and the total sale of English
newspaper in all the localities together ?
1) 4000
2) 5000
3) 3000
4) 2500
5) None of these
2. The sale of Hindi newspaper in locality P is approximately what percent of the total sale of Hindi
newspaper in all the localities together ?
1) 20%
2) 15%
3) 18%
4) 24%
5) None of these
3.What is the ratio of the sale of Hindi newspaper in locality R to the sale of English newspaper in locality
T?
1) 12 : 7
2) 11 : 7
3) 7 : 12
4) 8 : 12
5) None of these
4. The sale of English and Hindi newspaper is locality P is what percent of the sale of English and Hindi

newspaper in locality S ?
1) 220
2) 125
3) 120
4) 135
5) None of these
5. What is the total sale of English newspaper in all the localities together ?
1) 22,000
2) 20,000
3) 25,000
4) 21,000
5) None of these
6. What is the average sale of English and Hindi newspaper in locality R and S ?
1) 9000
2) 14000
3) 12000
4) 8000
5) None of these
Direction: Study the following table carefully and answer the questions given below :
Number of students enrolled with five colleges over the years :

7. If from college Q in 2009, 90 % of the students enrolled appeared in a competitive examination, out of
which 75 % students passed, how many students passed the examination ?
1) 280
2) 216
3) 243
4) 270
5) None of these
8. In 2007, from all the colleges together an overall 40 % of the students enrolled for a computer course
total how many students enrolled for these course ?
1) 700
2) 600
3) 800

4) 900
5) None of these
9. What is the ratio of the average number of student enrolled with colleges together in 2009 to that in
2010 ?
1) 117 : 108
2) 111 : 113
3) 110 : 113
4) 105 : 108
5) None of these
10. The Average number of students enrolled from college Q for all the years together is approximately
what percent of the average number of students enrolled from college R for all the years together ?
1) 83
2) 85
3) 87
4) 80
5) None of these
11. In 2008, from all colleges together 10 % students enrolled went abroad. Approximately how many
students went abroad?
1) 311
2) 211
3) 321
4) 521
5) None of these
12. What is the average number of students enrolled for college R for all the years is
1) 302
2) 502
3) 402
4) 602
5) None of these

Answers:
1. 3;
(23000 20000) = 3000
2. 4;
5500/23000 * 100 = 23.91 = 24%
3. 1;
6000/3500 = 12/7 = 12 : 7
4. 3;
9000/7500 * 100 = 120
5. 2;
6. 4;
(8000 + 7500)/2 = 16000/2 = 8000

7. 3;
360 * 9/10 * 3/4 = 243
8. 4;
480 + 350 + 380 + 500 + 540 40 % = 900
9. 3;
10. 1;
(320 350 300 360 340)/ (400 380 410 430 390) * 100
= 1670/2010 * 100 = 83%
10. 2;
(420 + 380 + 410 + 520 + 460) 10 %
= 2110 * 10/100 = 211
11. 3;
(400 380 410 430 390)/5 = 2010/5 = 402

bankersadda.com

http://www.bankersadda.com/2015/09/night-class-quant-quiz_3.html

Night Class: Quant Quiz


Direction: Study the following information carefully and answer the question given below it :
Percentage proportion of income of seven companies in a state during financial year 2013 14 :

1. If the expenditure of company D is 750 crore, what was the total income of all the given companies together ?
1) Rs. 4135 crore
2) Rs. 4115 crore
3) Rs. 4125 crore
4) Rs. 4025 crore
5) None of these
2. If in the given financial year income of company A was 440 crore. What was the approximately expenditure of
company C in that year ?
1) Rs. 45 crore
2) Rs. 55 crore
3) Rs. 46 crore
4) Rs. 35 crore
5) None of these
3. What is the ratio of the expenditure of company D to company C ?
1) 80 : 11
2) 11 : 80

3) 12 : 70
4) 70 : 12
5) None of these
4. If the difference between the income of company F and company A is Rs. 750 crore, what is the expenditure of
both the
companies together ?
1) Rs. 6400 crore
2) Rs. 7400 crore
3) Rs. 8400 crore
4) Rs. 5400 crore
5) None of these
5. Which of the following companies has recorded the maximum profit in this financial year if the combined
income of these companies is Rs. 4500 crore ?
1) A
2) B
3) F
4) C
5) None of these
6. If the income of company D and B together is Rs. 270 crore then fined the profit of company B in that year ?
1) Rs. 14.25 crore
2) Rs. 15.5 crore
3) Rs. 11.75 crore
4) Rs. 12.5 crore
5) None of these
7. In how many ways 6 boys and 3 girls be seated in a row so that the three girls do not sit together ?
1) 56 6 !
2) 66 7 !
3) 46 7 !
4) 76 6 !
5) None of these
8. In how many ways 6 boys and 6 girls be seated in a row alternatively ?
1) 2 6 ! 6 !
2) 4 6 ! 6 !
3) 5 6 ! 6 !
4) 6 ! 6 !
5) None of these
1. 3
20% * 100/110 * total = 750
T = 4125
2. 2
20% = 440
1% = 440/20 = 22
3% = 66 crore
Exp = 66 * 100/120 = 55 crore
3. 1; 80 : 11
4. 4;
5% = 750 1 % = 150

45*150/125 * 100 = 5400 crore


5. 3;
6. 3;
(20 % + 10 %) = 270
10% = 90
Ib = 90
Eb = 90*100/115 = 78.26
Pb = 11.739
7. 2;
9! 7 ! 3 ! = 7 ! ( 9 8 6) = 7 ! 66
8. 1;
6!6!+6!6!=26!6!

bankersadda.com

http://www.bankersadda.com/2015/09/night-class-quant-quiz_2.html

Night Class: Quant Quiz


Direction: Study the table carefully to answer the questions that follow :
Percentage of marks obtained by different students in different subjects :

P.S. - Kindly click on the image, if it it is not visible properly!!!


1.What is the total marks obtained by Nisha in all the subject together ?
1) 256
2) 406
3) 356
4) 425
5) None of these
2.What is the approximate average marks obtained by all the students together in Geography ?
1) 48
2) 38
3) 54
4) 28
5) None of these
3. The number of students who got 70 % marks or above in all the subjects are
1) 4
2) 3
3) None
4) 6
5) None of these
4. How many students have scored more than 120 marks in Maths ?
1) 3
2) 2
3) 1
4) 4
5) None of these
5. If 120 marks are required to pass in Maths, How many students pass in the Maths exams
1) 4
2) 5

3) 4
4) 3
5) None of these
6. In which subject the overall percentage is the highest
1) Maths
2) Physics
3) Hindi
4) History
5) None of these
7.What is the ratio of total number of marks obtained by Rakesh in Physics and Chemistry together to Chinu in
the same
subjects?
1) 17 : 23
2) 23 : 17
3) 23 : 18
4) 23 : 27
5) None of these
Answers:
1. 2; 150 + 75 + 50 + 48 + 48 + 35 = 406
2. 3;
(60 92 76 72 64 60 80)/7 * 75/100 = 54
3. 3;
4. 2;
120 marks out of 150 marks = 120/150 * 100 = 80%
Ashutosh and Nisha.
5. 4;
120 marks out of 150 marks = 120/150 * 100 = 80%
Ashutosh, Rakesh and Nisha.
6. 1;
7. 2;

bankersadda.com

http://www.bankersadda.com/2015/09/night-class-quant-quiz.html

Night Class: Quant Quiz


Directions (Q. 1-5): From a class total 360 students appeared in an exam. Three fourth of these are boys.
40 % boys failed in the exam, two third of the girls are failed in the exam and remaining passed :
1.What is the ratio of the total number of girls to the number of boys who failed in the exam ?
1) 4 : 9
2) 9 : 5
3) 5 : 6
4) 6 : 3
5) None of these

2.What is the sum of the number of boys who failed and number of girls who passed in the exam ?
1) 148
2) 158
3) 128
4) 138
5) None of these
3.What is the difference between the number of boys who are passed and number of girls who are failed
?
1) 102
2) 202
3) 52
4) 302
5) None of these
4.What is the ratio of the total number of boys and girls who are passed to the number of boys and girls
who are failed ?
1) 7 : 8
2) 8 : 7
3) 9 : 17
4) 5 : 7
5) None of these
5. Total number of boys passed is approximately what percent of total number of girls in the class ?
1) 60 %
2) 120 %
3) 150 %
4) 180 %
5) 270 %
Direction (Q. 6 10): A bag contains 4 red, 5 brown and 6 white balls. Three balls are drawn randomly :
6.What is the probability that balls drawn contains exactly two red balls ?
1) 54/145
2) 66/455
3) 46/105
4) 31/455

5) None of these
7.What is the probability that the balls drawn contains no brown ball ?
I) 11/65
2) 31/91
3) 24/91
4) 31/455
5) None of these
8. What is the probability that the balls drawn are not of the same colour ?
1) 34/455
2) 31/105
3) 74/105
4) 421/455
5) None of these
9. If two balls are drawn randomly then what is the probability that both the balls are of same colour ?
1) 31/105
2) 74/105
3) 34/455
4) 421/455
5) None of these
10.What is the probability that the three balls drawn are of different colour ?
1) 21/91
2) 31/105
3) 24/105
4) 24/91
5) None of these

Answers:
1. 3
Ratio: 90/108 = 5/6
2. 4
Sum = 108 + 30 = 138
3. 1
Diff = 162 - 60 = 102
4. 2
Ratio: 192/168 = 8/7
5. 4
Req% = 162/90 * 100 = 180%
6. 2
n(S) = 15C3 = (15 * 14 * 13)/6 = 455
2 red balls can be selected in 4C2 = 6 ways
And remaining one ball can be selected in 11C1 = 11 ways

P(E) = (6*11)/455 = 66/455


7. 3
n(S) = 15C3 = 455
Three balls have to be selected from 4 red and 6 white balls
No. of ways = 10C3 = 120
P(E) = 120/455 = 24/91
8. 4
n(S) = 15C3 =455
If all three balls are of same colour, then no. of ways = 4C3 + 5C3 + 6C3 = 4 + 10 + 20 = 34
P(E) = 34/455
For being different colours P(E) = 1 - 34/455 = 421/455
9. 1
n(S) = 15C2 = 105
n(E) = 4C2 + 5C2C + 6C2 = 6 + 10 + 15 = 31
P(E) = 31/105
10. 4
n(S) = 15C3 = 455
n(E) = 4C1 + 5C1 + 6C1 = 4 * 5 * 6 = 120
P(E) = 120/455 = 24/91

bankersadda.com

http://www.bankersadda.com/2015/09/night-class-quant-quiz_17.html

Night Class: Quant Quiz


Directions (Q. 14): In each of these questions, two equations numbered I and II with variables x and y are given.
You have to solve both the equations to find the value of x and y. Give answer
1) if x > y
2) if x >= y
3) if x < y
4) if x <= y
5) if x = y or relationship between x and y cannot be determined.
1.
I. x^2 + 8x + 15 = 0
II. y^2 12y + 32 = 0
2.
I. 4x^2 + 20y + 25 = 0
II. 4y^2 + 12y + 9 = 0
3.
I. 42x^2 + 89x + 45 = 0
II. 32y^2 + 4y 21 = 0
4.
I. 12x^2 13x + 3 = 0
II. 8y^2 + 22y 21 = 0
Directions: What value should come in the place of question mark (?) in the following questions?
5. [(87.5)^2 (12.5)^2]/75 = ?
1) 75
2) 80
3) 85
4) 100
5) 120
6. 238 15 + 120% of 25 = ?
1) 3600
2) 3650
3) 3560
4) 3800
5) 3500
7. 10 [4/5 + {7/5 - (1/6 + 1/8 - 1/3)}] = ?
1) 931/121
2) 931/120
3) 951/120
4) 931/122
5) 941/120
Directions: What approximate value should come in the place of question mark (?) in the following questions?
8. (4.159 * 3400.01 * 7.94)/ (16.99 * 25.15) = ?
1) 156

2) 256
3) 310
4) 356
5) 345
9. 298 64.95 1130.48 = ?
1) 18980
2) 17240
3) 18240
4) 15580
5) 16240
10. 149% of 670 1121.98 + 129.98 = ?
1) 13
2) 23
3) 29
4) 40
5) 32
Answers:
1. 3
2. 3
3. 5
4. 5
5. 4
6. 1
7. 2
8. 2
9. 3
10. 1

bankersadda.com

http://www.bankersadda.com/2015/08/night-classes-quant-quiz.html

Night Classes : Quant Quiz


Directions (1-5): Study the following graph and answer the questions that follow:

1. What was the approximate average number of candidates who applied for Bank A over all the years
together?
(1) 4.1 lakh
(2) 5 lakh
(3) 3.1 lakh
(4) 4.5 lakh
(5) 3.8 lakh
2. Total number of candidates who applied for Bank A over all the years together was approximately what
percentage of the total number of candidates who applied for Bank B over all the years together?
(1) 79
(2) 66
(3) 70
(4) 75
(5) 60
3. In which years the total number of candidates who applied for both the banks together is exactly
equal?
(1) 2004, 2005 and 2008
(2) 2005, 2006 and 2007
(3) 2004, 2005 and 2007
(4) 2004, 2006 and 2008
(5) 2006, 2007 and 2009
4. If 20 percent of candidates who applied for Bank B qualified in the year 2008, then what was the
number of candidates who have been disqualified in the same year?
(1) 7.2 lacs
(2) 72,000
(3) 6.4 lacs
(4) 64,000
(5) None of these
5. What was the respective ratio between the total number of candidate who applied for Bank A in the
year 2006 and 2008 and the total number of candidates who applied for Bank B in the year 2005 and 2008

together?
(1) 5 : 6
(2) 6 : 7
(3) 7 : 8
(4) 9 : 10
(5) None of these
Direction (6-10) what will come in place of question mark (?) in the following series
6. 161 , 199 , 241 , 287 , 337 , ?
(1) 391
(2) 401
(3) 412
(4) 416
(5) 421
7. 141 , 2885 , 4613 , 5613 , 6125 , ?
(1) 6311
(2) 6321
(3) 6331
(4) 6341
(5) 6351
8. 1664 , 4160 , 1040 , 2600 , ? , 1625
(1) 630
(2) 640
(3) 650
(4) 660
(5) 675
9. 43.5 , 57 , 70.5 , 84 , 97.5 , ?
(1) 109
(2) 111
(3) 115
(4) 121
(5) 124
10. 5 , 87 , 601 , ? , 10785 , 26415
(1) 2775
(2) 2848
(3) 2915
(4) 3005
(5) 3135

Das könnte Ihnen auch gefallen